You are on page 1of 117

Musculoskeletal 17Mar2009

1 – Histology: Connective Tissue


1.1) Which of the following types of connective tissue falls under the category of
connective tissue proper?
a) Blood
b) Cartilage
c) Bone
d) Adipose
e) Tendons
1.2) Which of the following is the main component in connective tissue, making up about
25% of the total protein content in mammals?
a) Fibroblast
b) Collagen
c) Myelin
d) Mesenchyme
e) Elastin
2.1) Which of the following is the main functional component in tendons, ligaments, and
aponeurosis?
a) Loose connective tissue
b) Dense regular connective tissue
c) Dense irregular connective tissue
d) Tendons
e) Ligaments
2.2) Which of the following is the initial site where pathogenic bacteria have breached
the epithelial surface, is the site of inflammatory and immune reactions, and undergoes
considerable swelling in areas such as the lamina propria of mucous membranes?
a) Loose connective tissue
b) Dense regular connective tissue
c) Dense irregular connective tissue
d) Tendons
e) Ligaments
2.3) Which of the following can be found in the submucosa of hollow organs, is made of
mostly collage fibers, and can withstand stress of varying organs and tissues?
a) Loose connective tissue
b) Dense regular connective tissue
c) Dense irregular connective tissue
d) Tendons
e) Ligaments
Match the definition with the term:
3.1) Fibrous sheet, multiple orthogonal layers, cornea of eye a) Tendons
3.2) Fibrous cord, stellate H&E cross-section, muscle to bone b) Ligaments
3.3) Band or sheet, fibrous tissue, parallel layers, bone to bone c) Aponeurosis
4.1) What type of collagen forms a nonfibrillar network (no banded fibrils) that provides
adhesion to the basement membrane?
a) Type I
b) Type II
c) Type III

DO NOT DISTRIBUTE -1-


Musculoskeletal 17Mar2009

d) Type IV
e) Type V
f) Type XI
4.2) What type of bonding gives collagen its strength?
a) Electrostatic
b) Van der Waals
c) Head to tail attachments
d) Hydrogen
e) Covalent
4.3) What vitamin is required for the function of prolyl hydroxylase and lysyl
hydroxylase, which are involved in the formation and stabilization of collagen?
a) Vitamin A
b) Vitamin B
c) Vitamin C
d) Vitamin D
e) Vitamin E
4.4) Reticular fibers, like collagen fibers, exhibit a 68nm banding pattern and provide a
supporting framework for the cellular constituents of various tissues and organs.
Reticular fibers are mostly composed of what type of collagen?
a) Type I
b) Type II
c) Type III
d) Type IV
e) Type V
4.5) Elastin molecules have an unusual polypeptide backbone that causes random coiling.
This allows for what function of elastic fibers?
a) Ability to undergo great sheer strength
b) Ability to support great weight
c) Ability to prevent changes in shape
d) Ability to stretch and distend
4.6) Unlike collagen, elastin lacks:
a) Proline
b) Glycine
c) Hydroxyproline
d) Hydroxylysine
e) A & B
f) C & D
4.7) Which of the following locations would NOT contains a great deal of extracellular
elastic material?
a) Larynx
b) Vertebral ligaments
c) Arteries
d) Costal rib junctions
5.1) Collagen, the most abundant type of connective tissue fiber, contains ____
polypeptides known as alpha chains, which intertwine to form a right-handed ____ helix.
a) Two; Double

DO NOT DISTRIBUTE -2-


Musculoskeletal 17Mar2009

b) Two; Triple
c) Three; Double
d) Three; Triple
5.2) In a collagen fiber, every third amino acid is a ____ molecule (except at the ends)
and is usually preceded by a hydroxyproline or hydroxylysine, and is usually followed by
a proline.
a) Proline
b) Lysine
c) Alanine
d) Valine
e) Glycine
5.3) What is true of the collagen fibrils in Type II collagen?
a) Thin fibrils, three identical chains
b) Thick fibrils, three identical chains
c) Thin fibrils, two identical chains and one different chain
d) Thick fibrils, two identical chains and one different chain
e) Thin fibrils, three different chains
6.1) Which of the following include type IV collagen?
a) Fibrillar collagens
b) Fibril-associated collagens with interrupted triple helixes (FACITs)
c) Hexagonal network-forming collagens
d) Transmembrane collagens
e) Multiplexins (collagens with multiple triple-helix domains and interruptions)
f) Basement membrane-forming collagens
6.2) Which of the following have gaps in their triple helixes that provide flexibility and
include types IX, XII, XIV, XVI, XIX, XX, and XXI collagens.
a) Fibrillar collagens
b) Fibril-associated collagens with interrupted triple helixes (FACITs)
c) Hexagonal network-forming collagens
d) Transmembrane collagens
e) Multiplexins (collagens with multiple triple-helix domains and interruptions)
f) Basement membrane-forming collagens
6.3) Which of the following comprises collagen types XV and XVIII, which reside in the
basement membrane zones?
a) Fibrillar collagens
b) Fibril-associated collagens with interrupted triple helixes (FACITs)
c) Hexagonal network-forming collagens
d) Transmembrane collagens
e) Multiplexins (collagens with multiple triple-helix domains and interruptions)
f) Basement membrane-forming collagens
6.4) Which of the following is characterized by uninterrupted glycine-proline-
hydroxyproline repeats, aggregates to form 68nm banded fibrils, and includes types I, II,
III, V, and XI collagen molecules?
a) Fibrillar collagens
b) Fibril-associated collagens with interrupted triple helixes (FACITs)
c) Hexagonal network-forming collagens

DO NOT DISTRIBUTE -3-


Musculoskeletal 17Mar2009

d) Transmembrane collagens
e) Multiplexins (collagens with multiple triple-helix domains and interruptions)
f) Basement membrane-forming collagens
6.5) Which of the following is represented by collagen types VIII and X?
a) Fibrillar collagens
b) Fibril-associated collagens with interrupted triple helixes (FACITs)
c) Hexagonal network-forming collagens
d) Transmembrane collagens
e) Multiplexins (collagens with multiple triple-helix domains and interruptions)
f) Basement membrane-forming collagens
6.6) Which of the following are types XIII, XVII (found within the hemidesmosome),
XXIII, and XXV collagen molecules?
a) Fibrillar collagens
b) Fibril-associated collagens with interrupted triple helixes (FACITs)
c) Hexagonal network-forming collagens
d) Transmembrane collagens
e) Multiplexins (collagens with multiple triple-helix domains and interruptions)
f) Basement membrane-forming collagens
7.1) The production of fibrillar collagen involves a series of events within the fibroblast
that leads to the production of procollagen, and is associated with all of the following
EXCEPT:
a) Type I collagen
b) Type II collagen
c) Type III collagen
d) Type IV collagen
e) Type V collagen
f) Type XI collagen
7.2) Long precursors called pro-!-chains (preprocollagen molecules) are synthesized in
what cellular location?
a) Nucleus
b) Lysosomes
c) rER
d) sER
e) Golgi
Match the Matrix Metalloproteinase (MMP) with what it degrades:
7.3) Type I, II, III, X collagens a) Membrane-type MMPs
7.4) Most collagens, laminin, elastin, fibronectin b) Matrilysins
7.5) Proteoglycans, fibronectin, denatured collagens c) Gelatinases
7.6) From cancer cells and have fibrinolytic activity d) Collagenases
7.7) Elastin, type IV collagen, laminin e) Stromelysins
7.8) Type IV collagen and proteoglycans f) Macrophage Metalloelastases
8) Which of the following components of the extracellular matrix (ECM) contains
dermatan sulfate, keratan sulfate, and hyaluronan?
a) Collagen
b) Elastic fibers
c) Proteoglycans

DO NOT DISTRIBUTE -4-


Musculoskeletal 17Mar2009

d) Multiadhesive glycoproteins
e) Glycosaminoglycans (GAGs)
9.1) Which of the following is true of the ground substance?
a) It occupies the space within cells
b) It has low water content
c) It is lost during histological preparation
d) It has a rough texture
e) It has a pink/purple hue prior to histological preparation
9.2) Which of the following is NOT a component of the ground substance?
a) Collagen
b) Glycosaminoglycans
c) Proteoglycans
d) Multiadhesive glycoproteins
10.1) What characteristic most differentiates myofibroblasts from fibroblasts?
a) Amount of rER
b) Endocytotic vesicles
c) Thin filaments
d) Golgi apparatus size
e) Lysosomes
10.2) Which of the following are phagocytotic cells derived from monocytes?
a) Macrophages
b) Mast cells
c) Fibroblasts
d) Adipose cells
e) Mesenchymal stem cells
10.3) Which of the following is the principal cell of connective tissue, synthesizes
collagen, and in some locations provides a replicating population of cells?
a) Fibroblasts
b) Macrophages
c) Adipose cells
d) Mast cells
e) Mesenchymal stem cells
10.4) Which of the following contains major histocompatibility complex II (MHC II)
molecules, which allow them to interact with helper CD4 T-lymphocytes?
a) Neutrophils
b) Macrophages
c) Adipose cells
d) Mast cells
e) Plasma cells
10.5) Mast cell granules contain several vasoactive and immunoreactive substances.
Which of the following granule substances is used as a marker for mast cell activation?
a) Histamine
b) Heparin
c) Leukotrienes C, D, E
d) Eosinophil chemotactic factor
e) Trypase, serine protease

DO NOT DISTRIBUTE -5-


Musculoskeletal 17Mar2009

10.6) Which of the following is involved in the synthesis of a variety of hormones,


inflammatory mediators, and growth factors?
a) Mast cells
b) Adipose cells
c) Macrophages
d) Fibroblasts
e) Mesenchymal stem cells
10.7) In what anatomical location are mast cells absent?
a) Liver
b) Kidney
c) Skin
d) Spleen
e) Brain
10.8) What type of cells are vascular pericytes (adventitial cells or perivascular cells)
found around capillaries and venules?
a) Mast cells
b) Adipose cells
c) Macrophages
d) Fibroblasts
e) Mesenchymal stem cells
11.1) Identify this tissue >>>>>>>>>>>
a) Loose connective
b) Dense regular connective
c) Dense irregular connective
11.2) Identify this tissue >>>>>>>>>>>>>>>>>>>>>>>>>>>
a) Loose connective
b) Dense regular connective
c) Dense irregular connective
11.3) Identify this tissue >>>>>>>>>>>
a) Loose connective
b) Dense regular connective
c) Dense irregular connective
11.4) Identify this image >>>>>>>>>>>>>>>>>>>>>>>>>>
a) Mast cell
b) Adipose cell
c) Macrophage
d) Fibroblasts
e) Mesenchymal stem cell
f) Collagen fiber
11.5) Identify this image >>>>>>>>>
a) Mast cell
b) Adipose cell
c) Macrophage
d) Fibroblasts
e) Mesenchymal stem cell
f) Collagen fiber

DO NOT DISTRIBUTE -6-


Musculoskeletal 17Mar2009

11.6) Identify this image >>>>>>>>>>>>>>>>>>>>>>>>>>>>


a) Mast cell
b) Adipose cell
c) Macrophage
d) Fibroblasts
e) Mesenchymal stem cell
f) Collagen fiber
11.7) Identify this image >>>>>>>
a) Mast cell
b) Adipose cell
c) Macrophage
d) Fibroblasts
e) Mesenchymal stem cell
f) Collagen fiber

2 – Histology: Cartilage
1.1) About what percentage of cartilage volume consists of extracellular matrix?
a) 5%
b) 25%
c) 50%
d) 75%
e) 95%
Match the description with the type of cartilage: a) Hyaline b) Elastic c) Fibrous
1.2) Found in specific ear and auditory locations and provides flexible support
1.3) Resists deformation under stress for intervertebral disks and pubic symphysis
1.4) Provides cushioning in synovial joints, epiphyseal plates, larynx, and fetal skeleton
1.5) Which type of cartilage does NOT have a perichondrium?
1.6) What type of cartilage does NOT undergo calcification?
2.1) What type of hyaline cartilage molecules have clinical value as markers of cartilage
turnover and degradation, such as in anchorin CII (cartilage annexin V)?
a) Collagen molecules
b) Proteoglycans
c) Multiadhesive glycoproteins
2.2) What type of collagen molecules are predominantly found in hyaline, as well as
other types, of cartilage?
a) Type I
b) Type II
c) Type III
d) Type IV
2.3) What type of hyaline cartilage molecules form the ground substance and help
prevent dehydration of the cartilage?
a) Collagen molecules
b) Proteoglycans
c) Multiadhesive glycoproteins
3.1) In hyaline cartilage, chondrocytes are distributed either singly or in clusters called
isogenous groups, which represent:

DO NOT DISTRIBUTE -7-


Musculoskeletal 17Mar2009

a) Cells that are dying


b) Cells that are in steady state
c) Cells that have recently divided
d) Cells that will no longer divide
3.2) Newly divided chondrocytes secrete metalloproteinases, which:
a) Help form the cartilage matrix
b) Prevent destruction of the cartilage matrix
c) Destroy lytic components of the cartilage matrix
d) Degrade the cartilage matrix
3.3) Which type cartilage has the fewest chondrocytes when viewed histologically?
a) Hyaline cartilage
b) Elastic cartilage
c) Fibrocartilage
3.4) Chondrocytes that are active in matrix production display areas of cytoplasmic:
a) Basophilia
b) Eosinophilia
4) Perichondrium is a dense connective tissue that functions in cartilage growth and
repair. What type of hyaline cartilage does NOT contain a perichondrium, which can
contribute to osteoarthritis?
a) Cricoid cartilage
b) External ear and auditory canal
c) Synovial joints and intervertebral disks
d) Costal cartilage of the rib cage
e) Articular joint surfaces and epiphyseal plates
f) A & D
5.1) Cartilage does not heal well after injury due to avascularity, chondrocyte immobility,
and the limited ability of mature chondrocytes to proliferate. Repair of cartilage mostly
involves the production of what type of tissue?
a) Dense connective
b) Loose connective
c) Elastic cartilage
d) Fibrocartilage
e) Bone
5.2) Hyaline cartilage is prone to calcification, being replaced by what type of tissue?
a) Dense connective
b) Loose connective
c) Elastic cartilage
d) Fibrocartilage
e) Bone
6.1) Identify this cartilage >>>>
a) Hyaline
b) Elastic
c) Fibrous
6.2) Identify this cartilage >>>>>>>>>>>>>>>>>>>>>>>>>
a) Hyaline
b) Elastic

DO NOT DISTRIBUTE -8-


Musculoskeletal 17Mar2009

c) Fibrous
6.3) Identify this cartilage >>>>
a) Hyaline
b) Elastic
c) Fibrous

3 – Histology: Bone
1.1) What mineral in the form of hydroxyapetite crystals differentiates bone from other
connective tissues?
a) Magnesium bisphosphate
b) Magnesium phosphate
c) Calcium bisphosphate
d) Calcium phosphate
e) Sodium permanganate
1.2) Which of the following is the major structural component of bone?
a) Type I collagen
b) Type II collagen
c) Type III collagen
d) Type IV collagen
e) Type VI collagen
1.3) Which of the following plays a major role in binding calcium during the
mineralization process of bone?
a) Cytokines
b) Osteocalcin
c) Osteonectin
d) Glycosaminoglycans
e) Sialoproteins
1.4) Which of the following is referred to as an osteocyte once the cell is surrounded with
its secreted extracellular bone matrix?
a) Endosteum
b) Osteoprogenitor cell
c) Osteoblast
d) Bone-lining cell
e) Osteoclast
1.5) Which of the following bones is considered a short bone?
a) Tibia
b) Carpals
c) Metacarpals
d) Sternum
e) Ethmoid
1.6) Bones are covered with a periosteum in all areas except when covered by:
a) Dense connective
b) Loose connective
c) Elastic cartilage
d) Fibrocartilage
e) Hyline cartilage

DO NOT DISTRIBUTE -9-


Musculoskeletal 17Mar2009

1.7) In what location would you find Sharpey fibers?


a) Joint articulations
b) Bone marrow
c) Ligament insertions
d) Tendon insertions
e) A & B
f) C & D
1.8) Which of the following lines tissue of both the compact bone facing the marrow
cavity and the trabeculae of spongy bone within the cavity?
a) Endosteum
b) Osteoprogenitor cells
c) Osteoblasts
d) Bone-lining cells
e) Osteoclasts
1.9) Which of the following arises developmentally from periosteal vessels that become
incorporated into the bone, and not from periosteal buds?
a) Epiphyseal artery
b) Metaphyseal artery
c) Nutrient artery
d) Periosteal artery
1.10) Which of the following is NOT true of immature bone?
a) Does not display an organized lamellated appearance
b) Contains more cells per unit than mature bone
c) Cell are randomly arranged, unlike mature bone
d) Matrix contains more ground substance and stains more intensely with
hemotoxylin
e) Is more heavily mineralized than mature bone
2.1) The Haversian canal contains the ____ supply of the bone osteon.
a) Lymphatic
b) Vascular
c) Nerve
d) A & B
e) B & C
2.2) Blood vessels and nerves travel from the periosteal later to the endosteal layer
through which of the following?
a) Lamellae
b) Osteon
c) Haversian canal
d) Volkmann canal
e) Lacuna
3.1) The transcription factor core binding factor alpha-1 (CBFA1) has what function?
a) Triggers differentiation of osteoprogenitor cells
b) Triggers destruction of osteoblasts
c) Triggers destruction of osteoclasts
d) Triggers increased osteoclast activity

DO NOT DISTRIBUTE - 10 -
Musculoskeletal 17Mar2009

3.2) Which of the following is responsible for calcification of the matrix by releasing
matrix vesicles into the matrix?
a) Osteocytes
b) Osteoprogenitor cells
c) Osteoblasts
d) Bone-lining cells
e) Osteoclasts
3.3) Which of the following is derived from mononuclear hemopoetic cells, namely
colony-forming unit granulo-monocytes (CFU-GM)?
a) Osteocytes
b) Osteoprogenitor cells
c) Osteoblasts
d) Bone-lining cells
e) Osteoclasts
3.4) Which of the following is a mature bone cell that is enclosed by bone matrix
(osteoid)?
a) Osteocytes
b) Osteoprogenitor cells
c) Osteoblasts
d) Osteoclasts
4.1) Which of the following bones would form, in general, by intramembranous
ossification?
a) Humerus
b) Femur
c) Radius
d) Pelvis
e) Skull
4.2) When a mesenchymal cell has differentiated during intramembranous ossification, it
expresses Cbfa1 transcription factor and is called an:
a) Osteocyte
b) Osteoprogenitor cell
c) Osteoblast
d) Osteoclast
4.3) Which of the following is the correct order of appositional bone growth?
a) Cartilage model => Periosteal collar => Cartilaginous matrix = > Blood vessels
b) Cartilage model => Periosteal collar = > Blood vessels => Cartilaginous matrix
c) Cartilage model => Blood vessels => Periosteal collar = > Cartilaginous matrix
d) Cartilaginous matrix => Periosteal collar => Blood vessels => Cartilage model
e) Cartilaginous matrix => Blood vessels => Periosteal collar => Cartilage model
4.4) What is the first sign of bone ossification?
a) Appearance of blood vessels
b) Appearance of a cuff of bone around the cartilage model
c) Appearance of a hyaline cartilage model
d) Lack of CFU-GMs
e) Decrease in Cbfa1 transcription factor
4.5) Growth in length of long bones depends on the presence of:

DO NOT DISTRIBUTE - 11 -
Musculoskeletal 17Mar2009

a) Osteoclasts
b) Diaphyseal cartilage
c) Epiphyseal cartilage
d) Periosteum
e) Metaphyseal cartilage
4.6) Which of the following long-bone zones is closest to the epiphysis?
a) Zone of reserve cartilage
b) Zone of proliferation
c) Zone of hypertrophy
d) Zone of calcified cartilage
e) Zone of resorption
4.7) In the development of new osteons, a tunnel is bored through compact bone by an
advancing cutting cone, which is made of:
a) Osteocytes
b) Osteoprogenitor cells
c) Osteoblasts
d) Osteoclasts
5) During bone mineralization, ____ Ca++ concentrations stimulate the osteoblast to
secrete alkaline phosphatase, which ____ the local concentration of PO4- ions.
a) High; Decrease
b) Low; Decrease
c) High; Increase
d) Low; Increase
6.1) Parathyroid hormone (PTH) acts on the bone to ____ blood calcium levels and
calcitonin acts to ____ blood calcium levels.
a) Increase; Increase
b) Decrease; Decrease
c) Increase; Decrease
d) Decrease; Increase
6.2) Bone is not able to repair itself after injury and thus requires immunologic cells to
remove injured tissue.
a) True
b) False, bone can repair itself
c) False, bone does not need repair in injured
7.1) Identify the Volkmann canal in this image: >>>>
a) A
b) B
c) C
d) D
e) E
7.2) Identify the periosteum in this image: >>>>>>>>
a) A
b) B
c) C
d) D
e) E

DO NOT DISTRIBUTE - 12 -
Musculoskeletal 17Mar2009

7.3) What is shown at the center of this image >>>>>>


a) Periosteum
b) Volkmann canal
c) Haversian canal
d) Lamellae
e) Lucuna
7.4) Identify the tissue in this image >>>>>>>>>>>>>
a) Osteoclasts
b) Osteoblasts
c) Hyaline cartilage
d) Mature bone
e) Immature bone
7.5) Identify the tissue in this image >>>>>>>>>>>>>
a) Osteoclasts
b) Osteoblasts
c) Hyaline cartilage
d) Mature bone
e) Immature bone
7.6) Where would this bone most likely be found? >>>>>>>>
a) Mandible
b) Humerus
c) Femur
d) Lumbar vertebra
e) Clavicle
7.7) What long-bone zone is shown? >>>>
a) Zone of reserve cartilage
b) Zone of proliferation
c) Zone of hypertrophy
d) Zone of calcified cartilage
e) Zone of resorption

4 – Histology: Muscle
1) Muscle thick filaments (about 15nm) are made of:
a) F-actin
b) G-actin
c) Myosin I
d) Myosin II
e) Glycosaminoglycans
2) Muscle is classified on the basis of the appearance of:
a) Fibroblasts
b) Actin
c) Myosin
d) Calmodulin
e) Contractile cells
3) The tongue would be classified as:
a) Smooth muscle

DO NOT DISTRIBUTE - 13 -
Musculoskeletal 17Mar2009

b) Skeletal muscle
c) Visceral muscle
d) Cardiac muscle
4) Cross-striations are seen due to the specific cytoarchitectural arrangement of both thin
and thick filaments. Which of the following would NOT exhibit this pattern?
a) Smooth muscle
b) Skeletal muscle
c) Visceral muscle
d) Cardiac muscle
5.1) Which of the following is the thick connective tissue layer that surrounds a group of
muscle fibers to form a bundle or fascicle?
a) Muscle fiber
b) Myoblast
c) Endomysium
d) Perimysium
e) Epimysium
5.2) The sarcolemma is made of the cytoplasmic boundary of the muscle cell (plasma
membrane).
a) True
b) False, it also contains the external (basal) lamina
c) False, it also contains the external (basal) lamina and surrounding reticular
layer
6) Red muscle fibers have ____ cytochrome, ____ myoglobin, and ____ mitochondria.
a) Little; Little; Many
b) Little; Little Few
c) Little; Large amounts of; Many
d) Large amounts of; Little; Few
e) Large amounts of; Large amounts of; Many
7.1) Compared with slow-twitch motor units, fast-twitch motor units show ____
resistance to fatigue and generate ____ muscle tension.
a) Great; Low
b) Little; Low
c) Great; High
d) Little; High
7.2) with slow-twitch motor units, fast-twitch motor units have ____ ATPase activity and
are found in the ____.
a) Great; Digits and extraocular muscles
b) Low; Digits and extraocular muscles
c) Great; Limbs
d) Low; Limbs
8.1) Which of the following bisects the I band?
a) A band
b) M line
c) Z line
d) H zone
8.2) Which of the following bisects the A band?

DO NOT DISTRIBUTE - 14 -
Musculoskeletal 17Mar2009

a) M line
b) I band
c) Z line
d) H zone
8.3) Which of the following bisects the H zone?
a) A band
b) I band
c) Z line
d) M line
9.1) Thin filaments contain:
a) G-actin and tropomyosin (TNC, TNI, TNT)
b) Titin and alpha-actinin
c) Desmin and nebulin
d) Tropomodulin
e) Dystrophin
9.2) Myosin II is composed of ____ polypeptide heavy chains and ____ light chains.
a) 1; 2
b) 2; 3
c) 2; 4
d) 4; 3
e) 4; 2
9.3) Troponin I (TNI) binds to:
a) Actin
b) Myosin
c) Calmodulin
d) Tropomysin
e) Calcium
9.4) The bare zone in the middle of the thick filament where there is no globular head, is
the location of the:
a) A band
b) I band
c) Z line
d) M line
e) H zone
Match the accessory protein with the description:
10.1) Anchors thin filaments to Z line a) Alpha actinin & nebulin
10.2) Actin binding protein b) Dystrophin
10.3) Attaches sarcomere to Z line c) Myomesin & C protein
10.4) Holds thick filaments at the M line d) Tropomodulin
10.5) When absent, causes DMD e) Tinin
10.6) Anchors thick filaments to Z line f) Desmin
11.1) Which of the following best describes rigor mortis (post death state), where ATP is
not present?
a) The myosin head is tightly bound to the actin molecule
b) The myosin head has uncoupled from the actin molecule
c) The myosin head is uncoupled and bent (5nm movement)

DO NOT DISTRIBUTE - 15 -
Musculoskeletal 17Mar2009

d) The myosin head is uncoupled and straightened (power stroke)


11.2) The sarcoplasmic reticulum (SR) and transverse tubule system (TTS) work to
rapidly deliver and remove which of the following?
a) TNT
b) TNI
c) TNC
d) Calcium
e) Calmodulin
11.3) In skeletal muscle, the cisternae (Ca++ reservoir) and T-tubule complex is formed
with ____ cisternae and ____ T-tubule(s).
a) 1; 1
b) 2; 2
c) 2; 1
d) 1; 2
e) 3; 3
12.1) Release of which of the following into the neuromuscular junction (NMJ) synaptic
cleft initiates muscle cell depolarization and contraction?
a) Calmodulin
b) Calcium (Ca++)
c) Acetylcholine (ACh)
d) Acetylcholine Esterase (AChE)
e) Sodium (Na+)
12.2) In what location would the ratio of neurons to muscle fibers be the smallest, such as
1 neuron to 3 fibers?
a) Back muscles
b) Eye muscles
c) Mastication muscles
d) Biceps brachii
e) Iliopsoas
12.3) Once acetylcholine traverses the NJM and depolarizes the sarcolemma, which of
the following ion exchanges occurs first?
a) Ca++ enters the cell
b) K+ enters the cell
c) K+ exits the cell
d) Na+ enters the cell
e) Na+ exits the cell
13.1) What type of cell is responsible for myotubes that extend between tendons of
developing muscle, developing muscle fibers with multiple central nuclei?
a) Satellite cells
b) Early myoblasts
c) Late myoblasts
13.2) What is the function of satellite cells in muscle injury?
a) They form myotubes in the intervated zone of muscle
b) They form myotubes between muscle tendons
c) They provide a structural mesh-like framework for muscle rebuilding
d) They function as stem cells giving rise to new my0blasts

DO NOT DISTRIBUTE - 16 -
Musculoskeletal 17Mar2009

14.1) What is shown in this image? >>>>>>>>>>>>>>>>>>>>


a) Smooth muscle
b) Skeletal muscle
c) Cardiac muscle
d) Musculotendinous junction
e) Neuromuscular junction
14.2) What is shown in this image? >>>>>>
a) Smooth muscle
b) Skeletal muscle
c) Cardiac muscle
d) Musculotendinous junction
e) Neuromuscular junction

5 – Biochemistry & Genetics: Musculoskeletal System


1.1) Which of the following X-linked recessive muscle disorders is the most rare?
a) Duchenne Muscular Dystrophy (DMD)
b) Becker Muscular Dystrophy (BMD)
c) Emery-Dreifuss Muscular Dystrophy (EMDM)
1.2) Which of the following autosomal muscle disorders involves mutations of one of the
three genes coding for type VI collagen?
a) Myotonic Muscular Dystrophy (MMD)
b) Facioscapulohumeral Dystrophy (FSHD)
c) Limb-Girdle Muscular Dystrophy (LGMD)
d) Bethlem Myopathy
1.3) DMD is the most severe and most common form of X-linked recessive muscle
disorders. BMD is a milder form, characterized by:
a) Brittle bones and arthritis
b) Later onset and faster progression
c) Quicker onset and faster progression
d) Later onset and slower progression
e) Quicker onset and slower progression
1.4) Duchenne Muscular Dystrophy (DMD) affects approximately how many males?
a) 1 in 3,500
b) 1 in 8,000
c) 1 in 20,000
d) 1 in 35,000
e) 1 in 80,000
1.5) Becker Muscular Dystrophy (BMD) affects approximately how many males?
a) 1 in 3,500
b) 1 in 8,000
c) 1 in 20,000
d) 1 in 35,000
e) 1 in 80,000
1.6) Although heterozygous carries of DMD are free of the disease, about what
percentage will have some muscle weakness?
a) 10%

DO NOT DISTRIBUTE - 17 -
Musculoskeletal 17Mar2009

b) 25%
c) 50%
d) 75%
e) 90%
1.7) A 10-year-old boy presents to the primary care office in a wheelchair. Physical exam
reveals enlarged calf muscles due to pseudohypertrophy. This patient is likely to survive
until their late teens or early twenties, as they most likely have:
a) Bethlem Myopathy
b) Myotonic Muscular Dystrophy (MMD)
c) Limb-Girdle Muscular Dystrophy (LGMD)
d) Facioscapulohumeral Dystrophy (FSHD)
e) Duchenne Muscular Dystrophy (DMD)
1.8) What gene is responsible for the basic defects in both DMD and BMD?
a) Long arm of the X-chromosome
b) Short arm of the X-chromosome
c) Long arm of the Y-chromosome
d) Short arm of the Y-chromosome
1.9) What protein which is nearly undetectable in most cases of DMD, is reduced in
amount or size in BMD, and which is encoded by less than 1% of the gene with the
remaining 99% being occupied by introns.
a) Desmin
b) Sarcoglycan
c) Synemin
d) Dystrophin
e) Myotonic protein kinase
1.10) Deletions and duplications of the gene responsible for DMD generally result from
what type of mutation?
a) Silent
b) Frameshift
c) Nonsense
d) Missense
e) Non-frameshift
1.11) What type of mutations are typically associated with BMD?
a) Silent
b) Frameshift
c) Nonsense
d) Missense
e) Non-frameshift
1.12) The muscle protein gene that is mutated in DMD patients is expressed in the brain.
What affect does this have on DMD patients?
a) Parkinson-like symptoms
b) Seizures
c) Spontaneous fainting
d) Low IQ
e) Autism

DO NOT DISTRIBUTE - 18 -
Musculoskeletal 17Mar2009

1.13) Deficiency of the intracellular protein dystrophin may lead to increased fragility of
the dystrophic muscle and possible permeability to what ion?
a) Na+
b) Cl-
c) Ca++
d) Mg++
e) K+
1.14) Which of the following enzymes in serum is markedly elevated (50-100 times
normal) in DMD?
a) ALT - Alamine aminotransferase
b) AST - Aspartate
c) CK - Creatine kinase
d) LDH - Lactate dehydrogenase
e) None are usually elevated
1.15) Both carrier detection and fetal diagnosis of DMD can be achieved by DNA testing,
employing which of the following?
a) Southern blotting
b) Northern blotting
c) Western blotting
d) Hybridization
e) PCR
1.16) Which of the following dystrophies can be autosomal dominant or recessive?
a) Myotonic Muscular Dystrophy (MMD)
b) Facioscapulohumeral Dystrophy (FSHD)
c) Limb-Girdle Muscular Dystrophy (LGMD)
1.22) Severity and onset of MMD is highly correlated to:
a) Chromosome 13
b) The number of amino acid deletions
c) The number of amino acid expansion
d) The arm of the affected chromosome
e) The size of the mutated protein
1.18) Which of the following, in congenital form, is inherited only from the mother and
affects about 20% of the mother’s offspring?
a) Duchenne Muscular Dystrophy (DMD)
b) Becker Muscular Dystrophy (BMD)
c) Myotonic Muscular Dystrophy (MMD)
d) Facioscapulohumeral Dystrophy (FSHD)
e) Limb-Girdle Muscular Dystrophy (LGMD)
1.19) Which of the following clinical manifestations of MMD is the first to be seen?
a) Frontal balding and testicular atrophy
b) Mask-like face with drooping eyelids
c) Cataracts and diabetes mellitus
d) Smooth muscle myotonia
e) Cardiac myotonia
1.20) Which of the following enzymes in serum is elevated in MMD?
a) ALT - Alamine aminotransferase

DO NOT DISTRIBUTE - 19 -
Musculoskeletal 17Mar2009

b) AST - Aspartate
c) CK - Creatine kinase
d) LDH - Lactate dehydrogenase
e) None are usually elevated
1.21) The gene defective in MMD has been mapped to what chromosome?
a) 13
b) 19
c) 21
d) 23
e) 4
1.22) Molecular diagnosis of MMD is done by using primers to flank the CGT (or GCT)
sequence. For expansions too large for PCR methods, what method is used?
a) Southern blotting
b) Northern blotting
c) Western blotting
d) Hybridization
1.23) A 21-year-old female patient presents with facial muscle fatigue and upper arm
weakness. She came to the clinic due to new weakness in her legs. Lab tests reveal CD
levels elevated about 4 times normal. Which of the following is the most likely?
a) Duchenne Muscular Dystrophy (DMD)
b) Becker Muscular Dystrophy (BMD)
c) Myotonic Muscular Dystrophy (MMD)
d) Facioscapulohumeral Dystrophy (FSHD)
e) Limb-Girdle Muscular Dystrophy (LGMD)
1.24) The long arm of what chromosome has the gene responsible for FSHD?
a) 4
b) 13
c) 19
d) 21
e) 23
1.25) LGMD has varied onset and symptoms, although the autosomal recessive form is
more common (childhood onset). Four of the most commonly occurring recessive forms
are a consequence of mutations in genes encoding for:
a) Desmin
b) Sarcoglycan
c) Synemin
d) Dystrophin
e) Myotonic protein kinase
1.26) CK levels in LGMD would be highest in what situation?
a) Infants
b) Adult patients
c) Dominant disorder patients
d) Recessive disorder patients
1.27) A Dutch patient presents with proximal muscle weakness and joint contractures.
CK levels are slightly elevated. The physician suspects an autosomal dominant disorder,
affecting chromosome 21. What disorder is the physician suspecting?

DO NOT DISTRIBUTE - 20 -
Musculoskeletal 17Mar2009

a) Myotonic Muscular Dystrophy (MMD)


b) Facioscapulohumeral Dystrophy (FSHD)
c) Limb-Girdle Muscular Dystrophy (LGMD)
d) Bethlem Myopathy
e) Oculopharyngeal Muscular Dystrophy (OPMD)
1.28) A French-Canadian patient presents at age 50 with drooping eyelids, weakness in
facial and pharyngeal muscles, and slightly elevated CK levels. Which of the following is
the most likely?
a) Myotonic Muscular Dystrophy (MMD)
b) Facioscapulohumeral Dystrophy (FSHD)
c) Limb-Girdle Muscular Dystrophy (LGMD)
d) Bethlem Myopathy
e) Oculopharyngeal Muscular Dystrophy (OPMD)
1.29) A Bukhara Jew from Isreal presents with apparent autosomal dominant OPMD.
What is the biochemical basis for this disorder, which causes protein molecules to clump
in the cell nuclei?
a) Expansion of an AT repeat
b) Expansion of a GC repeat
c) Deletion of an AT sequence
d) Deletion of a GC sequence
2.1) Myotonic Muscular Dystrophy (MMD) signs usually do not appear until after
adolescence and progress slowly. However, there is a tendency for the disorder to be
expressed earlier and become more severe from generation to generation, known as:
a) Anticipation
b) Pleiotropy
c) Genomic imprinting
d) Heterogeneity
e) Repeat expansion
2.2) Similar to Fragile X syndrome and Huntington disease, myotonic dystrophy results
from what genetic defect, involving CGT (or GCT) in MMD?
a) Doublet repeat deletion
b) Double repeat expansion
c) Triplet repeat deletion
d) Triple repeat expansion
2.3) What protein is affected in MMD which may modify signal transduction, calcium
ion flux, and increase risk of anaesthetic-induced malignant hyperthermia?
a) Desmin
b) Sarcoglycan
c) Synemin
d) Dystrophin
e) Myotonic protein kinase
3.1) Malignant Hyperthermia (MH) is a severe reaction to certain anaesthetics, such as
halothane, that had 80% fatality prior to the use of dantrolene (now about 10% fatality).
Which of the following is NOT a result of a MH reaction?
a) Muscle rigidity
b) Increased body temperature

DO NOT DISTRIBUTE - 21 -
Musculoskeletal 17Mar2009

c) Acidosis
d) Bradycardia
3.2) In swine, MH has been linked to a point mutation (cys for arg) in what area of the
sarcoplasmic reticulum?
a) Sodium release channel
b) Calcium release channel
c) Potassium release channel
d) Na+/K+ ATPase
e) Myosin ATPase
4.1) Opening of a defective calcium release channel appears to be prolonged in the
presence of halothane or muscle relaxants, leading to calcium ions flooding the
sarcoplasm. This flooding stimulates myosin ATPase, causing what symptom?
a) Increased heart rate
b) Increased heat
c) Increased ATP production
d) Muscle rigidity
e) Acidosis
4.2) During a MH reaction, stimulation of glycogen catabolism (by calcium ions), and of
aerobic mitrochondrial metabolism, produces ATP, CO2, and heat, thus leading to:
a) Muscle rigidity
b) Increased body temperature
c) Acidosis
d) All of the above

6 – Physiology: Exercise & Musculoskeletal System


1.1) A mailman is chased down the street by a customer’s pit-bull. Which of the
following terms best describes what the mailman is doing?
a) Physical activity
b) Exercise
c) Physical conditioning
d) Screaming
e) Crying
1.2) Physical conditioning enables toe body to meet and handle progressively greater
stresses with:
a) Short-term damage being done to the body
b) Long-term damage being done to the body
c) No short-term damage being done to the body
d) No long-term damage being done to the body
e) Great stress and vast alterations of homeostasis
1.3) Unlike exercise, physical activity does not necessarily need to meet planned goals.
Both involve the expenditure of energy via movement of:
a) Cardiac muscle
b) Skeletal muscle
c) Smooth muscle
d) Bones
e) None of the above

DO NOT DISTRIBUTE - 22 -
Musculoskeletal 17Mar2009

2.1) Which of the following is characteristic of health-related physical fitness as opposed


to motor-performance physical fitness?
a) Cardio kickboxing
b) Chin-ups
c) Sit-ups
d) 50-yard dash
e) Standing long-jump
2.2) To a greater degree than motor-performance physical fitness, health-related physical
fitness has a genetic component.
a) True
b) False
3.1) Which of the following is commonly used to measure cardio-respiratory (aerobic)
fitness, such as during continual high intensity activity?
a) Heart rate (HRmax)
b) Cardiac stroke volume (SV)
c) Cardiac output (CO)
d) Oxygen utilization (VO2max)
e) a-vO2 (arterial oxygenation – venous oxygenation)
3.2) Cardiac fitness is:
a) Directly related to the incidence of coronary heart disease and hypertension
b) Directly related to the incidence of coronary heart disease and indirectly related
to hypertension
c) Indirectly related to the incidence of coronary heart disease and hypertension
d) Indirectly related to the incidence of coronary heart disease and directly related
to hypertension
3.3) Cardiac output is defined as:
a) SV + HR
b) SV – HR
c) HR – SV
d) HR * SV
e) HR / SV
f) SV / HR
3.4) The volume of oxygen used per minute is defined as:
a) VO2 = CO * (arterial oxygen – venous oxygen)
b) VO2 = CO + (arterial oxygen – venous oxygen)
c) VO2 = CO * (arterial oxygen + venous oxygen)
d) VO2 = CO + (arterial oxygen + venous oxygen)
3.5) What is the typical hemoglobin (Hb) content in whole blood for men (Hematocrit
about 47%)?
a) 12 mg Hb/dl
b) 14 mg Hb/dl
c) 16 mg Hb/dl
d) 19 mg Hb/dl
e) 21 mg Hb/dl
3.6) What is the typical hemoglobin (Hb) content in whole blood for women (Hematocrit
about 42%)?

DO NOT DISTRIBUTE - 23 -
Musculoskeletal 17Mar2009

a) 12 mg Hb/dl
b) 14 mg Hb/dl
c) 16 mg Hb/dl
d) 19 mg Hb/dl
e) 21 mg Hb/dl
3.7) Each gram of Hb can maximally bind 1.34 ml O2, thus, for a typical man, the
maximal oxygen content carried by the Hb in one deciliter of blood is about:
a) 12 ml oxygen
b) 14 ml oxygen
c) 16 ml oxygen
d) 19 ml oxygen
e) 21 ml oxygen
3.8) At rest, VO2 for a 70kg person is approximately:
a) (70/min * 144 ml) * (19 ml/dl – 9ml/dl) = 1.0 L/minute
b) (70/min * 72 ml) * (19 ml/dl – 4ml/dl) = 0.75 L/minute
c) (70/min * 72 ml) * (19 ml/dl – 9ml/dl) = 0.5 L/minute
d) (70/min * 144 ml) * (19 ml/dl – 14 ml/dl) = 0.5 L/minute
e) (70/min * 72 ml) * (19 ml/dl – 14 ml/dl) = 0.25 L/minute
3.9) At STPD (standard temperature and pressure, dry), a VO2max of 25 ml O2/kg/min
would correlate to a person being:
a) Unfit
b) Sedentary (average)
c) Minimally trained
d) Well trained
e) Elite athlete
3.10) At STPD, a VO2max of 50ml O2/kg/min would correlate to a person being:
a) Unfit
b) Sedentary (average)
c) Minimally trained
d) Well trained
e) Elite athlete
3.11) At STPD, a VO2max of 75ml O2/kg/min would correlate to a person being:
a) Unfit
b) Sedentary (average)
c) Minimally trained
d) Well trained
e) Elite athlete
3.12) Muscular strength and endurance is important for routine tasks, especially for the
elderly and infirmed, as well as preventing sports related injuries and a lack of these
commonly lead to:
a) Shoulder dislocation
b) Colles wrist fracture
c) Osteoarthritis
d) Lower back pain
e) Deep vein thromboses

DO NOT DISTRIBUTE - 24 -
Musculoskeletal 17Mar2009

3.13) Lack of flexibility in the lower back and posterior thigh has been shown to
contribute to:
a) Psoas syndrome
b) Piriformis syndrome
c) Gluteal pain
d) Knee and hip pain
e) Lower back pain
3.14) Which of the following is indicated by the ability of the body to maintain a constant
and non-limiting ATP concentration through combustion of energy sources?
a) Musculoskeletal fitness
b) Morphological fitness
c) Flexibility
d) Motor fitness
e) Metabolic fitness
3.15) Which of the following would NOT be a result of poor morphological fitness?
a) Type II diabetes mellitus
b) Cancer
c) Muscular dystrophy
d) Cardiovascular disease
e) Osteoporosis
3.16) Which of the following deals with body composition, such as Body Mass Index
(BMI)?
a) Musculoskeletal fitness
b) Morphological fitness
c) Flexibility
d) Motor fitness
e) Metabolic fitness
4.1) Fatigue of the peripheral nervous system and reflexes is a result of:
a) Decreased acetylcholine availability
b) Decreased number of neuromuscular junction receptors
c) Decreased number of nodes of Ranvier
d) Decreased axon conduction velocity
e) Increased saltatory conduction velocity
4.2) Cardiac output (Q) can increase about 6-fold from resting value of:
a) 2 liters/minute
b) 3 liters/minute
c) 4 liters/minute
d) 5 liters/minute
e) 6 liters/minute
4.3) What is the theoretical maximum heart rate for a 70 year old?
a) 70
b) 130
c) 150
d) 170
e) 220

DO NOT DISTRIBUTE - 25 -
Musculoskeletal 17Mar2009

4.4) Stroke Volume (SV) can rise from 70-75 ml resting to about what value with
maximal activity?
a) 80-90 ml
b) 90-120 ml
c) 120-150 ml
d) 150-180 ml
e) 180-220 ml
4.5) Which of the following is the limiting central cardiovascular factor which can be
altered by training?
a) Heart rate (HRmax)
b) Cardiac stroke volume (SV)
c) Cardiac output (CO)
d) Oxygen utilization (VO2max)
e) Arterial pressure
4.6) At what point of exercise intensity does HR replace SV as the major contributing
factor for increasing cardiac output (Q)?
a) 10% VO2max
b) 20% VO2max
c) 30% VO2max
d) 40% VO2max
e) 50% VO2max
4.7) During increased physical activity the sympathetic nervous system causes
vasoconstriction and blood will be redistributed to which area, which will receive about
80-85% of cardiac output?
a) Kidneys
b) Spleen
c) Liver
d) Muscles
e) Skin
4.8) At rest, what percentage of oxygen is extracted from the oxy-hemoglobin (HbO2)
available?
a) 5%
b) 25%
c) 50%
d) 75%
e) 95%
4.9) The Bohr effect involves a rightward shift of the oxyhemoglobin dissociation curve,
increasing off-loading of oxygen from hemoglobin, increasing the a-vO2 difference, and
only minimally affecting on-loading of oxygen in the pulmonary capillaries. Which of the
following factors would NOT be increased?
a) O2
b) CO2
c) H+
d) Temperature
4.10) Aerobic training can increase all of the following EXCEPT:
a) Mitochondrial enzyme concentrations

DO NOT DISTRIBUTE - 26 -
Musculoskeletal 17Mar2009

b) Capillary density
c) Mitochondrial number
d) Collateral circulation
e) Peripheral vascular resistance (R)
4.11) What is true of perfusion pressure (P = Q * R) as local (vasodilatory) tissue factors
overwhelm sympathetic nervous system constriction, opening vessels?
a) Perfusion pressure decreases and activity must increase
b) Perfusion pressure increases and activity must increase
c) Perfusion pressure decreases and activity must decrease
d) Perfusion pressure increases and activity must increase
5.1) Which of the following is NOT a metabolism byproduct that accumulates after
muscle contraction?
a) H+
b) Ca++
c) Pi (inorganic orthophosphate)
d) ADP
e) Lactate
5.2) Which of the following components is removed during muscle cross-bridge power
stroke?
a) Pi
b) ATP
c) ADP
d) Myosin
e) Actin
5.3) Which of the following is NOT true regarding inorganic orthophosphate (Pi)?
a) It accumulates during cross-bridge cycling
b) It accumulates with increased activity of calcium-ATPase and Na/K-ATPase
c) It stabilizes cross-bridges and allows them to produce more force
d) It decreases the calcium-sensitive contractile apparatus
e) It may alter uptake, storage, and release kinetics of the sarcoplasmic reticulum
f) It may form insoluble calcium phosphate salts when concentrations increase
5.4) Hydrogen ions (H+) interfere directly with calcium binding to ____, leading to
decreased force production and decreased calcium-sensitivity of the contractile apparatus.
a) Actin
b) Tropomyosin
c) Troponin I (TnI)
d) Troponin T (TnT)
e) Troponin C (TnC)
5.5) If ADP concentrations in the cytoplasm increase, the biochemical equilibrium of
ADP release from the cross-bridge head is altered such that the rate of release (S1) will
be ____ and the cross-bridge cycling rate (Vmax) will be ____.
a) Increase; Decreased
b) Decreased; Increased
c) Increased; Increased
d) Decreased; Decreased

DO NOT DISTRIBUTE - 27 -
Musculoskeletal 17Mar2009

5.6) Muscle lactate has a ____ effect on force generation of the contractile proteins and
may also directly affect the calcium-ATPase of muscle.
a) Major stimulatory
b) Major inhibitory
c) Minor stimulatory
d) Minor inhibitory
5.7) Even during normal atmospheric gas concentrations and without severe pathological
conditions, the respiratory system can drastically limit physical performance.
a) True
b) False
6) Which of the following factors that contributes to maximal muscle force generation is
associated with the central nervous system?
a) Muscle biochemistry and physiology
b) Fitness, training, and genetics
c) Connective tissue
d) Proprioception
e) Biomechanics
7.1) Which of the following types of muscle contraction can lead to injuries and typically
to greater delayed onset muscle soreness (DOMS)?
a) Isometric activation
b) Isotonic contraction
c) Eccentric contraction
7.2) Which of the following involves shortening of the muscle thus producing less force
as all the motor unites do not fully activate through motion?
a) Isometric activation
b) Isotonic contraction
c) Eccentric contraction
8) Testing muscle endurance involving a given force (about 60%) being lifted one time
and held at a fixed angle is testing using:
a) Isometric activation
b) Isotonic contraction
c) Eccentric contraction
9) Progressive Resistance Exercise (PRE) created by DeLorme (1945) is the basis of all
modern strength training systems. With PRE, initial lifting repetitions are ____ and
increased until repetitions are 9-10, at which point an increased load is started with initial
repetitions.
a) 1
b) 3
c) 5
d) 6
e) 7
10) A strength training regimen involving 12-15 repetitions, for three sets, two to three
times weekly, is designed for maximal:
a) Muscle mass
b) Muscle endurance
c) Muscle strength

DO NOT DISTRIBUTE - 28 -
Musculoskeletal 17Mar2009

d) A & B
e) A & C
f) B & C
11.1) Which of the following is considered the best training method, however is the most
stressful and thus is not commonly used as it can cause tears and DOMS?
a) Free weight training
b) Eccentric training
c) Isometric training methods
d) Isokinetic training methods
11.2) Which of the following circumvents the “weakest point phenomena”, may involve
expensive machinery, and requires the individual to be very motivated to work in this
mode of training?
a) Free weight training
b) Eccentric training
c) Isometric training methods
d) Isokinetic training methods
11.3) Which of the following allows for training of synergistic and postural muscles but
can be limited due to the length-tension relationship?
a) Free weight training
b) Eccentric training
c) Isometric training methods
d) Isokinetic training methods
11.4) Which of the following would involves maximal voluntary contraction (MVC)
repeated until fatigue is reached?
a) Free weight training
b) Eccentric training
c) Isometric training methods
d) Isokinetic training methods
11.5) Free weights are preferred over weight lifting machines?
a) True
b) False
11.6) How many days should be given between strenuous strength training activities in
order to recuperate?
a) 1
b) 2
c) 3
d) 5
12) During a strength training program, an individual is strengthening their biceps. Which
other muscles should they strengthen on the same day?
a) Trapezius
b) Pectoralis major
c) Quadriceps
d) Triceps
e) Latrissumus dorsi
13.1) What type of muscle training optimizes the motor unit recruitment ability?
a) Strength training

DO NOT DISTRIBUTE - 29 -
Musculoskeletal 17Mar2009

b) Speed training
c) Endurance training
d) Mass training
13.2) Since neurological adaptations of the central nervous system (CNS) can increase
the strength of a collateral limb when training one limb, it is recommended during free
weight training to alternate dumbbell repetitions between limbs.
a) True
b) False
13.3) Muscle hypertrophy can be enhanced with increased protein intake and increased
retention of:
a) Oxygen
b) Carbon
c) Nitrogen
d) Potassium
e) Magnesium
13.4) Which of the following techniques used by body-builders leads to hypertrophy
being spread equally among muscle fiber types, not just fast-twitch muscle?
a) Increased load
b) Decreased load
c) Increased repetitions
d) Decreased repetitions
13.5) Hyperplasia, or abnormal growth, would NOT occur in what location?
a) Endometrium
b) Adrenal gland
c) Muscle
d) Breast
e) Prostate
13.6) Which of the following is NOT true regarding the effects of strength training?
a) Increased bone density and girth
b) Thicker tendons, ligaments, and cartilage
c) Increased healing rate
d) Decreased musculoskeletal injuries
e) Decreased glycolytic enzyme levels
14) Which of the following is considered the preferential technique for improving
flexibility?
a) Explosive stretching techniques
b) Bouncing stretching techniques, done over 15 seconds
c) Stretching to the point of pain and holding until the pain resides
d) Stretching to range of motion and holding for 5-10 seconds
e) Stretching to range of motion and holding for 10-20 seconds
15.1) Which of the following is decreased with strength training exercises?
a) Risk of osteoporosis
b) Risk of high blood pressure
c) Risk of diabetes
d) Risk of myocardial infarction
e) Risk of neurological disease

DO NOT DISTRIBUTE - 30 -
Musculoskeletal 17Mar2009

15.2) About what percentage of adult runners are injured (one week or more) over the
course of a year?
a) 5%
b) 25%
c) 50%
d) 75%
e) 95%
15.3) Individuals who run more than ____ miles per week are much more likely to be
injured than those who run fewer miles.
a) 5
b) 10
c) 15
d) 20
e) 26.2
15.4) An individual is diagnosed with mild hypertension and decides to begin a cardio
and weight-lifting regimen. During the first week, they develop exercise pain in their left
bicep with great soreness after workout. What should this individual do?
a) Train their right bicep more allowing for the CNS to remodel
b) Train their left tricep to allow for reciprocal support
c) Increase load on left bicep and worth through the pain
d) Stop exercising for a few days and see if the pain and soreness goes away
e) Not change, it is common to have intense pain with new workout regimens

7 – Microbiology: Bacterial Infections


1.1) Reactive arthritis is an inflammatory response of the ____ caused by ____ infection.
a) Bone; Direct
b) Bone; Indirect
c) Joint; Direct
d) Joint; Indirect
1.2) Septic arthritis affects about 20,000 people per year in the United States. Which of
the following is the most common causes?
a) Bacteria
b) Viruses
c) Fungi
d) Parasites
2) Which of the following would likely NOT be associated with reactive arthritis?
a) Klebsiella pneumoniae
b) Chlamydophila trachomatis
c) Salmonella, Campylobacter
d) Yersinia, Shigella
e) Listeria monocytogenes
3.1) A 20-year-old male patient presents with chronic pain in their lower spine. A blood
test shows the patient has a variant of the HLA-B27 gene. Cross-reactivity of antigens
from what bacteria are involved in this patient’s case?
a) Chlamydophila
b) Klebsiella

DO NOT DISTRIBUTE - 31 -
Musculoskeletal 17Mar2009

c) Salmonella
d) Campylocabter
e) Listeria
3.2) A patient presents with urethritis, arthritis, conjunctivitis, and mucocutaneous lesions
on the soles of the feet (keratoderma blennorrhagica). Which of the following is the most
likely cause?
a) Campylobacter
b) Salmonella
c) Chlamydophila
d) Yersinia
e) Shigella
4.1) A 25-year-old female medical student who is sexually active presents with suspected
septic arthritis of the left elbow and multiple skin lesions. Synovial fluid culture is
negative. Which of the following is the most likely pathogen?
a) Staph aureus
b) Strep viridans
c) Strep pneumonia
d) Neisseria gonorrhea
e) Pseudomonas aeruginosa
4.2) A 3-year-old child presents with acute joint pain in the right knee and a fever. The
patient has mild pain in their left knee and hip. Which of the following is the most likely
pathogen?
a) Staph aureus
b) Strep viridans
c) Strep pneumonia
d) Neisseria gonorrhea
e) E. coli
5.1) A patient presents with joint pain at the knee and loss of some range of motion in the
affected leg. Upon exam, the knee is hot and swollen. The patient has chills, fever, and
admits to IV drug use. Lab test is negative for MRSA. Which of the following is the most
likely cause?
a) Staph aureus
b) Strep viridans
c) Strep pneumonia
d) Neisseria gonorrhea
e) Pseudomonas aeruginosa
5.2) Which joint, other than the knee, is commonly affected by non-gonococcal arthritis
in children?
a) Hands
b) Feet
c) Wrist
d) Ankle
e) Hip
5.3) What is the most likely cause of non-gonococcal septic arthritis in adults?
a) Staph aureus
b) Strep viridans

DO NOT DISTRIBUTE - 32 -
Musculoskeletal 17Mar2009

c) Strep pneumonia
d) Staph epidermidis
e) E. coli
5.4) Which of the following would be the most likely bacteria seen in septic arthritis of a
prosthetic joint?
a) Neisseria gonorrhea
b) Staph epidermidis
c) Strep pneumonia
d) Strep viridans
e) Group B Strep
5.5) What percentage of patients with disseminated (chronic) Lyme disease develop
arthritis?
a) 20%
b) 40%
c) 60%
d) 80%
e) 100%
5.6) Which of the following is NOT true regarding the diagnosis of septic arthritis?
a) Synovial fluid is positive 75% of the time for Gram-positive cases
b) Synovial fluid is negative 75% of the time for gonococcal cases
c) Synovial fluid shows increased glucose levels with arthritis
d) Radiography studies are secondary to gram staining
e) CBC check for elevated WBCs and >80% neutrophils
6.1) Chronic osteomyelitis is characterized by necrotic bone, and can also involve
vascular insufficiency for what type of patients?
a) Infants
b) Congestive heart failure (CHF)
c) Chronic obstructive pulmonary disease (COPD)
d) Diabetic
e) Elderly
6.2) Which of the following is most commonly seen in neonates with hematogenous
osteomyelitis?
a) Fever
b) Chills
c) Sweats
d) Erythema over involved bones
e) Symptoms are usually absent
6.3) Which of the following forms of osteomyelitis is the most difficult to treat?
a) Progressive inflammatory
b) Acute hematogenous
c) Acute direct infection
d) Chronic
6.4) Which of the following is the hallmark sign of chronic osteomyelitis?
a) Pain over infected area with absence of systemic symptoms
b) High fever, bone pain, and elevated WBC count
c) Inability to ambulate and severe nausea

DO NOT DISTRIBUTE - 33 -
Musculoskeletal 17Mar2009

d) Sequestrum surrounded by involcrum


e) Multiple hairline fractures viewed radiographically
7.1) Which of the following bacteria is associated with acute osteomyelitis in sickle cell
patients (0.36% annual incidence)?
a) Campylobacter
b) Salmonella
c) Chlamydophila
d) Yersinia
e) Shigella
7.2) Acute osteomyelitis occurs in about 16% of foot puncture patients and 30-40% in
what type of patients?
a) Elderly
b) Congestive heart failure (CHF)
c) Patients with prosthetic devices
d) Diabetic
e) Infants
7.3) Which of the following bones is the most commonly affected in acute osteomyelitis?
a) Tibia
b) Femur
c) Fibula
d) Humerus
e) Ulna
7.4) Hematogenous osteomyelitis, which is usually spread from blood after injury,
surgery, or prosthesis, is most commonly found in what type of patients?
a) Children
b) Adults
c) Elderly
d) Males
e) Females
7.5) What patient group shows a high incidence of hematogenous osteomyelitis being
caused by Group B Strep?
a) Infants
b) Children
c) Adults
d) Diabetics
e) Females
7.6) Which of the following is the most common cause of hematogenous osteomyelitis in
children and adults, as well as the most common cause of direct osteomyelitis?
a) Strep pyogenes
b) H. influenza type B
c) Serretia marcescens
d) E. coli
e) Staph aureus
7.7) What patient group shows an incidence of hematogenous osteomyelitis being caused
by H. influenza type B and Strep pyogenes?
a) Infants

DO NOT DISTRIBUTE - 34 -
Musculoskeletal 17Mar2009

b) Children
c) Adults
d) Diabetics
e) Females

8 – Microbiology: Autoimmunity
1.1) Which of the following would NOT occur if self antigen is missing from the thymus
or if MHC is unable to bind antigen?
a) Fewer auto-reactive T cells would be deleted (negative selection) and more
escape to the periphery
b) DNA may leak out of dying cells and it is not cleared if complement is
defective
c) Physical cell barriers may become defective
d) If a tissue becomes infected, cells of innate immune response stop expressing
co-stimulatory molecules and prevent cytokine release
e) Infection may trigger molecular mimicry
1.2) Which of the following is NOT associated with ectopic expression of MHC?
a) Insulin-Dependent Diabetes Mellitus (IDDM)
b) Grave’s disease
c) Increased IFN-" with trauma
d) Increased IFN- " with Systemic Lupus Erythematosus (SLE)
e) Decreased IFN- " in intestinal epithelial and melanoma cells
1.3) Which of the following would NOT likely cause polyclonal B-cell activation?
a) Gram-negative bacteria
b) Gram-positive bacteria
c) Cytomegalovirus (CMV)
d) Epstein-Barr Virus (EBV)
1.4) Which of the following autoimmunity diseases is NOT associated with HLA
serotype DR3?
a) Grave disease
b) Myasthenia gravis
c) Systemic lupus erythematosus
d) Insulin-dependent diabetes mellitus
e) Hashimoto thyroiditis
1.5) Rheumatoid arthritis and perphigus vulgaris are associated with what HLA serotype?
a) B27
b) DR2
c) DR3
d) DR4
e) DR5
1.6) Goodpasture’s syndrome and multiple sclerosis are associated with what HLA
serotype?
a) B27
b) DR2
c) DR3
d) DR4

DO NOT DISTRIBUTE - 35 -
Musculoskeletal 17Mar2009

e) DR5
1.7) An individual develops a wide range of autoantibodies against endocrine glands,
liver, skin, blood cells, and platelets. Autoimmune polyglandular syndrome is suspected.
What mechanism of autoimmunity is involved?
a) Autoimmune regulation (AIRE)
b) Release of sequestered antigens
c) Molecular mimicry
d) Inappropriate expression of HLA
e) Breakdown of tolerance
2) Exposure of T-cells to normally sequestered antigens at a later time may result in their
activation, as in what situation?
a) Diabetes
b) Genetic mutation
c) Trauma
d) Sickle cell anemia
e) Myocardial infarction
3) Which of the following autoimmunity mechanisms is involved in rheumatic fever and
post-rabies encephalitis?
a) Autoimmune regulation (AIRE)
b) Release of sequestered antigens
c) Molecular mimicry
d) Inappropriate expression of HLA
e) Breakdown of tolerance
Match the disease with the human host protein mimicked in autoimmunity:
4.1) Human cytomegalovirus a) Acetylcholine receptor
4.2) Poliovirus b) HLA-B27 molecule
4.3) Papilloma virus & rabies virus c) Human IgG constant region
4.4) Klebsiella pneumonia d) Corticotropin & myelin basic protein (MBP)
4.5) Adenovirus e) Alpha-gliadin
4.6) Human immunodeficiency virus f) HLA-DR molecule
4.7) Measles virus g) Insulin receptor
5.1) A female patient presents with a type II hypersensitivity involving progressive
muscle weakness, droopy eyelids, double vision, weakened face muscles, and slightly
impaired breathing. Which of the following is the most likely?
a) Myasthenia gravis
b) Grave disease
c) Ankylosing spondylitis
d) Hashimoto thyroiditis
e) Rheumatoid arthritis
5.2) Auto-antibodies attack which of the following in myasthenia gravis?
a) Acetylcholine receptor
b) HLA-B27 molecule
c) Human IgG constant region
d) Myelin basic protein (MBP)
e) Insulin receptor

DO NOT DISTRIBUTE - 36 -
Musculoskeletal 17Mar2009

5.3) Rheumatoid arthritis is a ____ hypersensitivity and systemic lupus erythematosus is


a ____ hypersensitivity.
a) Type II; Type II
b) Type III; Type III
c) Type IV; Type IV
d) Type II; Type III
e) Type III; Type IV
5.4) A 30-year-old patient presents with chronic inflammation of the finger joints,
elevated IgG, elevated IgM, and positive serotype for HLA-DR4. Which of the following
would NOT be a useful treatment for this patient?
a) Gold therapy
b) NSAIDS (COX-2 blockers)
c) Thymus removal
d) Corticosteroids
e) Anti-TNF-! (infliximab)
5.5) A 30-year-old female patient presents with malar (butterfly) rash on their face,
multiple-site tissue inflammation, and tissue necrosis. After doing an anti-DS DNA test,
the clinician starts treatment involving immunosupression and anti-malarial drugs. Which
of the following diseases does this patient have?
a) Myasthenia gravis
b) Grave disease
c) Ankylosing spondylitis
d) Hashimoto thyroiditis
e) Systemic lupus erythematosus
5.6) Multiple sclerosis, a type IV hypersensitivity, involves a subtype of T-helper cells
(Tdth) infiltrating the CNS white matter and reacting with which of the following?
a) Acetylcholine receptor
b) HLA-B27 molecule
c) Human IgG constant region
d) Myelin basic protein (MBP)
e) Gliadin
5.7) Ankylosing spondylitis and reactive arthritis (Reiter syndrome) are associated with
which HLA serotype?
a) B27
b) DR2
c) DR3
d) DR4
e) DR5
5.8) INF-# is a treatment option for which of the following?
a) Myasthenia gravis
b) Multiple sclerosis
c) Ankylosing spondylitis
d) Hashimoto thyroiditis
e) Rheumatoid arthritis
5.9) A middle-aged male patient presents with lower back pain. A radiograph shows a
“bamboo spine” and lab results are positive for Klebsiella species. The clinician begins

DO NOT DISTRIBUTE - 37 -
Musculoskeletal 17Mar2009

treatment with an anti-TNF alpha drug (remicade, enbrel, humira). Which of the
following does this patient have?
a) Reactive arthritis
b) Myasthenia gravis
c) Systemic lupus erythematosus
d) Ankylosing spondylitis
e) Hashimoto thyroiditis
5.10) A young male patient presents with wrist arthritis, conjunctivitis, and urethritis. Lab
results are positive for Chlamydia. Which of the following is the most likely?
a) Reactive arthritis
b) Myasthenia gravis
c) Systemic lupus erythematosus
d) Ankylosing spondylitis
e) Hashimoto thyroiditis
5.11) In which of the following experimental therapeutic approaches would immune
response be directed toward the T-cell receptor (TCR) region of an autoimmune clone?
a) T-cell vaccination
b) Peptide blockade of MHC
c) Monoclonal antibody, anti-CD4
d) Monoclonal antibody, anti-TCR
e) Monoclonal antibody, anti-MHC
5.12) In the peptide blockage for Celiac disease, which of the following is deaminated to
bind to HLA-DQ2, the TCR, and tissue transglutaminase?
a) Acetylcholine receptor
b) HLA-B27 molecule
c) Human IgG constant region
d) Myelin basic protein (MBP)
e) Gliadin
5.13) Which of the following potential treatments using monoclonal antibodies would
affect the autoreactive T-cell directly?
a) Peptide blockage
b) Anti-CD4
c) Anti-IL-2R
d) Anti-MHC
e) Anti-TCR

9 – Pharmacology: Synthesis of Eicosanoids


1.1) Which of the following is associated with COX-I (PGH synthase-I) instead of COX-
II (PGH synthase-II)?
a) Upregulation by growth factors
b) Upregulation by shear stress
c) Upregulation by tumor promotors
d) Upregulation by cytokins
e) Generation of prostanoids such as for gastric epithelial cytoprotection
f) Major source of prostanoids in inflammation and cancer
Match the arachidonate products with the description:

DO NOT DISTRIBUTE - 38 -
Musculoskeletal 17Mar2009

1.2) Aggregates platelets, vasoconstriction a) Alprostadil (PGE1)


1.3) Inhibits platelet aggregation, powerful vasodilator b) Misoprostol (PGE1)
1.4) PGF2 used to treat open angle glaucoma c) PGE2 & PGF2a
1.5) Used in obstetrics to induce labor d) Latanoprost
1.6) Prevents peptic ulcer and used to terminate pregnancy e) Prostacyclin
1.7) Maintains ductus artereosus in neonates f) Thromboxane (TXA2)
2.1) Metabolism of arachidonic acid by lipoxygenases (LOX) produces
hydroperoxyeicosatetraenoic acids (HPETEs), which rapidly convert to hydroxyl
derivatives (HETEs) and leukotrienes. They are associated with all of the following
EXCEPT:
a) Asthma
b) Anaphylactic shock
c) Epilepsy
d) Cardiovascular disease
2.2) LTC4 and LTD4 are primary components of slow-reacting substance of anaphylaxis
(SRS-A), from cysteinyl leukotrienes, and are potent:
a) Vasoconstrictors
b) Vasodilators
c) Bronchoconstrictors
d) Bronchodilators
2.3) The presense of lipoxins would indicate:
a) Start of inflammation
b) Increased inflammation
c) Anaphylactic reaction
d) Resolution of inflammation
3) The cytochrome P450 monooxygenases (epoxygenases) products, 4
epoxyeicosatrienoic acid (EET) and dihydroxyeicosatrienoic acid (DHET), are associated
with calcium-activated potassium channels and have what effect?
a) Smooth muscle cell hyperpolarization and vasodilation
b) Smooth muscle cell hypopolarization and vasodilation
c) Smooth muscle cell hyperpolarization and vasoconstriction
d) Smooth muscle cell hypopolarization and vasoconstriction

10 – Pharmacology: Eicosanoids
1.1) Which of the following prostaglandins/thromboxanes is thought to be the
vasoconstrictor substance in hepatorenal syndrome, see in patient with cirrhosis?
a) TXA2
b) PGF2a
c) PG2a
d) PGE2
e) DP1
1.2) Which of the following prostaglandins/thromboxanes is expressed in asthma?
a) TXA2
b) PGF2a
c) PG2a
d) PGE2

DO NOT DISTRIBUTE - 39 -
Musculoskeletal 17Mar2009

e) DP1
1.3) In about 10% of patients taking non-steroidal anti-inflammatory drugs (NSAIDs), a
shift in arachidonate metabolism from COX-I metabolism to leukotrienes formation
causes which of the following?
a) Stomach ulcers
b) Diarrhea
c) Bronchospasm
d) Bradycardia
e) Anaphylaxis
1.4) Which of the following enhances platelet aggregation?
a) Low concentrations of PGE2
b) High concentrations of PGE2
c) PGD2
d) PGI2
e) TXA2
1.5) Urinary metabolites of which of the following increase in patient experiencing
myocardial infarctions, and is suppressed by low-dosage aspirin?
a) Low concentrations of PGE2
b) High concentrations of PGE2
c) PGD2
d) PGI2
e) TXA2
1.6) Macrophage COX2 appears to contribute roughly 10% of increment in
thromboxanes biosynthesis observed in what population, while the rest is derived from
platelets?
a) Diabetics
b) Smokers
c) Post-MI patients
d) Epileptics
e) Sickle-cell patients
1.7) Which of the following increase glomerular filtration through vasodilating effects?
a) TXA2 & PGD2
b) PGE2 & PGD2
c) PGD2 & PGI2
d) PGE2 & PGI2
e) PGI2 & TXA2
1.8) Prostaglandins increase water ____ and increase sodium ____ in the kidney.
a) Absorption; Absorption
b) Absorption; Excretion
c) Excretion; Absorption
d) Excretion; Excretion
1.9) Synthase inhibitors of which of the following should improve renal function during
inflammatory conditions?
a) TXA2
b) PGF2a
c) PG2a

DO NOT DISTRIBUTE - 40 -
Musculoskeletal 17Mar2009

d) PGE2
e) PGD2
1.10) Along with PGI2, which of the following causes uterine relaxation?
a) Low concentrations of PGE2
b) High concentrations of PGE2
c) PGF2a
d) TXA2
1.11) Which of the following is used along with oxytocin for induced parturition (birth)?
a) Low concentrations of PGE2
b) High concentrations of PGE2
c) PGF2a
d) TXA2
e) PGI2
1.12) Which of the following increases body temperature (EP3 receptors) when
administered directly into cerebral ventricles?
a) TXA2
b) PGF2a
c) PG2a
d) PGE2
e) PGD2
1.13) Which of the following induces fever?
a) PGD2 & TXA2
b) PGF2a & PGI2
c) PDG2 & PGI2
d) PGF2a & TXA2
1.14) Vasoconstriction during treatment with COX inhibitors is due to increased
norepinephrine (NE) and inhibition of which of the following?
a) PGD2 & TXA2
b) PGE2 & PGI2
c) PDG2 & PGI2
d) PGE2 & TXA2
1.15) Hyperalgesia is produced by which of the following, with the release of eicosanoids
during inflammation serving to amplify nociceptors?
a) LTA4
b) LTB4
c) LTC4
d) LTD4
e) LTE4
1.16) Which of the following occurs with prostaglandins associated with skeletal tissue?
a) Decreased bone resorption and formation
b) Increased bone resorption and formation
c) Increased bone formation
d) Increased bone resorption
2.1) Prostaglandins have what affect on inflammation?
a) Inducers of inflammation
b) Suppressors of inflammation

DO NOT DISTRIBUTE - 41 -
Musculoskeletal 17Mar2009

c) Increase current inflammation response


d) Cause anaphylaxis
2.2) Which of the following is true regarding 12-(R)-HETE?
a) Chemo-attractant and potent stimulator of Na/K-ATPase
b) Chemo-attractant and potent inhibitor of Na/K-ATPase
c) Not a chemo-attractant and potent stimulator of Na/K-ATPase
d) Not a chemo-attractant and potent inhibitor of Na/K-ATPase
2.3) What affect do lipoxin A and B have on the heart in vitro?
a) Reduce myocardial contractility
b) Reduce coronary blood flow
c) Coronary vasoconstrictors
d) Coronary vasodilators
e) A & B (cardiac depression)
2.4) What affect do LTC4a and LTD4 have on the heart?
a) Reduce myocardial contractility
b) Reduce coronary blood flow
c) Coronary vasoconstrictors
d) Coronary vasodilators
e) A & B (cardiac depression)
2.5) Incidental use of NSAIDS is associated with a 40-50%:
a) Reduction in lung tumor formation
b) Increase in lung tumor formation
c) Reduction in colon tumor formation
d) Increase in colon tumor formation
e) Reduction in testicular tumor formation

11 – Pharmacology: Inhibition of Eicosanoid Synthesis


1.1) Corticosteroids block which of the following pathways by stimulating the synthesis
of annexins (lipocortins)?
a) Prostaglandins
b) Prostacyclins
c) Thromboxanes
d) Leukotrienes
e) All of the above
1.2) Non-steroidal anti-inflammatory drugs (NSAIDs), such as indomethacin and
ibuprofen, block which of the following pathways
a) Prostaglandins
b) Thromboxanes
c) Leukotrienes
d) A & B
e) B & C
f) A & C

12 – Pharmacology: Clinical Eicosanoids


1.1) Which of the following is an antiprogestin that is combined with misoprostol to
produce early abortion and has an increased risk of sepsis with vaginal administration?

DO NOT DISTRIBUTE - 42 -
Musculoskeletal 17Mar2009

a) PGE2 & PGF2a


b) Dinoprostone
c) Mifepristone & RU-486
d) Carboprost tromethamine
1.2) Which of the following has oxytocic actions and is used to terminate pregnancy at
any stage by promoting uterine contractions?
a) PGE2 & PGF2a
b) Dinoprostone
c) Mifepristone & RU-486
d) Carboprost tromethamine
1.3) Which of the following is administered vaginally to induce 2nd trimester abortions,
benign hydatiform mole, ripening of the cervix for induction of labor, and has a first pass
metabolism in the lungs of about 95%?
a) PGE2 & PGF2a
b) Dinoprostone
c) Mifepristone & RU-486
d) Carboprost tromethamine
1.4) which of the following is administered intramuscular (IM) and has a success rate of
about 80% at terminating 2nd trimester pregnancies?
a) PGE2 & PGF2a
b) Dinoprostone
c) Mifepristone & RU-486
d) Carboprost tromethamine
1.5) Which of the following is used as a cervical softening (ripening) agent to induce
labor and shorten the time to onset of labor and delivery?
a) PGE2 & PGF2a
b) Dinoprostone
c) Mifepristone & RU-486
d) Carboprost tromethamine
1.6) PGF2a, a facilitator of labor, should be used in caution with mothers that have:
a) Cardiac devices
b) Epilepsy
c) Alcohol intolerance
d) Hyperbilirubinemia
e) Asthma
1.7) Primary dysmenorrhea can be relieved in 75-85% of cases with ____. Another
option is ____, which is effective but due to acetylation of platelet COX and thus
inhibition of platelet TXA2 synthesis, it may increase menstral bleeding.
a) NSAIDS; Aspirin
b) NSAIDS; Heparin
c) Aspirin; NSAIDS
d) Aspirin; Heparin
2) Alprostadil (PGE1) is used to treat which of the following?
a) Uterine tumors
b) Premature ejaculation
c) Erectile dysfunction

DO NOT DISTRIBUTE - 43 -
Musculoskeletal 17Mar2009

d) Urinary incontinence
e) Chlamydia
3.1) Which of the following would be used to lower peripheral, pulmonary, and coronary
resistance and has specific use for portopulmonary hypertension?
a) Prostaglandin
b) Prostacyclin
c) Thromboxane
d) Leukotriene
e) All of the above
3.2) Which of the following would be used to inhibit synthesis of PGE2 and thus close
the ductus arteriosus in a newborn?
a) NSAIDs
b) Leukotriene inhibitors
c) Thromboxane inhibitors
d) COX inhibitors
e) Smooth muscle relaxants
4) ____ promotes platelet aggregation and ____ inhibits platelet aggregation.
a) TXA2; PGI2
b) PGD2; PGI2
c) TXA2; PGE2
d) PGE2; PGE2
e) PGI2; TXA2
5) all of the following inhibit leukotrienes-receptors, thus are effective in asthma. Which
one also inhibits lipoxygenase?
a) Zafirlukast
b) Montolukast
c) Zileuton
6) Misoprostol can be used for prevention of which of the following?
a) PG-induced asthma
b) Coronary artery dilation
c) Peripheral artery constriction
d) Bacterial-induced peptic ulcers
e) NSAID-induced peptic ulcers
7) A patient is on a topical prostaglandin analog to reduce ocular hypertension and
prevent glaucoma. Exam reveals a darkening of the iris to brown (heterochromia). Which
of the following is the patient most likely on as it is stable, long-acting, and the most
commonly prescribed?
a) Latanoprost
b) Travoprost
c) Unoprostone
d) Bimatoprost

13 – Pharmacology: Non-Steroidal Anti-Inflammatory Drugs (NSAIDs)


1.1) Which of the following is used to treat inflammatory arthritis but have limited long
term use due to toxicity?
a) NSAIDs

DO NOT DISTRIBUTE - 44 -
Musculoskeletal 17Mar2009

b) Glucocorticoids
c) SAARDs (DMARDs)
1.2) Which of the following is used for slow bone damage in rheumatoid arthritis?
a) NSAIDs
b) Glucocorticoids
c) SAARDs (DMARDs)
2) Which of the following is NOT true regarding aspirin?
a) Anti-inflammtory
b) Anti-pyretic
c) Analgesic
d) COX-2 inhibitor
e) Aggregates platelets
3.1) Increasing salicyclate dosage (e.g. Aspirin) will increase salicyclate concentration
disproportionately, thus increasing the elimination half-life.
a) True
b) False
3.2) Aspirin causes COX inhibition in the CNS as well as inhibition of interleukin-1,
which has what effect?
a) Anti-inflammation
b) Analgesic
c) Anti-pyretic
d) Anti-platelet
3.3) Aspirin, in low-dosage, is used for preventing all of the following EXCEPT:
a) Colon cancer
b) Rheumatic fever
c) Unstable angina
d) Coronary artery thrombosis
e) Transient ischemic attacks
4.1) Which of the following is inhibited by COX-2, not COX-1?
a) GI tract function
b) Renal tract function
c) Platelet function
d) Macrophage differentiation
e) Inflammation
4.2) Celecoxib (Celebrex) is a COX-2 inhibitor used to treat osteoarthritis, rheumatoid
arthritis, acute pain, and menstrual pain. What “black-box” warning is listed for
celecoxib?
a) Gastrointestinal and renal
b) Gastrointestinal and cardiac
c) Renal and hepatic
d) Hepatic and cardiac
e) Cardiac and neurologic
5.1) Celecoxib is a sulfonamide and thus may cause:
a) Renal insufficiency
b) Increased menstrual pain
c) Acute joint pain

DO NOT DISTRIBUTE - 45 -
Musculoskeletal 17Mar2009

d) Skin rashes
e) Diaphoresis
5.2) Which of the following is a selective COX-2 inhibitor and would be associated with
fewer clinical GI symptoms and complications when treating rheumatoid disease?
a) Piroxicam (Feldene)
b) Diclofenac (Voltaren)
c) Meloxicam (Mobic)
d) Naproxen (Aleve)
5.3) Ibuprofen and indomethacin can both be used to close a patent ductus arteriosus in
preterm infants. Comparing the two drugs, ibuprofen will ____ urine output and cause
____ fluid retention.
a) Decrease; Less
b) Decrease; More
c) Increase; Less
d) Increase; More
5.4) Oral ibuprofen prescribed in lower doses (< 2,400mg/day) has:
a) Both analgesic and anti-inflammatory efficacy
b) Analgesic but no anti-inflammatory efficacy
c) Anti-inflammatory but no analgesic efficacy
d) Neither analgesic nor anti-inflammatory efficacy
5.5) Indomethacin is a non-selective COX inhibitor that can be used for all of the
following EXCEPT:
a) Sweet syndrome
b) Juvenile rheumatoid arthritis
c) Pain relief
d) Cerebral palsy
e) Conjunctival inflammation
5.6) Which of the following is the least common of the listed side-effects for
indomethacin?
a) Abdominal pain
b) Diarrhea
c) Pancreatitis
d) Headache
e) Psycosis
5.7) Ketoprofen inhibits:
a) COX
b) Lipoxygenase
c) COX and lipoxygenase
d) None of the above
5.8) Which of the following can be used instead of morphine for mild to moderate post-
surgical pain and when used with an opioid it decreases the opioid requirement?
a) Ketoprofen
b) Indomethacin
c) Ibuprofen
d) Meloxicam
e) Ketorlac

DO NOT DISTRIBUTE - 46 -
Musculoskeletal 17Mar2009

5.9) Which of the following adverse effects occurs twice as frequently with over the
counter (OTC) naproxen versus ibuprofen?
a) Myocardial infarction
b) Vomitting
c) GI bleeding
d) Syncope
e) Atrial fibrillation
5.10) Which of the following is used for rheumatic disease, may inhibit the development
of certain cancers, and is associated with some severe side effects like Stevens-Johnson
syndrome and thrombocytopenia?
a) Diclofenac
b) Etodolac
c) Nabumetone
d) Sulindac
e) Oxaproxin
f) Piroxicam
6.1) Adverse effects of acetaminophen (Tylenol) are associated with the:
a) GI tract
b) Spleen
c) Liver
d) Kidneys
e) Heart
6.2) Adverse effects of acetylsalicylic acid (ASA, Aspirin) are associated with the:
a) GI tract
b) Spleen
c) Liver
d) Kidneys
e) Heart

14 – Pharmacology: Disease-Modifying Antirheumatic Drugs (DMARDs)


1) NSAIDs can provide relief for rheumatoid arthritis as well as altering the progression
of the disease, such as arresting slow joint destruction.
a) True
b) False
2) Which of the following is true regarding disease modifying antirheumatic drugs
(DMARDs)?
a) Short-term drugs used to provide relief from pain and inflammation
b) Short-term drugs used to stop joint destruction
c) Long-term drugs used to provide relief from pain and inflammation
d) Long-term drugs used to stop joint destruction
3) DMARDs are less toxic when compared with NSAIDs.
a) True
b) False
Match the DMARD with the mechanism of action:
4.1) Decreases IgA and IgM rheumatoid factor a) Methotrexate
4.2) Toxic metabolite of an antibiotic b) Chlorambucil

DO NOT DISTRIBUTE - 47 -
Musculoskeletal 17Mar2009

4.3) Alters morphology and function of macrophages c) Cyclophosphamide


4.4) Unclear mechanism, used for malaria d) Cyclosporine
4.5) Inhibits cytosine monophosphate dehydrogenase e) Azathioprine
4.6) Suppresses inosinic acid synthesis via 6-thioguanine f) Mycophenolate mofetil
4.7) Inhibits IL-1, IL-2 receptor production g) Chloroquine (hydro)
4.8) Active metabolite in phosphoramide mustard h) Gold
4.9) Cross links DNA preventing cell replication i) Penicillamine
4.10) Inhibits aminoimidazolecarboxamine ribunucleotide j) Sulfasalazine
5.1) Methotrexate is contraindicated in what situation?
a) Wegener granulomatosis
b) Giant cell arteritis
c) Systemic lupus erythmatosus
d) Diabetics
e) Pregnancy
5.2) Which of the following drugs decreases the rate of appearance of new erosions is
associated with hypersensitivity lung adverse reactions with acute shortness of breath?
a) Chlorambucil
b) Cyclosporine
c) Methotrexate
d) Gold
e) Azathioprine
5.3) Which of the following drugs is used for Behçet disease and is associated with bone
marrow suppression and an increased risk of leukemia?
a) Chlorambucil
b) Cyclosporine
c) Methotrexate
d) Gold
e) Azathioprine
5.4) Which of the following is NOT an adverse effect of cyclophosphamide, which is
given orally to treat rheumatic disease such as SLE?
a) Bone marrow suppression
b) Alopecia
c) Hemorrhagic cystitis
d) Pneumonia
e) Bladder carcinoma
5.5) Grapefruit juice increases the bioavilability of which of the following?
a) Chlorambucil
b) Cyclosporine
c) Methotrexate
d) Gold
e) Azathioprine
5.6) Which of the following must be closely monitored when giving cyclosporine?
a) D-dimer, Fibrin degradation products (FDP)
b) Blood urea nitrogen (BUN), Creatinine
c) Aspartate aminotransferase (AST)
d) Lactate dehydrogenase (LDH)

DO NOT DISTRIBUTE - 48 -
Musculoskeletal 17Mar2009

e) Bilirubin
5.7) Which of the following is used for treatment of renal disease due to systemic lupus
and vasculitis, and decreases the incidence of fungal infections?
a) Chloroquine
b) Cyclosporine
c) Methotrexate
d) Mycophenolate mofetil
e) Azathioprine
5.8) Which of the following is an anti-malarial also used for Sjögren syndrome?
a) Chloroquine
b) Cyclosporine
c) Methotrexate
d) Mycophenolate mofetil
e) Sulfasalazine
5.9) Which of the following is rarely used for rheumatoid arthritis, slows the radiologic
progression of rheumatoid disease, and is used to treat asthma in Japan?
a) Mycophenolate mofetil
b) Cyclosporine
c) Methotrexate
d) Gold
e) Azathioprine
5.10) Penicillamine is rarely used because of toxicity.
a) True
b) False
5.11) Which of the following is associated with reversible infertility in men, has about
15% absorption when given orally, and is used in rheumatoid arthritis, juvenile chronic
arthritis, and ankylosing spondylitis?
a) Chloroquine
b) Cyclosporine
c) Penicillamine
d) Mycophenolate mofetil
e) Sulfasalazine
5.12) A young patient presents with symptoms suggesting lupus. They have been taking a
drug for over a year for their acne, but cannot recall the name. The patient appears to
have a blue pigmentation to their skin. Which of the following is the most likely drug?
a) Chloroquine
b) Cyclosporine
c) Minocycline
d) Mycophenolate mofetil
e) Sulfasalazine
6) Inhibiting TNF-alpha and reduce inflammation seen in rheumatoid arthritis. Which of
the following would be a side-effect of TNF-alpha inhibitors?
a) Thrombocytopenia
b) Leukocytopenia
c) Hematuria
d) Granulocytopenia

DO NOT DISTRIBUTE - 49 -
Musculoskeletal 17Mar2009

e) Anemia
7) Which of the following is a recombinant fusion protein, not an IgG1 monoclonal
antibody?
a) Adalimumab
b) Infliximab
c) Etanercept
8.1) Which of the following should be given if a patient with rheumatoid arthritis is not
responding adequately to DMARDs or TNF antagonists?
a) Adalimumab
b) Infliximab
c) Etanercept
d) Abatacept
e) Leflunomide
8.2) Which of the following causes diarrhea in one of four patients and is used in
combination with methotrexate?
a) Adalimumab
b) Infliximab
c) Etanercept
d) Abatacept
e) Leflunomide
9) If a patient is on methotrexate for rheumatoid arthritis, which of the following would
NOT be beneficial if added as a therapy?
a) Cyclosporine
b) chloroquine
c) Sulfasalazine
d) Leflunomide
e) Infliximab

15 – Pharmacology: Drugs Used in Gout


1) In an acute gout attack, which of the following is phagocytosed by synoviocytes,
releasing prostaglandins, lysosomal enzymes, and IL-1 to further amplify the
inflammatory response?
a) Iron-containing molecules
b) Urate crystals
c) Bacteria
d) Lysed blood cells
e) Heavy metals
2) The major goals of gout therapy are to relieve acute gouty attacks and prevent
recurrent gouty episodes involving urate lithiasis.
a) True
b) False, treatment is aimed only at acute gouty attacks
Match the anti-gout drug with the description:
3.1) Probenecid, used to treat trophaceous gout a) Colchicine
3.2) Nonpurine inhibitor of xanthine oxidase b) NSAIDs
3.3) Anti-inflammatory, binds to tubulin, inhibits LTB4 c) Uricosuric agents
3.4) Inhibit prostaglandin synthase, increase uric acid urine excretion d) Allopurinol

DO NOT DISTRIBUTE - 50 -
Musculoskeletal 17Mar2009

3.5) Xanthine oxidase inhibitor to reduce total uric acid burden e) Febuxostat
4.1) Which of the following shows a “paradoxical effect” when treating gout and requires
the patient maintaining high urine volume and urine pH above 6.0?
a) Colchicine
b) NSAIDs
c) Uricosuric agents
d) Allopurinol
e) Febuxostat
4.2) Which of the following should NOT be used when treating gout because it causes
retention of uric acid by inhibiting secretory transport?
a) Colchicine
b) Aspirin
c) Acetamoniphen
d) Ibuprofen
e) Febuxostat
5.1) Which of the following anti-gout drugs causes diarrhea when given in oral form?
a) Colchicine
b) NSAIDs
c) Uricosuric agents
d) Allopurinol
e) Febuxostat
5.2) Which of the following is associated with burning throat pain, bloody diarrhea, and
shock with acute intoxication after overdose?
a) Allopurinol
b) NSAIDs
c) Febuxostat
d) Colchicine
e) Uricosuric agents
5.3) Which of the following should NOT be used in patients who excrete large amounts
of uric acid?
a) Colchicine
b) NSAIDs
c) Uricosuric agents
d) Allopurinol
e) Febuxostat
5.4) Which of the following is the preferred treatment for gout?
a) Allopurinol
b) NSAIDs
c) Febuxostat
d) Colchicine
e) Uricosuric agents
5.5) When using allopurinol to treat gout, which of the following drugs should be used
until the steady state serum uric acid level has normalized or decreased?
a) Uricosuric agents
b) NSAIDs
c) Febuxostat

DO NOT DISTRIBUTE - 51 -
Musculoskeletal 17Mar2009

d) Colchicine
5.6) Which of the following is well tolerated in patient with a history of allopurinol
intolerance?
a) Uricosuric agents
b) NSAIDs
c) Febuxostat
d) Colchicine

16 – Pharmacology: Skeletal Muscle Relaxants


1.1) What is the mechanism for d-tubocurarine, or “curare” used in poison darts?
a) Non-depolarizing (antagonist) neuromuscular blocking (ACh)
b) Depolarizing (agonist) neuromuscular blocking (ACh)
c) Antagonist neuromuscular breakdown blocking (AChE)
d) Agonist neuromuscular breakdown blocking (AChE)
1.2) What is the mechanism for succinylcholine
a) Non-depolarizing (antagonist) neuromuscular blocking (ACh)
b) Depolarizing (agonist) neuromuscular blocking (ACh)
c) Antagonist neuromuscular breakdown blocking (AChE)
d) Agonist neuromuscular breakdown blocking (AChE)
2.1) Which of the following drugs has a duration less than 35 minutes?
a) Tubocurarine
b) Pancuronium
c) Mivacurium
d) Pipecuronium
e) Cisatracurium
2.2) Which of the following drugs has a duration of less than 8 minutes?
a) Vecuronium
b) Atracurium
c) Rocuronium
d) Tubocurarine
e) Succinylcholine
3) In contract to longer-acting non-depolarizing blocking agents (e.g. Tubocurarine),
depolarizing agents such as succinylcholine work quickly due to their:
a) Ring structure
b) Protection of “double-ACh”
c) Absence of “double-ACh”
d) Linear structure
e) Stereoisomerism
4.1) What is the mechanism of action for rocuronium?
a) Compete with ACh at nicotinic receptor site
b) Compete with AChE at neuromuscular junction
c) Depolarization of nicotinic receptor
d) Desensitization of neuromuscular end plate
e) C & D
4.2) What is the mechanism of action for succinylcholine?
a) Compete with ACh at nicotinic receptor site

DO NOT DISTRIBUTE - 52 -
Musculoskeletal 17Mar2009

b) Compete with AChE at neuromuscular junction


c) Depolarization of nicotinic receptor
d) Desensitization of neuromuscular end plate
e) C & D
5) Neostigmine (cholinesterase inhibitor) will have an ____ effect when used with
rocuronium and an ____ effect when used with succinylcholine (phase II).
a) Additive; Antagonistic
b) Antagonistic; Additive
c) Additive; Additive
d) Antagonistic; Antagonistic
6) When giving a non-depolarizing blocker, which of the following is the last muscle to
become paralyzed?
a) Face
b) Hand
c) Abdomen
d) Thigh
e) Diaphragm
7.1) which of the following can produce hypotension, systemic histamine release, and
ganglionic blockage with larger dosages?
a) Tubocurarine
b) Vecuronium
c) Pancuronium
d) Tubocurarine
e) Succinylcholine
7.2) Which of the following can cause a moderate increase in heart rate (tachycardia) and
a small increase in cardiac output, with little or no change in systemic vascular
resistance?
a) Tubocurarine
b) Vecuronium
c) Pancuronium
d) Tubocurarine
e) Succinylcholine
7.3) Which of the following can cause stimulation of the sinus node of the heart, leading
to cardiac arrhythmias, when administered during halothane anesthesia?
a) Tubocurarine
b) Vecuronium
c) Pancuronium
d) Tubocurarine
e) Succinylcholine
7.4) Which of the following is the most likely if a second dose of succinylcholine is given
five minutes after the initial dose?
a) Tachycardia
b) Bradicardia
c) Hypertension
d) Hypotension
e) Shock

DO NOT DISTRIBUTE - 53 -
Musculoskeletal 17Mar2009

8) If succinylcholine is given to a patient with burns, nerve damage, neuromuscular


disease, closed head injury, or other trauma, which of the following dangerous situations
can arise?
a) Hypernatremia
b) Hyponatriemia
c) Hyperkalemia
d) Hypokalemia
e) Hypercalcemia
9.1) A patient is given succinylcholine alone with a volatile anesthetic and suffers an
adverse reaction of malignant hyperthermia. What drug should be administered?
a) Calcium carbonate
b) Epinepherine
c) Rocuronium
d) Dantrolene
e) Cyclobenzaprine
9.2) When given with muscle relaxants, antibiotics enhance the neuromuscular blockade.
a) True
b) False, they prevent the muscle relaxants from working
9.3) When given with muscle relaxants such as before surgery, which of the following
would succinylcholine cause?
a) Fasciculations
b) Analgesia
c) Cramping
d) Hiccups
e) Uncontrollable salivation
10.1) In patients with spasticity (e.g. cerebral palsy, multiple sclerosis, stroke), reducing
the activity to Ia fiber sthat excite the primary motor neuron would have what effect?
a) Increased hyperactive reflex arc
b) Muscle tetanus
c) Decreased hyperactive reflex arc
d) Muscle flaccidity
e) Muscle atrophy
11) Which of the following drugs would be used to reverse a non-depolarizing
neuromuscular blockage?
a) Succinylcholine
b) Pyridostigmine
c) Dantrolene
d) Rocuronium
e) Mivacurium
12.1) Which of the following is a gamma-aminobutyric acid (GABA) mimetic agent that
can reduce pain in patients with spasticity by reducing excitatory transmitters in the brain
and spinal cord?
a) Baclofen
b) Diazepam
c) Dantrolene
d) Tizanidine

DO NOT DISTRIBUTE - 54 -
Musculoskeletal 17Mar2009

e) Botulinum toxin
12.2) Which of the following is a benzodiazepine acting on GABA synapses to reduce
spasticity and possibly aid patients with cord transection?
a) Tizanidine
b) Baclofen
c) Botulinum toxin
d) Diazepam
e) Dantrolene
12.3) Which of the following reduces skeletal muscle strength by interfering with
excitation-contraction coupling in muscle fibers
a) Baclofen
b) Diazepam
c) Dantrolene
d) Tizanidine
e) Botulinum toxin
12.4) Which of the following has a poorly understood mechanism for reducing spasticity
and is an agonist similar to clonidine and imidazoline?
a) Tizanidine
b) Baclofen
c) Botulinum toxin
d) Diazepam
e) Dantrolene
12.5) Which of the following is used for ophthalmic purposes and local muscle spasms,
such as in patients with spastic disorders (e.g. cerebral palsy)?
a) Baclofen
b) Diazepam
c) Dantrolene
d) Tizanidine
e) Botulinum toxin
13) Which of the following is a ryanodine (RyRs) receptor antagonist?
a) Baclofen
b) Diazepam
c) Dantrolene
d) Tizanidine
e) Botulinum toxin
14.1) Which of the following can help with acute, local spasms, which act at the level of
the brainstem?
a) Calcium carbonate
b) Epinepherine
c) Rocuronium
d) Dantrolene
e) Cyclobenzaprine
14.2) Strong anti-muscarinic drugs may cause significant sedation, confusion and
transient visual hallucinations. They may be ineffective in treating muscle spasm due to
cerebral palsy or spinal cord injury.
a) True

DO NOT DISTRIBUTE - 55 -
Musculoskeletal 17Mar2009

b) False, they are very effective

17 – Pathology: Bones, Joints, & Soft Tissue Tumors


1.1) On the short arm of chromosome 4, there is a gene that codes for the fibroblast
growth factor (FGF) receptor 3. What occurs with activation of FGF 3?
a) Bone formation is stimulated
b) Bone formation is inhibited
c) Cartilage formation is stimulated
d) Cartilage formation is inhibited
e) Muscle formation is stimulated
1.2) Achondroplasia is the most common disease of the growth plate and a major cause of
dwarfism (heterozygous). The result is a reduction in the proliferation of chondrocytes in
the growth plate due to:
a) Immunologic attack of the chondrocytes
b) Metaplasia of chondrocytes to osteocytes
c) Paracrine cell signaling defects
d) Direct damage of the chondrocytes
e) Inactivation of FGF 3
1.3) Which of the following is NOT a phenotype of autosomal dominant achondroplasia?
a) Shortened proximal extremities
b) Trunk of relatively normal length
c) Depressed nasal root
d) Enlarged head
e) Sunken forehead
2.1) Osteogenesis imperfects, or “brittle bone disease,” affects structures rich in type I
collagen. Which of the following would be the least affected?
a) Joints and bone
b) Ears
c) Skin
d) Kidney glomeruli
e) Teeth and eyes
Match the description of osteogenesis imperfecta with the type:
2.2) Prenatal death, blue sclera, multiple bone fractures a) Type I
2.3) Postnatal fractures, normal sclera, short stature b) Type II
2.4) Blue sclera, skeletal fragility, dentinogenesis, hearing impairment c) Type III
2.5) Progressively deforming, kyphoscoliosis, blue sclera with white d) Type IV
3) Osteopetrosis (“marble bone disease”, Albers-Schonberg disease) is a diffuse skeletal
sclerosis where bones are very fragile and crumble like chalk. Which of the following is
NOT an associated symptom?
a) Cranial nerve disorders
b) Repeated infections
c) Hydrocephaly
d) Anemia
e) Excess carbonic anhydrase
4.1) Which of the following would be primary, not secondary, osteoporosis?
a) Endocrine disorders

DO NOT DISTRIBUTE - 56 -
Musculoskeletal 17Mar2009

b) Rhematological disease drug effects


c) Postmenopausal
d) Neoplasia
e) Gastrointestinal
4.2) Which of the following is NOT a suggested cause for osteoporosis?
a) Age-related change in bone cell and bone matrix
b) Reduced physical activity reduces bone remodeling stimuli
c) Genetic factors such as vitamin D receptor allele
d) Increase in Ca++ absorption
e) Hormonal influences
5.1) A middle-aged patient presents with skeletal, neuromuscular, and cardiovascular
problems. Radiography shows bones that appear large and structurally unsound.
Histological testing shows markedly thickened calvarium and a “collage of matrix
madness.” What is the likely viral cause?
a) Adenoviridae
b) Paramyoviridae
c) Herpesviridea
d) Orthomyxoviridae
e) Togaviridae
5.2) What is the malignant neoplasm of increased risk in Paget disease (osteitis
deformans)?
a) Giant cell tumor
b) Giant cell reparative granuloma
c) Extraosseous masses hematopoiesis
d) Hodkin lymphoma
e) Osteosarcoma
6) Rickets (in children) and osteomalacia (in adults) can be caused by disturbed
metabolism or deficiency in what vitamin?
a) Vitamin A
b) Vitamin B3
c) Vitamin B6
d) Vitamin B12
e) Vitamin D
7.1) A patient presents with osteitis fibrosa cystic, radiographic abnormalities on the
radial aspect of middle phalanges, increased bone activity, peritrabecular fibrosis, and
cystic brown tumors. The patient is believed to have a secondary form of
hyperparathyroidism because of:
a) Autonomous hyperplasia
b) Tumor (adenoma) of the parathyroid gland
c) Prolonged hypocalcemia
d) Increased muscle tone
e) Decreased ocular reflexes
7.2) What symptoms are used to differentiate the primary form of hyperparathyroidism?
a) Autonomous hyperplasia
b) Tumor (adenoma) of the parathyroid gland
c) Prolonged hypocalcemia

DO NOT DISTRIBUTE - 57 -
Musculoskeletal 17Mar2009

d) A & B
e) B & C
8.1) Which of the following would NOT be true of lab results from a patient with renal
osteodystrophy?
a) Increased calcium
b) Increased phosphate
c) Increased calcitriol (Vitamin D)
d) Increased parathyroid hormone (PTH)
8.2) What type of bone growth is seen in renal osteodystrophy?
a) High bone turnover
b) Normal bone turnover
c) Low bone turnover
d) A & B
e) B & C
f) A & C
9.1) Which of the following is NOT associated with avascular necrosis (osteonecrosis)?
a) SCUBA diving injury (dysbarism, Caisson disease)
b) Venous hypotension
c) Corticosteroids
d) Bone fracture
e) Alcoholism
f) Gaucher disease
9.2) Which of the following is the most common site of avascular necrosis?
a) Long bone epiphysis
b) Short bone epiphysis
c) Hip
d) Long bone diaphysis
e) Short bone diaphysis
10.1) Which of the following pyogenic (pus forming) bacteria is the most commonly
isolated in frank osteomyelitis and subacute osteomyelitis (Brodie abscess)?
a) Haemophilus influenzae
b) Enterobacter species
c) Group A Stretptococcal species
d) Staphylococcus aureus
e) Escherichia coli
10.2) What bone(s) are affected in tubercular osteomyelitis (Pott disease)?
a) Forearms
b) Femur
c) Hip
d) Spine
e) Skull
11.1) Osteoid osteoma, which occurs more frequently in the appendicular skeleton, is
associated with painful lesions caused by excess prostaglandin E2 produced by
proliferating osteoblasts. Which of the following would best relieve the pain?
a) Acetaminophen
b) Ibuprofen

DO NOT DISTRIBUTE - 58 -
Musculoskeletal 17Mar2009

c) Aspirin
d) Naproxen sodium
e) Ketoprofen
11.2) Osteoblastoma, which does not respond to salicylates, yields dull, achy pain in what
anatomical location?
a) Femurs
b) Spine
c) Hip
d) Ribs
e) Feet
11.3) Which of the following shows a “blown-out” appearance, stroma surrounding
tumor bone with dilated capillaries, and randomly interconnected trabeculae of woven
bone prominently rimmed by osteoblasts?
a) Chondrosarcoma
b) Osteoid osteoma
c) Osteoblastoma
d) Aneurysmal bone cyst
e) Giant cell tumor
12) Which of the following is NOT true of osteosarcoma?
a) People under the age of 20 are at increased risk
b) Elderly are at an increased risk
c) Metaphysis of long bones is a common location
d) The knee is a common location
e) Normal tumor giant cells are seen with hypochromatic nuclei
13.1) Which of the following, also known as a bone spur, is the most frequently observed
neoplasm of the skeleton?
a) Osteochondroma
b) Osteoid osteoma
c) Osteoblastoma
d) Aneurysmal bone cyst
e) Giant cell tumor
13.2) Osteochondroma occurs most frequently in which of the following anatomical
locations?
a) Spine
b) Elbow and wrist
c) Shoulder and knee
d) Hands and feet
e) Ribcage
14.1) Chondromas are benign tumors of:
a) Fibrocartilage
b) Hyaline cartilage
c) Elastic cartilage
d) Bone
e) Blood
14.2) Ollier disease is characterized by:
a) Multiple enchondromas throughout the skeleton

DO NOT DISTRIBUTE - 59 -
Musculoskeletal 17Mar2009

b) Multiple enchondromas in a localized area


c) Enchondromatosis associated with soft tissue hemangiomas
d) A single enchondroma in the axial skeleton
e) A single enchondroma in the appendicular skeleton
14.3) A 30-year-old woman presented with painful hemangiomas and enchondromas in
her arms, predominantly in the left hand. The hemangiomas had existed since childhood.
They were slowly increasing in size, especially the girth of the left thumb. Multiple
resections of enchondromas and transarterial carpal embolizations of the hemangiomas
with polyvinyl alcohol particles had been performed during the previous 9 years. At the
age of 22, the small finger of the left hand was removed. At the age of 25, a pathological
fracture of the left humerus occurred due to an enchondroma. Which of the following is
the most likely?
a) Ollier disease
b) Tucot syndrome
c) Fugate disease
d) McCune-Albright syndrome
e) Maffucci syndrome
15) Which of the following is a rare neoplasm showing a chicken-wire pattern of
mineralization and a striking predilection for epiphyses and apopyses (epiphyseal
equivalents, such as the iliac crest)?
a) Osteoid osteoma
b) Osteoblastoma
c) Chondrosarcoma
d) Chondroblastoma
e) Chondromyxoid fibroma
16) Which of the following presents as nodules of hyaline and tissue delineated by
fibrous septae, most frequently arising in the lower limb near the knee joint?
a) Chondrosarcoma
b) Osteoblastoma
c) Chondrosarcoma
d) Chondroblastoma
e) Chondromyxoid fibroma
17.1) Which of the following affects the center portion of the skeleton (ribs, pelvis,
shoulders) and can involve hypercellularity?
a) Chondrosarcoma
b) Osteoblastoma
c) Chondrosarcoma
d) Chondroblastoma
e) Chondromyxoid fibroma
f) Fibrous dysplasia
Match the subtype of chondrosarcoma with the description?
17.2) Low-grade chondrosarcomas with second high-grade a) Conventional
17.3) Islands of well-differentiated hyaline cartilage b) Clear cell
17.4) Large bulky tumors made of gray-white glistening tissue c) Dedifferentiated
17.5) Long bone epiphyses, large malignant chondrocytes d) Mesenchymal

DO NOT DISTRIBUTE - 60 -
Musculoskeletal 17Mar2009

18) Fibrous cortical defects and non-ossifying fibromas are seen in children older than 2
and typically arise in which of the following anatomical locations?
a) Distal humerus
b) Proximal radius
c) Costochondral rib junction
d) Distal femur
e) Sacrum
19.1) Which of the following is the most common location for monostotic fibrous
dysplasia, characterized histologially as trabeculae in shapes mimicking Chinese
characters?
a) Ribs
b) Tibia
c) Jawbone
d) Calvaria
e) Humerus
19.2) What disease is characterized by polystotic fibrous dysplasia associated with café
au lair skin pigmentation and endocrinopathies?
a) Ollier disease
b) Tucot syndrome
c) Fugate disease
d) McCune-Albright syndrome
e) Maffucci syndrome
20) Fibrosarcoma affects the metaphyses of long bones and the pelvic floor in the middle-
aged and elderly. What differentiates fibrosarcoma from malignant fibrous histiocytoma
(MFH) histopathologically?
a) Fibrosarcoma has a storiform pattern and MFH has a herringbone pattern
b) Fibrosarcoma has a herringbone pattern and MFH has a storiform pattern
c) Fibrosarcoma has a storiform pattern and MFH has a palsading pattern
d) Fibrosarcoma has a palsading pattern and MFH has a storiform pattern
e) Fibrosarcoma has a herringbone pattern and MFH has a palsading pattern
21) A male patient presents with hip pain. Radiograph shows a permeative lytic lesion
with periosteal reaction, showing a lamellated “onion skin” appearance. A diagnosis of
primitive neuroectodermal tumor (Ewing Sarcoma) is made. Which of the following is
the most likely age of the patient?
a) Less than 2 years old
b) 3 years old
c) 15 years old
d) 40 years old
e) Elderly
22) A 30-year-old patient presents with pain in the distal femur and proximal tibia.
Histopathology reveals mononuclear cells are proliferating rapidly (mitoses). Scattered
within the slid background are numerous osteoclast-type cells with 100 or more nuclei
that have identical features to those of the mononuclear cells. Which of the following is
the most likely?
a) Chondrosarcoma
b) Osteoid osteoma

DO NOT DISTRIBUTE - 61 -
Musculoskeletal 17Mar2009

c) Osteoblastoma
d) Aneurysmal bone cyst
e) Giant cell tumor
23.1) In adults, 75% of skeletal metastases (metastatic malignant tumors of bone)
originate from cancers of all of the following EXCEPT:
a) Prostate
b) Breast
c) Uterus
d) Kidney
e) Lung
23.2) In children, most skeletal metastases (to axial skeleton) arise from all of the
following EXCEPT:
a) Neuroblastoma
b) Wilms tumor
c) Osteosarcoma
d) Rhabdomyosarcoma
e) Chondroblastoma
24) Degenerative joint disease (osteoarthritis) is characterized by the progressive erosion
of articular cartilage, appears as an aging phenomenon, and most affects all of the
following locations EXCEPT:
a) Hips
b) Knees
c) Ribs & sternum
d) Proximal & distal interphalangeal joints
e) 1st carpometacarpal joint & 1st tarsometatarsal joint
25.1) A 60-year-old female presents with severe rheumatoid arthritis of the hands. The
chronic inflammation most likely deviated her digits in which direction?
a) Flexion
b) Extension
c) Radial side
d) Ulnar side
25.2) A patient presents with morning stiffness, arthritis in five joints, and palisaded
granulomas. Radiograph shows marked destruction of the epiphyseal bones in the fingers.
Which of the following would NOT likely be seen via serum blood and synovial tests?
a) Presence of rheumatoid factor
b) IgM antibody reaction with Fc portions of auto-IgG
c) Neutrophils in synovial fluid
d) Low protein content
e) Low mucin content
26) Juvenile rheumatoid arthritis starts before the age of 16 and is present for at least 6
weeks. Which of the following is a similarity between juvenile and adult rheumatoid
arthritis?
a) Oligoarthritis is more common
b) Systemic onset is more frequent
c) Large joints are affected more often than small joints
d) Rheumatoid nodules and factor are usually absent

DO NOT DISTRIBUTE - 62 -
Musculoskeletal 17Mar2009

e) Genetic association with particular haplotypes (DRB1)


27.1) Ankylosing spondylitis and Reactive arthritis are associated with what human
leukocyte antigen (HLA) serotype?
a) B27
b) DR2
c) DR3
d) DR4
e) DR5
27.2) Which of the following joints, along with apophyseal joints, is most affected by
ankylosing spondylitis?
a) Occipitoatlantal (OA)
b) Atlantoaxial (AA)
c) Lumbosacral
d) Sacroiliac
e) Transverse processes
28) Which of the following is NOT part of the symptoms triad associated with Reactive
arthritis?
a) Nongonococcal urethritis
b) Cervicitis
c) Osteoarthritis
d) Conjunctivitis
29) Psoriatic arthritis affects more than 10% of people with psoriasis, is less severe than
rheumatoid arthritis, and affects what location first in 50% of patients?
a) Distal interphalangeal joints
b) Ankles
c) Hips
d) Knees
e) 1st carpometacarpal joint
30.1) A 20-year-old patient presents with an acutely painful, hot, swollen knee.
Musculoskeletal exam yields restricted range of motion and fever. Lab tests show
leukocytosis and no gonococcal infection. Which of the following is the most likely?
a) Strep pyogenes
b) H. influenza type B
c) Serretia marcescens
d) E. coli
e) Staph aureus
30.2) A 16-year-old boy from Erie, Pennsylvania presents with achy knees and shoulders.
Weeks ago, they felt chest discomfort. Prior to that, they had been playing with friends in
the woods. The clinician suspects an immunologic response against antigens (e.g. ospA)
that cross-react with proteins in the joints. Which of the following is most likely?
a) Suppurative arthritis
b) Tuberculous arthritis
c) Lyme arthritis
d) Viral arthritis
f) Gouty arthritis

DO NOT DISTRIBUTE - 63 -
Musculoskeletal 17Mar2009

31) A patient presents with joint pain. Tests reveal crystallization of urates (trophi) within
the joint spaces. Which of the following is the most likely?
a) Pseudo-gouty arthritis
b) Tuberculous arthritis
c) Lyme arthritis
d) Viral arthritis
f) Gouty arthritis
32) A patient presents with severe osteoarthritis. Genetic testing reveals a mutation in the
ANKH gene. Intra-cellular crystal formation involving calcium pyrophosphate deposits
are found. Which of the following is the most likely?
a) Pseudo-gouty arthritis
b) Tuberculous arthritis
c) Lyme arthritis
d) Viral arthritis
f) Gouty arthritis
33) Which of the following locations is the most common for a ganglion (synovial) cyst?
a) Ankle
b) Wrist
c) Neck
d) Shoulder
e) Hip
34) Pigmented villonodular synovitis (PVNS), which is similar to giant cell tumor of the
tendon sheath (GCT), presents with joint pain, recurrent swelling, and red-brown to
mottled orange-yellow lesion on the joint synovium. What is the most common location
of PVNS?
a) Ankle
b) Wrist
c) Knee
d) Shoulder
e) Foot
35) Which of the following fat cell tumors (lipomas) is painful, along with the typically
soft texture and mobility?
a) Fibrolipoma
b) Conventional lipoma
c) Spindle cell lipoma
d) Angiolipoma
e) Pleomorphic lipoma
36) Along with proximal extremities, what location is common for liposarcomas?
a) Distal extremities
b) Base of skull
c) Retroperitoneal
d) Lumbar spine
e) Substernal
37) Which of the following locations is the most common for nodular fasciitis?
a) Dorsal forearm
b) Volar forearm

DO NOT DISTRIBUTE - 64 -
Musculoskeletal 17Mar2009

c) Palmar surface
d) Chest
e) Back
38) Myostitis ossificans most affects the musculature of which of the following
locations?
a) Distal extremities
b) Proximal extremities
c) Retroperitoneal
d) Lumbar spine
e) Chest
39) Which of the following is NOT a common anatomical location for benign
fibromatosis?
a) Penis
b) Palm of hand
c) Sole of foot
d) Breast
Match the subtype of desmoids tumor with the description:
40.1) Affects the musculature of the shoulder, chest, back a) Extra-abdominal
40.2) Affects abdominal wall in women during or after pregnancy b) Abdominal
40.3) Affects patient having familial adenomatous polyposis c) Intra-abdominal
41) Which of the following benign fibrous histiocytomas (of dermis or subcutaneous
tissue) affects children?
a) Dermatofibromas
b) Xanthogranulomas
c) Epithelioid histiocytomas
d) Reticulohistiocytomas
42) Along with proximal extremities, what location is common for malignant fibrous
histicytomas?
a) Distal extremities
b) Base of skull
c) Retroperitoneal
d) Lumbar spine
e) Uterus
43.1) Which of the following is NOT a common location of embryonal
rhabdomyosarcoma, seen in children?
a) Deep musculature of the extremities
b) Nasal cavity
c) Orbit
d) Middle ear
e) Prostate and paratesticular region
43.2) Which of the following is the most common location of alveolar
rhabdomyosarcoma, seen in early to mid-adolescence?
a) Deep musculature of the extremities
b) Nasal cavity
c) Orbit
d) Middle ear

DO NOT DISTRIBUTE - 65 -
Musculoskeletal 17Mar2009

e) Prostate and paratesticular region


44) Which of the following is the most common location of leiomyoma?
a) Nipples
b) Scrotum
c) Labia
d) Uterus
e) Skin erector pili
45) Leiomyosarcoma, which affects extremities and the retroperitoneum, differs from
leiomyoma histologically in that leiomyosarcoma shows:
a) Spindle cells that intersect at right angles and blunt-ended elongated nuclei
b) Spindle cells with interweaving fascicles and blunt-ended elongated nuclei
c) Cigar-shaped nuclei arranged in interweaving fascicles
d) Cigar-shaped nuclei and spindle cells that intersect at right angles
46) A 30-year-old patient presents with deep tissue pain near the knee and thigh. Lab
results show a classic bi-phasic histopathology associated with synovial sarcoma. The
slide would thus show spindle cell and:
a) Epithelial-like cells
b) Hyaline-like cells
c) Lysed red blood cells
d) Muscle-like cells
e) Bone-like cells

18 – Pathology: Peripheral Nerve & Skeletal Muscle


1.1) What type of cells produce myelin for nerve fibers?
a) Stellate cells
b) Sertoli cells
c) Schwan cells
d) Epithelial cells
e) Fibroblasts
1.2) Peripheral nervous system (PNS) myelin contains a higher number of
sphingomyelins and glycoproteins than the central nervous system (CNS).
a) True
b) False
1.3) Which of the following surrounds individual nerve fibers?
a) Epineurium
b) Perineurium
c) Endoneurium
2) Type I muscle fibers are high in ____ and type II muscle fibers are high in ____.
a) Glycolytic enzymes; Myoglobin and oxidative enzymes
b) Myoglobin and oxidative enzymes; Glycolytic enzymes
c) Glycolytic enzymes and myoglobin; Oxidative enzymes
d) Oxidative enzymes; Glycolytic enzymes and myoglobin
3) In Guillan-Barre syndrome, segmental demyelination occurs. This is due to
dysfunction of what type of cell?
a) Stellate cells
b) Sertoli cells

DO NOT DISTRIBUTE - 66 -
Musculoskeletal 17Mar2009

c) Schwan cells
d) Epithelial cells
e) Fibroblasts
4.1) In axonal degeneration, destruction of the axon leads to secondary disintegration of
its myelin sheath. Associated Wallerian degeneration occurs with which part of the nerve,
leading to muscle atrophy?
a) Proximal portion
b) Distal portion
c) Body
d) Soma
e) Axon
4.2) Type II muscle atrophy is associated with which of the following?
a) Colles fracture
b) Osteomyelitis
c) Fibromyalgia
d) Impingement of joint
e) Casting of fracture limb
5) Which of the following is evidence of muscle regeneration?
a) Multiple closely aggregated thinly myelinated small caliber axons
b) Multiple widespread thinly myelinated small caliber axons
c) Multiple closely aggregated heavily myelinated small caliber axons
d) Multiple closely aggregated heavily myelinated large caliber axons
e) Multiple widespread heavily myelinated large caliber axons
6) In segmental necrosis of muscle fibers, what is deposited over time along with fatty
infiltration?
a) Actin
b) Myosin
c) Collagen
d) Uric acid crystals
e) Bone & calcium
7.1) A patient presents with weakness of the distal limbs with rapidly advancing proximal
function “ascending paralysis.” Histology reveals inflammation and demyelination of
spinal nerve roots and peripheral nerves (radiculopathy). Which of the following is the
most likely?
a) Guillain-Barré syndrome
b) Tucot syndrome
c) Fugate disease
d) McCune-Albright syndrome
e) Maffucci syndrome
7.2) Which of the following would be the least likely to be associated with an immune-
mediated neuropathy?
a) Staph aureus
b) Campylobacter jejuni
c) Cytomegalovirus
d) Epstein-Barr virus
e) M. pneumonia

DO NOT DISTRIBUTE - 67 -
Musculoskeletal 17Mar2009

8.1) Which of the following leads to painful, vesicular skin eruptions affecting specific
sensory dermatomes?
a) Lepromatous leprosy
b) Tuberculoid leprosy
c) Diphtheria
d) Varicella-zoster virus
8.2) Which of the following causes inflammation injuries to cutaneous nerves and affects
nerve cell myelination?
a) Lepromatous leprosy
b) Tuberculoid leprosy
c) Diphtheria
d) Varicella-zoster virus
9.1) Which of the following usually presents at childhood with progressive muscle
atrophy of the calf and is usually associated with s peripheral myelin protein 22?
a) Hereditary motor and sensory neuropathy type II (HMSN-II)
b) Charcot-Marie-Tooth disease (CMT, HMSN-I)
c) Dejerin-Sottas disease (HMSN-III)
9.2) Which of the following presents in adult-hood and is associated with loss of
myelinated axons but does not show nerve enlargement?
a) Hereditary motor and sensory neuropathy type II (HMSN-II)
b) Charcot-Marie-Tooth disease (CMT, HMSN-I)
c) Dejerin-Sottas disease (HMSN-III)
9.3) Which of the following begins in early childhood and progresses slowly, involves
palpable nerves, and is manifested by delay in developmental milestones such as
acquisition of motor skills?
a) Hereditary motor and sensory neuropathy type II (HMSN-II)
b) Charcot-Marie-Tooth disease (CMT, HMSN-I)
c) Dejerin-Sottas disease (HMSN-III)
10.1) Patients with which of the following have a 50% chance of developing peripheral
neuropathy including distal symmetric sensory and sensorimotor neuropathy?
a) Myocardial infarction
b) Diabetes mellitus
c) Congestive heart failure
d) Asthma
e) Osteoblastoma
10.2) Peripheral neuropathy termed neuropathic beriberi is a result in deficiency of:
a) B12
b) B6
c) Vitamin E
d) Thiamine
e) Calcium
10.3) Heavy metals are a prominent cause of environmental toxic neuropathy.
a) True
b) False
11) Carpal Tunnel syndrome, affecting the median nerve at the wrist, is what type of
neuropathy?

DO NOT DISTRIBUTE - 68 -
Musculoskeletal 17Mar2009

a) Laceration
b) Avulsion
c) Compression
d) Lesion
e) Demyelinated
12) An infant presents with Werdnig-Hoffman disease, a common spinal muscular
atrophy (infantile motor neuron disease) involving the SMN1 gene on what
chromosome?
a) 3
b) 5
c) 13
d) 22
e) 23
13) A patient presents with pseudohyperatrophy of the calf and is diagnosed with
muscular dystrophy. What genetic pattern does this disease show?
a) Autosomal dominant
b) Autosomal recessive
c) X-linked
d) Y-linked
e) Co-dominant
14) Which of the following presents with prominent contractures, especially of the
elbows and ankles?
a) Facioscapulohumeral muscular dystrophy
b) Oculopharyngeal muscular dystrophy
c) Emery-Dreifuss muscular dystrophy
d) Congenital muscular dystrophy
e) Myotonic dystrophy
15) A patient presents with difficulty “releasing their grip after a firm handshake.”
Histology of the subsarcolemma band reveals a distinct ring fiber. Patient history reveals
a family history of a disorder, which has gotten worse over generations (anticipation).
Which of the following is the most likely?
a) Facioscapulohumeral muscular dystrophy
b) Oculopharyngeal muscular dystrophy
c) Emery-Dreifuss muscular dystrophy
d) Congenital muscular dystrophy
e) Myotonic dystrophy
16.1) A patient presents to the Emergency Room after having malignant hyperthermia
(hyperprexia) during surgery. It is known that the skeletal muscle sodium channel protein
(SCN4A) has a mutation. Which of the following types of periodic paralysis is likely for
this patient?
a) Hypernatremia
b) Hyponatremia
c) Hyperkalemia
d) Hypokalemia
e) Hypercalcemia

DO NOT DISTRIBUTE - 69 -
Musculoskeletal 17Mar2009

16.2) What type of channel is encoded for by the gene that causes hypokalemic periodic
paralysis?
a) Sodium
b) Potassium
c) Calcium
d) Chlorine
e) Magnesium
17) Which of the following is NOT a clinical feature of congenital myopathy?
a) Floppy Baby syndrome
b) Early onset in life
c) Rapid progression
d) Generalized muscle weakness
e) Hypotonia
18.1) A patient presents with paracrystalline “parking lot” mitochondrial inclusions
affecting mtDNA. What type of inheritance will be seen?
a) Dominant
b) Recessive
c) Paternal
d) Maternal
e) None (death)
18.2) Abnormalities of the carnitine transport system will lead to what type of inclusions?
a) Uric acid
b) Lipid
c) Polysaccharides
d) Collagen
e) Magnesium salts
19.1) Which of the following presents with alilac or heliotropic discoloration of the upper
eyelids with periorbital edema?
a) Inclusion body myositis
b) Polymyositis
c) Dermatomyositis
19.2) Which of the following affects distal muscles, especially the extensors of the knee
and flexors of the wrist and fingers?
a) Inclusion body myositis
b) Polymyositis
c) Dermatomyositis
Match the myopathy with the description:
20.1) Proximal muscle weakness and atrophy a) Thyrotoxic
20.2) Exophthalmic ophthalmoplegia b) Ethanol
20.3) Rhabdomyolysis with globinuria c) Drug-induced
21) What type of receptors are damaged (lost) in the autoimmune disease myasthenia
gravis (MG)?
a) Primary muscle spindle receptors
b) Secondary muscle spindle receptors
c) Golgi tendon receptors
d) Acetylcholine receptors

DO NOT DISTRIBUTE - 70 -
Musculoskeletal 17Mar2009

e) Acetylcholine esterase receptors


22) Lambert-Eaton myasthenic syndrome affects which of the following?
a) Nerve axon
b) Nerve soma
c) Nerve dendrite
d) Myelin sheath
e) Neuromuscular junction

19 – Orthopedics: Textbook Questions


1) A 46-year-old male patient complains of axial neck pain and right-sided scapular pain.
The patient perceives the pain as deep and aching. The pain began 6 months after a motor
vehicle accident in which the patient was the front passenger. The patient denies motor
weakness, numbness, or burning. The patient has taken over-the-counter Advil for his
pain with little relief. What is his likely diagnosis?
a) Right cervical radiculopathy
b) Atlanto-occipital dislocation
c) Anterior cord syndrome
d) Right cervical zygapophyseal joint disease
e) Muscle strain
2) A 51-year-old female tennis player presents with 2 months of anterior shoulder pain
that is more severe when she plays tennis. Her pain is relieved with rest. On exam, her
shoulder is very tender, limiting both active and passive range of motion. On X-ray, a
large calcification is seen just above the greater tuberosity. You diagnose her with
calcified tendonitis and treat her for 6 weeks with a sling, modification of activities,
NSAIDs, and heat. However, despite this treatment, her symptoms persist. The most
appropriate next step in treatment is:
a) Shoulder immobilization
b) Intra-articular steroid injection
c) Ultrasound-guided aspiration and lavage of the calcification
d) Open surgical removal of the calcification
e) Arthroscopic surgical removal of the calcification
3) A 46-year-old male plumber complains of right elbow pain that is worse at work.
Recently, even picking up a pencil or the phone has become painful. The pain is located
at the lateral epicondyle. On exam, resisted wrist extension reproduces the symptoms.
What is the diagnosis and what is the first-line therapy?
a) Ulnar collateral ligament injury; physical therapy, NSAIDs, rest
b) Lateral epicondylitis; physical therapy, ultrasound, steroid injection, ice
c) Ulnar collateral ligament injury; surgery
d) Lateral epicondylitis; surgery
e) Osteochondritis dissecans; arthroscopic lesion removal, abrasion chondroplasty
4) Kyphoplasty is used to treat:
a) Discongenic chronic low back pain
b) Lumbosacral radiculopathy
c) Osteoporotic vertebral compression fracture
d) Medial meniscus tear
e) Anterior cruciate ligament tear

DO NOT DISTRIBUTE - 71 -
Musculoskeletal 17Mar2009

5) A patient with a C5 radiculopathy will typically complain of:


a) Weak shoulder abduction, weak elbow flexion, numbness in her lateral arm and
elbow
b) Weak shoulder abduction, weak elbow extension, numbness in her lateral arm
and elbow
c) Weak elbow flexion, weak wrist extension, numbness in her fourth and firth
digits
d) Weak elbow flexion, weak elbow extension, numbness in her arm and third
digit
e) Weak wrist flexion, weak finger flexion, numbness in her first three digits
6) A 62-year-old male presents with buttock pain that radiates down his lateral thigh into
the top of his foot. He denies any weakness. He states that the pain is worse when leaning
forward to tie his shoes. He denies any bowel or bladder changes, recent weight loss, or
urinary symptoms. On exam there is no gait abnormality. Straight-leg raise reproduces
symptoms at 40 degrees of flexion. His extensor and Achilles reflexes are brisk and
symmetrical. This patient most likely has:
a) L4 radiculopathy
b) L5 radiculopathy
c) S1 radiculopathy
d) Zygapophyseal joint pain with referral pain pattern in the leg
e) Discogenic low back pain
7) Cubital tunnel syndrome is a compression neuropathy of:
a) The median nerve at the elbow that results in numbness and tingling in the first
three digits
b) The ulnar nerve at the elbow that results in numbness, tingling, and weakness
in the first three digits
c) The ulnar nerve at the wrist that results in numbness and tingling in the fourth
and fifth digits
d) The radial nerve at the elbow that results in numbness, tingling, and weakness
in the fourth and fifth digits
e) The ulnar nerve at the elbow that results in numbness, tingling, and weakness
in the fourth and fifth digits
8) In a patient diagnosed with slipped capital femoral epiphysis, the initial treatment
consists of:
a) Physical therapy, rest, and heat modalities
b) Epidural steroid injections under fluoroscopic guidance
c) Modification of activities, splinting, physical therapy
d) Surgical intervention
e) Physical therapy and/or intra-articular injection of hyaluronic acid
9) A 16-year-old female patient presents to the clinic complaining of right anterior knee
pain. She denies any history of trauma. She is a dancer and the pain is worse with
movement. On exam, the anterior tibial tubercle is tender. There is no point line
tenderness or restriction of movement. Her Q-angle is 16 degrees. She is able to squat on
both feet, but this causes pain in her knees. The most likely diagnosis in this patient is:
a) Osteochondritis dissecans
b) Anterior cruciate ligament injury

DO NOT DISTRIBUTE - 72 -
Musculoskeletal 17Mar2009

c) Patellofemoral syndrome
d) Osgood-Schlatter disease
e) Osteoarthritis
10) A 40-year-old male presents with knee pain and swelling. He reports that the pain is
mostly on the medial side of his knee. His symptoms began immediately after being
tackled from the side during a weekend football game with friends. On exam, there is a
normal Q-angle. There is excessive gapping when a valgus stress is applied to the knee in
30 degrees of flexion. There is no excessive gapping when the same valgus stress is
applied to the knee in full extension. The most likely diagnosis in this patient is:
a) Medial meniscal tear
b) Anterior cruciate ligament tear
c) Anterior cruciate ligament tear and medical collateral ligament tear
d) Medical collateral ligament tear
e) Posterior cruciate ligament tear
11) A type III Garden classification femoral neck fracture refers to:
a) A valgus impaction of the femoral head
b) A varus displacement of the femoral head
c) A complete, nondisplaced fracture
d) A complete loss of continuity between the fragments
e) An avulsed fragment of bone
12) A 32-year-old male patient presents with a history of anterior shoulder pain that
began 2 months ago. The pain is intermittent and is made worse by playing tennis. In the
last week, the pain has kept him awake at night occasionally and prevents him from
sleeping on his affected side. On exam, he has a positive Neer test and a negative speed
test. The patient is able to abduct and slowly lower his arm without difficulty. The most
likely diagnosis is:
a) Rotator cuff impingement
b) Rotator cuff partial-thickness tear
c) Rotator cuff full-thickness tear
d) Bicipital tendonitis
e) Acromioclavicular joint injury
13) In this test, the examiner flexes one leg to 90 degrees while stabilizing the back of the
patient’s thigh. The examiner leans on the heel of the patient to compress the medial and
lateral menisci between the tibia and femur. The examiner then rotates the leg into
internal and external rotation while maintaining a downward compression force. What is
the name of this test, and what is it used to diagnose?
a) McMurray test; meniscal tear
b) Wilson test; meniscal tear
c) McMurray test; collateral ligament tear
d) Wilson test; collateral ligament tear
e) Apley compression test; meniscal tear
14) A grade III ankle spain:
a) Involves only a stretch of the ligament
b) Is a complete rupture of the ligament
c) Is a partial ligament tear
d) Is a crush injury

DO NOT DISTRIBUTE - 73 -
Musculoskeletal 17Mar2009

e) Is a complete tear of at least two ligaments


15) A 43-year-old female presents with a complaint of wrist pain for the last 2 days. On
further questioning, she reveals that the pain is on the radial side of her wrist and began
shortly after gardening for the first time in years. On exam, there is tenderness over the
tip of the radial styloid. She has a positive Finkelstein test. She is diagnosed with de
Quervain tendosynovitis. The most appropriate imaging studies to order are:
a) X-ray only
b) X-ray and MRI
c) X-ray and CT
d) MRI only
e) No imaging studies are necessary
16) A Salter-Harris type II fracture is:
a) A displaced or “slipped” physis
b) A fracture through the metaphysic, physis, and epiphysis
c) A fracture above the physis, involving the metaphysis
d) A fracture below the physis, involving only the epiphysis
e) A crush injury
17) Select the primary hip extensor muscle and its nerve innervation:
a) Gluteus medius; superior gluteal nerve
b) Gluteus medius; inferior gluteal nerve
c) Gluteus maximus; superior gluteal nerve
d) Gluteus maximus; inferior gluteal nerve
e) Gluteus maximus; obturator nerve
18) The three most common causes of chronic low back pain, in descending order, are:
a) Discogenic; sacroiliac; zygapophyseal
b) Myofascial; sacroiliac; discogenic
c) Discogenic; zygapophyseal; contracted hip flexors
d) Buckled ligamentum flavum; discogenic; contracted hip extensors
e) Discogenic; zygapophyseal; sacroiliac
19) Kienböck disease is:
a) Avascular necrosis and collapse of the lunate
b) Avascular necrosis and collapse of the scaphoid
c) Avascular necrosis and collapse of the triquetrium
d) A distal radius fracture resulting from a fall onto an extended hand
e) A distal radius fracture with ulna dislocation after a traumatic event
20) The rotator cuff muscles are:
a) Supraspinatus, infraspinatus, teres major, teres minor
b) Supraspinatus, infraspinatus, teres major, subscapularis
c) Supraspinatus, teres minus, subscapularis
d) Deltoid, supraspinatus, infraspinatus, teres minor
e) Supraspinatus, infraspinatus, teres minor, subscapularis
21) A 32-year-old female presents with complaint of wrist pain. She states the pain began
after falling onto her outstretched hand 5 days ago. She was hoping the pain would go
away on its own, but when it did not resolve she decided to come to the clinic. You
palpate her wrist and find tenderness in the anatomic snuff box. You order plain film

DO NOT DISTRIBUTE - 74 -
Musculoskeletal 17Mar2009

radiographs and a CT scan. Your diagnosis of a scaphoid fracture is confirmed. The


fracture is displaced. Selected the most appropriate initial management:
a) Open reduction and internal fixation
b) Closed reduction
c) A splint for 6 weeks followed by repeat radiograph and/or CT scan
d) No treatment; close observation and follow-up are all that is needed
e) Ice and physical therapy with early mobilization of the wrist joint to avoid
contracture
22) Following a posterior approach for a hip replacement, hip precautions include:
a) No hip flexion past 90 degrees, no hip abduction beyond neutral, and no hip
internal rotation past neutral
b) No hip flexion past 90 degrees, no hip adduction beyond neutral, and no hip
internal rotation past neutral
c) No hip extension past 30 degrees, no hip abduction beyond neutral, and no hip
internal rotation past neutral
d) No hip extension past 30 degrees, no hip abduction past 20 degrees, and no
internal rotation past neutral
e) No hip flexion past 90 degrees, no hip adduction beyond neutral, and no hip
external rotation
23) Accidental damage to the tibial nerve during dissection of the posterior knee may
result in:
a) Weak knee extension
b) Weak knee flexion
c) Weak ankle dorsiflexion
d) Weak ankle plantar flexion
e) Weak extensor hallucis longus
24) A 26-year-old female presents with complaint of knee pain. The pain is described as
dull and aching and located primarily in the anterior knee. She states that the pain is
worse when she goes down stairs or sits for a prolonged period. She denies any history of
knee trauma. On exam, her Q-angle is measured to be 20 degrees and the patella shows
some abnormal lateral tracking. The joint line is not tender. There is retropatellar
tenderness. Slight crepitus is appreciated with knee flexion and extension. Based on the
information given, the most likely diagnosis is:
a) Osgood-Schlatter disease
b) Patella fracture
c) Medial meniscus injury
d) Early knee osteoarthritis
e) Patellofemoral syndrome
25) The platysma is a superficial muscle in the ____ and is innervated by the ____.
a) Neck; axillary nerve
b) Neck; CN VII
c) Shoulder; Facial nerve
d) Forearm; Median nerve
e) Forearm; Radial nerve
26) A football player sustains an injury after falling onto his shoulder. Immediately
following the injury, the player reports that his shoulder has become painful and swollen

DO NOT DISTRIBUTE - 75 -
Musculoskeletal 17Mar2009

over the end of the clavicle, and the patient has difficulty adducting his arm across his
body. If this patient has sustained a type I acromioclavicular injury, plain film
radiography will reveal:
a) No displacement of the clavicle
b) Up to 25% displacement of the clavicle with no increase in coracoclavicular
distance
c) Up to 50% displacement with no increase in coracoclavicular distance
d) Up to 50% displacement with an increase in coracoclavicular distance
e) Greater than 50% displacement with emergent surgery indicated
27) A 38-year-old woman presents with numbness and tingling in her first three digits.
She also reports a vague aching sensation in her wrist. On further questioning, the patient
reports that her symptoms are worse while typing at her desk. You suspect this patient
may have carpal tunnel syndrome. The most sensitive physical examination test for
carpal tunnel syndrome is:
a) Tinel sign
b) Phalen test
c) McMurray test
d) Compression test
e) Finkelstein test
28) Using provocative discogram with post-discogram CT evaluation to evaluate for
discogenic back pain, a grade III fissure in the annulus fibrosus refers to a fissure that
penetrates:
a) The inner third of the annulus
b) The inner half of the annulus
c) The outer third of the annulus
d) Circumferentially around less than half of the rim of the annulus
e) Circumferentially around the rim of the annulus
29) An infant is evaluated and found to have evidence of hip instability. The most
appropriate diagnostic test is:
a) X-ray
b) CT
c) MRI with contrast
d) MRI without contrast
e) Ultrasonography
30) Injury to the common peroneal nerve results in decreased ability to:
a) Plantarflex the foot
b) Extend the knee and dorsiflex the foot
c) Flex the knee and dorsiflex the foot
d) Dorsiflex the foot and invert the foot
e) Dorsiflex the foot and evert the foot
31) A 36-year-old male presents with a complaint of slowly progressive right medial heel
pain. The patient states that the pain is worse when he first wakes up and takes his first
step of the day. On physical examination, there is no notable atrophy of the muscles.
Reflexes are brisk and intact bilaterally. Passive dorsiflexion exacerbates the patient’s
symptoms. Based on the information provided, the first diagnostic imaging to order is:
a) X-ray

DO NOT DISTRIBUTE - 76 -
Musculoskeletal 17Mar2009

b) CT
c) MRI
d) Bone scan
e) No imaging is necessary
32) The Hawkins test is used to evaluate for:
a) Meniscus injury
b) Posterior shoulder instability
c) Biceps tendon instability
d) Shoulder impingement syndrome
e) Osteochondritis dissecans in the elbow
33) Supracondylar fractures are the most common type of pediatric elbow fracture. In the
Gartland classification of supracondylar fractures, type II refers to:
a) Anterior gapping with some rotational mal-alignment and an intact posterior
hinge
b) Anterior gapping with some rotational mal-alignment and an unstable posterior
hinge
c) Anterior gapping without rotational mal-alignment and an intact posterior hinge
d) Anterior gapping without rotational mal-alignment and an unstable posterior
hinge
e) No anterior gapping and moderate rotational mal-alignment
34) The tendons of the extensor carpi radialis longus and extensor carpi radialis brevis
muscles are found in the:
a) First dorsal wrist tunnel
b) Second dorsal wrist tunnel
c) Third dorsal wrist tunnel
d) Forth dorsal wrist tunnel
e) Fifth dorsal wrist tunnel
35) A young girl who is skeletally immature and has a scoliotic curve should be treated
with full-time bracing if her curve is:
a) Less than 10 degrees
b) Between 10 and 20 degrees
c) Between 20 and 40 degrees
d) Greater than 60 degrees
e) Full-time bracing is never appropriate in a skeletally immature adolescent
36) The quadriceps muscles include the:
a) Rectus femoris, vastus lateralis, vastus medialis, iliopsoas
b) Rectus femoris, vastus lateralis, vastus medialis, biceps femoris
c) Rectus femoris, biceps femoris, vastus intermedius, sartorius
d) Biceps femoris, vastus lateralis, vastus medialis, vastus intermedius
e) Rectus femoris, vastus lateralis, vastus medialis, vastus intermedius
37) A 14-year-old male presents with a complain of a limp and left groin and knee pain.
He states that the pain is dull and aching. He states he has no shooting pains, numbness,
tingling, or burning. He cannot recall any history or trauma, but the pain does seem to be
worse when he plays basketball. On physical examination, the patient is moderately
overweight. Gait examination reveals outtoeing and mild antalgic gain pattern. When his
hip is externally rotated, the patient tends to flex his hip. The most likely diagnosis is:

DO NOT DISTRIBUTE - 77 -
Musculoskeletal 17Mar2009

a) Legg-Calvé-Perthes disease
b) Axial low back pain with referral pain pattern
c) L4 radiculopathy
d) Transient synovitis
e) Slipped capital femoral epiphysis
38) A 22-year-old male presents with a complaint of left knee pain. The pain is aching in
quality. Occasionally, the patient reports that his knee catches and locks. The patient does
not recall any history of his knee “giving out.” The patient cannot recall any history of
significant trauma except for a large bruise on his knee that he suffered after a football
collision two years ago. On examination, the patient is instructed to lie on his back. His
knee is flexed to 90 degrees and put into internal rotation and slowly extended. When his
knee is at 30 degrees of flexion, he reports significant pain. With his knee still in 30
degrees of flexion, his knee is rotated into external rotation and the pain is relieved. This
is called the ____ and it is diagnostic of ____.
a) Wilson sign; osteochondritis dissecans
b) Wilson sign; medial meniscus injury
c) McMurray test; lateral meniscus injury
d) McMurray test; osteochondritis dissecans
e) Yergason test; medial meniscus injury
39) Using the Schatzker classification, a tibial plateau fracture that involves only the
medial tibial plateau is a:
a) Type I fracture
b) Type II fracture
c) Type III fracture
d) Type IV fracture
e) Type V fracture
40) An infant is found to have an inverted left heel, forefoot adduction and supination,
and the sole of the foot faces posteromedially. A clinical diagnosis of idiopathic club foot
(equinovarus deformity) is made. The next appropriate step in the management of this
patient is:
a) Radiographic examination
b) CT evaluation
c) Casting with or without Achilles tendon lengthening
d) No casting; proceed with immediate surgical correction
e) MRI
41) A 33-year-old woman presents with a complaint of pain between the second and third
web spaces of her right foot. On further questioning, she reports that the “pain” is really
more of a burning sensation. There is tenderness over the involved web space. The most
likely diagnosis is:
a) Ganglion cyst
b) Hallux valgus
c) Stress fracture
d) Interdigital neuroma
e) L5 radicular pain
42) Monteggia fracture is characterized by:
a) Isolated ulna fracture

DO NOT DISTRIBUTE - 78 -
Musculoskeletal 17Mar2009

b) Proximal radius fracture and radial head dislocation


c) Crush fracture of the distal ulna and radial head dislocation
d) Distal ulna fracture and radial head dislocation
e) Proximal ulna fracture and radial head dislocation
43) During a posterior approach to the hip, the sciatic nerve is most commonly found to
be exiting:
a) Below the piriformis
b) Above the piriformis
c) Through the gluteus maximus
d) Through the obturator internus
e) The sciatic nerve is not encountered in the posterior approach to the hip
44) The primary external rotator muscles of the shoulder are the infraspinatus and teres
minor muscles. Respectively, these two muscles are innervated by the ____ nerve and the
____ nerve.
a) Suprascapular; Subscapular
b) Subscapular; Axillary
c) Suprascapular; Axillary
d) Axillary; Suprascapular
e) Suprascapular; Subclavius
45) Injury to the median nerve during surgery of the elbow may result in loss of:
a) Forearm pronation, wrist and digit extension, and thumb apposition
b) Forearm supination, wrist and digit extension, and thumb apposition
c) Forearm pronation, wrist extension, digit flexion, and thumb apposition
d) Sensation on the lateral portion of the palm and the palmar part of the first
three digits, loss of forearm pronation, and loss of wrist and digit extension
e) Forearm pronation, wrist and digit flexion, and thumb apposition
46) A patient presents with shooting pain that radiates down the neck and arm into the
hand. The examiner passively extends the patient’s neck and rotates it to the painful side.
The examiner then applies gentle axial compression to the top of the head. This is called
the ____ and it is used to help diagnose ____.
a) Johnson test; cervical radicular symptoms
b) Johnson test; cervical zygapophyseal joint disease
c) Spurling test; cervical radicular symptoms
d) Spurling test; cervical zygapophyseal joint disease
e) Spurling test; cervical spondylotic myelopathy
47) The O’Brien compression test is useful in differentiating a superior labral anterior
posterior (SLAP) lesion and acromioclavicular (AC) joint disease. The O’Brien test is
performed by having the patient stand with the shoulder flexed at 90 degrees and the
elbow in full extension. Next, the shoulder is adducted 10 to 15 degrees and internally
rotated so that the thumb points to the floor. From behind the patient, the examiner then
applies a downward-directed force to the arm as the patient resists. The patient’s hand is
then supinated and the maneuver is repeated. The patient has a positive O’Brien test,
suggesting SLAP lesion, if:
a) Pain is worse when the hand is in pronation, and provided that the pain is
experienced at the AC joint

DO NOT DISTRIBUTE - 79 -
Musculoskeletal 17Mar2009

b) Pain is worse when the hand is in pronation, and provided that the pain is
experienced within the shoulder
c) Pain is worse when the hand is in supination, and provided that the pain is
experienced at the AC joint
d) Pain is worse when the hand is in supination, and provided that the pain is
experienced within the shoulder
e) Pain is equally severe with the hand in supination and pronation, and the pain is
experienced within the shoulder and radiates halfway down the lateral aspect of
the arm
48) In contrast to other parts of the body, osteochondritis dissecans of the ankle is thought
to be most likely caused by:
a) Ischemia
b) Trauma
c) Genetics
d) Idiopathic
e) Radiation exposure
49) The Thompson test is used to evaluate for:
a) Knee medial meniscus tear
b) Hip flexor contraction
c) Hip extensor contraction
d) Achilles tendon rupture
e) Achilles tendonitis
50) After a total hip replacement using polymethylmethacrylate cement for fixation, the
patient may return to weight-bearing as tolerated:
a) One day after surgery
b) Two weeks after surgery
c) Four weeks after surgery
d) Six weeks after surgery if there is evidence of bone healing on radiographs
e) No sooner than 8 weeks after surgery regardless of evidence of bone healing on
radiographs

20 – Rheumatology: Rheumatoid Arthritis


1) Which of the following are immunogenetic markers commonly seen in patients with
rheumatoid arthritis?
a) HLA-B27 & HLA-DR1
b) HLA-DR1 & HLA-DR2
c) HLA-DR2 & HLA-DR4
d) HLA-DR4 & HLA-DR1
e) HLA-DR2 & HLA-B27
2) Rheumatoid factor can be detected in all of the following EXCEPT:
a) Primary Sjögren syndrome
b) SLE
c) Hepatitis C
d) Systemic vasculitis
e) Amyloid disease

DO NOT DISTRIBUTE - 80 -
Musculoskeletal 17Mar2009

3) Which of the following antibodies is present at the onset of rheumatoid arthritis and in
higher titer associated with progressive erosive disease?
a) Porcine pancreatic amylase (Anti-PPA)
b) Citrillinated protein (Anti-CCP)
c) Anti-SSB (La)
d) Anti-SSA (Ro)
e) Anti-Sm
4) Prolonged joint stiffness and “gelling” is seen with recurrence when a patient with
rheumatoid arthritis rests. Which of the following joints is the least likely to be affected?
a) Metacarpophalangeal (MCP)
b) Proximal interphalangeal (PIP)
c) Distal interphalangeal (DIP)
d) Metatarsophalangeal (MTP)
e) Knee
5) Patients with rheumatoid arthritis often suffer from fatigue, muscle pain, and weight-
loss. In patients with active arthritis, how long will morning joint stiffness last?
a) At least 1 minute
b) At least 5 minutes
c) At least 15 minutes
d) At least 30 minutes
e) At least an hour
6) How is cervical spine involvement diagnosed in rheumatoid arthritis?
a) MRI scan of each cervical vertebra
b) Nuclear bone imagining scan
c) Flexion/extension radiograph showing subluxations
d) Rotational radiographs showing sublucations
e) Side-bending radiographs showing nerve or vessel impingement
7) Which of the following is NOT a symptom associated with cervical spine involvement
in patients with rheumatoid arthritis?
a) Shooting pain down the posterior thighs
b) Tetraplegia (quadriplegia)
c) Cervical myelopathy
d) Drop (falling) attacks
e) Hand weakness or parasthesias
8) Which of the following patients with rheumatoid arthritis and cervical spine
involvement would be indicated for surgical treatment?
a) Extreme pain that subsides with head rotation
b) Syncope with slight flexion or extension
c) Minor subluxation of C6 over C7
d) Intractable neck stiffness
e) Greatly decreased range of cervical motion
9) A patient who was previously diagnosed with Lyme disease presents with complaint of
a “large bubble behind their knee the popped.” They have knee pain and can feel the cyst
fluid in their calf. Eccymosis is seen around the ankle (crescent sign). Intra-articular
injection of which of the following can help treat this patient?
a) Gold therapy

DO NOT DISTRIBUTE - 81 -
Musculoskeletal 17Mar2009

b) NSAIDS (COX-2 blockers)


c) Thymus removal
d) Corticosteroids
e) Anti-TNF-! (infliximab)
10) A patient presents with diffuse swelling between finger joints. Palpable grating is felt
within the flexor tendon sheaths in the palms with movement of the digits. The patient’s
symptoms decrease with NSAIDS and corticosteroid injections. Which of the following
does the patient most likely have?
a) SLE
b) Rheumatoid arthritis
c) Septic arthritis
d) Vasculitis
e) Tenosynovitis
11) Rheumatoid arthritis is the second most common etiology associated with carpal
tunnel syndrome. Which of the following is the most common?
a) Diabetes
b) Vasculitis
c) Sepsis
d) Pregnancy
e) Epilepsy
12) Which of the following is used for prolonged symptoms of carpal tunnel syndrome?
a) Resting splints
b) Anti-inflammatory drugs
c) Ice & compression
d) Glucocorticosteroid injections
e) Surgical aponeurotic sheath
13) Which of the following rheumatoid arthritis presentations involves hyper-extension
of the distal interphalangeal joint and flexion of the proximal interphalangeal joint?
a) Boutonniere deformity
b) Swan-neck deformity
c) Ulnar deviation
d) None of the above
14) Breakdown of rheumatoid nodules can lead to infection of the bursae, bone, or joints.
Which of the following is a common location of rheumatoid nodules?
a) Heart
b) Lungs
c) Skin
d) Sclera
e) Dura mater
15) Patients with severe, deforming arthritis and a high titer of rheumatoid factor are at
risk for which of the following?
a) SLE
b) Stress fractures
c) Septic arthritis
d) Vasculitis
e) Tenosynovitis

DO NOT DISTRIBUTE - 82 -
Musculoskeletal 17Mar2009

16) Which of the following is associated with palpable purpura or cutaneous ulcerations,
particularly over the malleoli?
a) Obliterative endarteropathy
b) Leukocytoclastic vasculitis
c) Rheumatoid arthritis
d) Rheumatoid vasculitis
17) In rheumatoid pleural effusions, which of the following is characteristic of the pleural
fluid exudate?
a) High concentrations of iron
b) High concentrations of leukocytes
c) Low concentrations of calcium
d) Low concentrations of glucose
e) High concentrations of ADP
18) A patient presents with systemic disease. Pulmonary nodules are seen showing a coin
lesion as well as some nodules that cavitate. A bronchopleural fistula is found as well.
Which of the following is the likely cause of this presentation?
a) Obliterative endarteropathy
b) Leukocytoclastic vasculitis
c) Rheumatoid arthritis
d) Hemochromatosis
e) Primary Sjögren syndrome
19) a patient presents with difficulty breathing and diffuse, dry crackles on lung
auscultation. Radiography shows a reticular nodular pattern in the lung bases. The
clinician believes the patient has interstitial fibrosis, which is highly associated with
which of the following?
a) Working in a coal mine
b) Cigarette smoking
c) Lupus (SLE)
d) Congestive heart failure (CHF)
e) Asbestos inhalation
20) What high-resolution imaging modality is used to differentiate between the different
interstitial rheumatoid lung syndromes?
a) Chest x-ray
b) Computed tomography (CT)
c) Magnetic resonance imaging (MRI)
d) Positron emission tomography (PET)
e) Ultrasound
21) A patient with rheumatoid arthritis presents with unexplained edema and ascities.
Although the patient does not present with symptoms of pericarditis, the clinician does
studies and confirms recurrent effusive pericarditis. Which of the following is necessary
for this patient?
a) Treatment with beta-blockers
b) Treatment with anti-hypertensives
c) Treatment with diuretics
d) Surgical pericardectomy
e) No treatment is necessary

DO NOT DISTRIBUTE - 83 -
Musculoskeletal 17Mar2009

22) Which of the following would NOT be increased in a rheumatoid arthritis patient
with liver impairment?
a) Alkaline phosphatase
b) Aspartate aminotransferase
c) Gamma-glutamyltransferase
d) Acute-phase proteins
e) Albumin
23) Which of the following is the most common ophthalmic complication of rheumatoid
arthritis?
a) Keratoconjunctitis sicca
b) Glaucoma
c) Cataracts
d) Retinopathy
e) Strabismus
24) Which of the following is commonly seen in patients with rheumatoid arthritis but
may not be detected for months after the initial onset of joint symptoms?
a) Normocytic anemia (Hb 10)
b) CCP antibodies
c) Rheumatoid factor (IgM)
d) Leukocytosis, thrombocytosis
e) Hypoalbuminemia, hypergammaglobulinemia
25) What is the frequency of rheumatoid factor in patients with rheumatoid arthritis?
a) 100%
b) 99%
c) 90%
d) 50%
e) 1%
26) Which of the following diseases is the least likely to have a high-titer rheumatoid
factor?
a) Rheumatoid arthritis
b) Sjögren syndrome
c) Mixed cryoglobulinemia
d) Subacute bacterial endocarditis
e) Chronic fibromyalgia
27) Which of the following is the most specific for rheumatoid arthritis and may be
present when rheumatoid factor is absent?
a) Anti-PPA
b) Anti-CCP
c) Anti-SSB (La)
d) Anti-SSA (Ro)
e) Anti-Sm
28) The synovial fluid in patients with rheumatoid arthritis is cloudy, light yellow, has
poor viscocity, and has 10,000-75,000 leukocytes per microliter, which are
predominantly:
a) Neutrophils
b) Eosinophils

DO NOT DISTRIBUTE - 84 -
Musculoskeletal 17Mar2009

c) Basophils
d) Lymphocytes
e) Macrophages
29) Which of the following is a radiographic finding seen particularly at the end-stage of
rheumatoid arthritis?
a) Soft tissue swelling
b) Periarticular osteoporosis
c) Symmetric narrowing of the joint space
d) Subluxation of joints
e) Marginal bony erosions of the joint
30) Which of the following is NOT a criterion for diagnosing rheumatoid arthritis set by
the American Rheumatism Association?
a) Inflammatory joint symptoms lasing more than 6 weeks
b) Arthritis of a single joint
c) Arthritis in the wrist, MCP, or PIP joints
d) One or more hours of morning stiffness
e) Rheumatoid nodules, serum rheumatoid factor
31) After 15 years of rheumatoid arthritis, what percentage of patients are completely
disabled and unable to work?
a) 1%
b) 5%
c) 10%
d) 15%
e) 25%
32) Patients at a high risk of early disability from rheumatoid arthritis have persisting
polyarthritis with:
a) Positive rheumatoid factor
b) Anti-CCP antibody
c) Either A or B
d) Both A and B
e) Neither A nor B
33) Which of the following is the initial medication used in the treatment of rheumatoid
arthritis?
a) DMARDs
b) NSAIDs
c) Steroids
d) Allopurinol
e) Febuxostat
34) Which of the following is the second-line treatment for rheumatoid arthritis as they
are slow-acting and may need months before their effect can be assessed?
a) DMARDs
b) NSAIDs
c) Steroids
d) Allopurinol
e) Febuxostat
35) which of the following is NOT a DMARD?

DO NOT DISTRIBUTE - 85 -
Musculoskeletal 17Mar2009

a) Methotrexate
b) Hydroxychloroquine
c) Sulfasalazine
d) Minocycline
e) Dantrolene
f) Leflunomide
g) Cyclosporine
h) Azathioprine
i) Anticytokines
36) Uninterrupted treatment with DMARDs should last for how long after rheumatoid
arthritis is diagnosed?
a) 24 hours
b) 3-6 days
c) 3-6 weeks
d) 3-6 months
e) 3-6 years
37) Which of the following is considered the most important therapeutic option for
preserving or enhancing function in rheumatoid arthritis?
a) NSAIDs
b) DMARDs
c) Oral steroids
d) Steroid injections
e) Surgical procedures
38) A patient with rheumatoid arthritis has been treated for nearly a year with DMARDs.
They have tried NSAIDs and oral steroids. They now have intractable pain and
compromised joint function. Which of the following is the best option for this patient?
a) Synovectomy
b) Arthroplasty
c) Changing DMARD dosage
d) Changing to a new DMARD
e) Using a combination of DMARDs

21 – Rheumatology: Conditions Related to Rheumatoid Arthritis


1) A patient presents with polyarticular inflammation. Pitting edema of the hands
(RS3PE) can be seen. The patient is given a low dose of prednisone after being diagnosed
with seronegative rheumatoid arthritis. What is the clinical age of onset for this type of
patient?
a) 20
b) 30
c) 40
d) 50
e) 60
2) Which of the following is NOT associated with juvenile rheumatoid arthritis (Still
disease)?
a) Seronegativity (RF & ANA)
b) Fever

DO NOT DISTRIBUTE - 86 -
Musculoskeletal 17Mar2009

c) Rash
d) Conjunctivitis
e) Arthritis
3.1) What is the age of onset for Adult Still disease?
a) 1-16
b) 16-35
c) 40-50
d) 50-60
e) 60-70
3.2) Which of the following is NOT a preferred joint location for Adult Still disease?
a) Wrist
b) Fingers
c) Hips
d) Shoulders
e) Knees
3.3) A college student presents with a long history of sore throat. They have made
repeated trips to the University walk-in clinic to have rapid Strep cultures done, all of
which have been negative. They present with a fever and general arthritis. Which of the
following will be present and elevated in this patient?
a) Rheumatoid factor
b) Antinuclear antibodies
c) Serum ferritin
d) Serum albumin
e) Leukocytes
4) A patient presents with leukopenia and splenomegaly. They are currently being treated
for rheumatoid arthritis. The clinician diagnoses Felty syndrome, which occurs how
many years or more after onset of rheumatoid arthritis?
a) 1
b) 2
c) 5
d) 12
e) 16
5) A patient presents several years after being diagnosed with rheumatoid arthritis. They
now have leukopenia, splenomegaly, and recurrent bacterial infections. Lab results will
show high titers of which of the following?
a) Rheumatoid factor
b) Antinuclear antibodies
c) Serum ferritin
d) Serum albumin
e) Leukocytes
6) Which of the following is NOT a classic symptom seen in Sjögren syndrome?
a) Connective tissue disease
b) Keratoconjunctivitis sicca with lacrimal gland enlargement
c) Keratoconjunctivitis sicca without lacrimal gland enlargement
d) Xerostomia with salivary gland enlargement
e) Xerostomia without salivary gland enlargement

DO NOT DISTRIBUTE - 87 -
Musculoskeletal 17Mar2009

f) Increased serum ferritin


7.1) Glandular infiltration by which of the following is seen histologically in patients
with Sjögren disease?
a) B-cells
b) CD4 lymphocytes
c) CD8 lymphocytes
d) Natural killer cells
e) Uric acid
7.2) Which of the following drugs can be used to improve salivary and lacrimal gland
function?
a) Bethanechol
b) Pilocarpine
c) Pirenzepine
d) Scopolamine
e) Atropine
8) Patients with Sjögren syndrome are at an increased risk of developing which of the
following?
a) Osteoarthritis
b) Testicular cancer
c) Renal calcifications
d) Hodgkin lymphoma
e) Non-Hodgkin lymphoma

22 – Rheumatology: Osteoarthritis
1) Which of the following is the most common rheumatic disease?
a) Rheumatoid arthritis
b) Septic arthritis
c) Osteoarthritis
d) Nonarticular rheumatism
e) Sjögren syndrome
2) A patient presents with progressive focal degeneration of articular cartilage. Which of
the following would be seen radiographically if the patient had osteoarthritis?
a) Osteophytes
b) Joint subluxation
c) Heberden nodes
d) Bouchard nodes
e) Fusion of bones
3.1) Which of the following best describes the pain associated with osteoarthritis?
a) Deep ache while resting
b) Deep ache during activity
c) Shooting pain while resting
d) Shooting pain during activity
e) “Pins and needles”
3.2) Which of the following describes the location of Heberden nodes?
a) Distal interphalangeal joints (DIPs)
b) Proximal interphalangeal joints (PIPs)

DO NOT DISTRIBUTE - 88 -
Musculoskeletal 17Mar2009

c) Carpometacarpal joints
d) Tarsometatarsal joints
e) Knees
4) Which of the following would NOT be found on physical exam of a patient with
osteoarthritis?
a) Joint margin tendeness
b) Fine crepitus
c) Bony enlargements, mild synovial thickening
d) Bony deformities, derangement of local tissues
e) Extended range of motion in affected joints (hyperflexibility)
5) Radiographic and physical examination findings can accurately predict a patient’s
osteoarthritis symptoms.
a) True
b) False
6) Which of the following affects only the distal interphalangeal joint?
a) Generalized osteoarthritis
b) Isolated nodal osteoarthritis
c) Isolated hip osteoarthritis
d) Erosive osteoarthritis
e) Diffuse idiopathic skeletal hyperostosis
7) Generalized osteroarthritis affects the DIPs, PIPs, 1st carpometacarpal joints, knees,
hips, and spine. It is most commonly seen in:
a) Men older than 50
b) Middle-aged men
c) Middle-aged pre-menopausal women
d) Middle-aged post-menopausal women
e) Children
8) Which of the following develops in many cases of erosive osteoarthritis, affecting the
DIPs and PIPs?
a) Synovitis
b) Vasculitis & claudication
c) Soft tissue necrosis
d) Osteophytes & ankylosis
e) Fusion of affected joints
9) An obese, diabetic patient presents with osteophytosis that connects four vertebrae
while preserving vertebral disk space. The patient is diagnosed with diffuse idiopathic
skeletal hyperostosis (DISH). Which of the following groups of patients is most
commonly affected?
a) Men older than 50
b) Middle-aged men
c) Middle-aged pre-menopausal women
d) Middle-aged post-menopausal women
e) Children
10) A patient presents with stiffness and swelling in the shoulder and MCP joints with
widespread chondrocalcinosis several months after a motorcycle accident. Which of the
following is the most likely?

DO NOT DISTRIBUTE - 89 -
Musculoskeletal 17Mar2009

a) Rheumatoid arthritis
b) Septic arthritis
c) Primary osteoarthritis
d) Secondary osteoarthritis
e) Gout
11) Which of the following inherited disorders is NOT complicated by osteoarthritis?
a) Ochronosis
b) Wilson disease
c) Paget disease
d) Hemochromatosis
e) Acromegaly
12) A patient presents with osteoarthritis. When the patient stands to urinate, the urine
darkens in color. Homogentistic acis is found in the urine. Examination of synovial fluid
in the hips and knees reveals calcium pyrophosphate crystals. Radiographic study shows
calcified intervertebral disks at multiple levels. Which of the following is the most likely?
a) Ochronosis
b) Wilson disease
c) Paget disease
d) Hemochromatosis
e) Acromegaly
13) a patient presents with diabetes mellitus, pituitary insufficiency, degenerative
arthritis, and bronze skin pigmentation. Arthritis is localized to the MCP joints and
shoulders. Which of the following is the most likely?
a) Ochronosis
b) Wilson disease
c) Paget disease
d) Hemochromatosis
e) Acromegaly
14) A30-year-old patient presents with unexplained hepatitis. Radiographic study shows
subchondral cysts, sclerosis, and radio-dense lesions at the affected joint margins. Serum
levels of ceruloplasmin are found to be less than 200 mg/L. Physical exam reveals a
brown Kayser-Fleiscer ring in the left eye. Which of the following is the most likely?
a) Ochronosis
b) Wilson disease
c) Paget disease
d) Hemochromatosis
e) Acromegaly
15) Which of the following patients are at risk for neuroarthropathy (Charcot joint)?
a) Epileptics
b) Hyperlipedemia patients
c) Diabetics
d) Stroke victims
e) Patients with hemochromatosis
16) Protection and treatment for infection of which of the following locations is the most
commonly seen in Charcot joint disease?
a) Hands

DO NOT DISTRIBUTE - 90 -
Musculoskeletal 17Mar2009

b) Feet
c) Knees
d) Hip
e) Spine
17) Which of the following is NOT a risk factor for aseptic necrosis of the bone
(avascular necrosis)?
a) Corticosteroid use
b) Hypertension
c) Alcoholism
d) Sickle cell disease
e) SLE
18) Which of the following is the most common location of avascular necrosis?
a) Shoulders
b) Knees
c) Ankles
d) Wrists
e) Hip
19) Which of the following is the most sensitive test for diagnosis of avascular necrosis?
a) Plain radiography (x-ray)
b) Computed tomography (CT)
c) Magnetic resonance imaging (MRI)
d) Positron emission tomography (PET)
e) Ultrasound
20) A patient presents with painful distal long bone periostitis, non-inflammatory arthritis
at the ankles, and clubbing of the fingernails. Which of the following is the most likely?
a) Diffuse idiopathic skeletal hyperostosis (DISH)
b) Wilson disease
c) Spondylolisthesis
d) Hypertrophic osteoarthropathy
e) Neuroarthropathy (Charcot joint)
21) Which of the following would NOT be seen on a radiograph of an osteoarthritic
patient?
a) Sclerosis at the PIP and DIP joints
b) Joint-space narrowing at the first carpometacarpal joints
c) Osteophyte formation at the metacarpophalangeal joints
d) Subchondral cysts at the PIPs
e) Buttressing of angle joints at the DIPs
22.1) Which of the following is caused by a defect in the bony structure of the posterior
neural arch?
a) Spondylosis
b) Spondylolysis
c) Spondylolisthesis
22.2) Which of the following has radiographic findings including anteriolateral spinous
osteophytes, degenerative disk disease with disk-space narrowing, and facet sclerosis?
a) Spondylosis
b) Spondylolysis

DO NOT DISTRIBUTE - 91 -
Musculoskeletal 17Mar2009

c) Spondylolisthesis
22.3) Which of the following occurs with bilateral bony defects causing subluxation of
one vertebrae onto the other.
a) Spondylosis
b) Spondylolysis
c) Spondylolisthesis
23) Which of the following is NOT a cause of spondylolisthesis?
a) Trauma
b) Congenital
c) Osteoarthritis
d) Avascular necrosis
24) Therapeutic goals for patients with osteoarthritis include relieving pain, preserving
joint motion/function, and preventing further injury of cartilage.
a) True
b) False, the only goal is relieving pain
25) Which of the following drugs would be given as the initial treatment for
osteoarthritis?
a) Naproxen (Aleve, NSAID)
b) Ibuprofen (NSAID)
c) Methotrexate (DMARD)
d) Acetaminophen (Tylenol, Analgesic)
e) Hydrochlorothiazid (HCTZ, Hydrodiuril, Diuretic)
26) Which of the following is NOT an indication for total join arthroplasty?
a) Radiographically advanced osteoarthritis
b) Night pain that does not change with positional adjustment
c) Adolescent anterior (patellar) knee paint
d) Lockup or giving-way of a weight bearing joints associated with falls
e) Joint symptoms compromising activities of daily living (ADLs)

23 – Rheumatology: Arthritis in Chronic Renal Failure


1) After 10 years of hemodialysis, 65% of patients will have destructive arthritis,
shoulder pain, stiffness syndrome, or carpal tunnel syndrome due to deposits of which of
the following in the joint space?
a) Urate crystals
b) Calcium pyrophosphate
c) Iron
d) Tubulin
e) Amyloid

24 – Rheumatology: Nonarticular Rheumatism


1.1) A female patient presents with headache, parasthesia, numbness, and irritable bowel
syndrome (IBS). History reveals the patient had growing pains as a child. Which of the
following evaluation techniques could be used to help diagnose fibromyalgia?
a) Bilateral muscle weakness
b) Unilateral muscle hypertrophy
c) Tender points

DO NOT DISTRIBUTE - 92 -
Musculoskeletal 17Mar2009

d) Abnormal gait
e) Acupuncture
1.2) What lab test is used to confirm a diagnosis of fibromyalgia?
a) Complete blood count (CBC)
b) Sedimentation rate (ESR)
c) Thyroid function
d) Liver transaminase values
e) No lab test can confirm a diagnosis
2) Which of the following is NOT a finding needed for performing a more extensive
examination or possibly surgical decompression in a patient with acute low back pain?
a) Objective leg weakness
b) Difficulty sitting up
c) Bladder dysfunction
d) Bowel dysfunction
3) Which of the following symptoms suggest a neoplastic or infectious process?
a) Substantial weight loss or pain that interferes with sleep
b) Fever with heart palpitations
c) Claudication during activities of daily living (ADL) and malaise
d) Nausea and vomiting or diarrhea
e) Severe anxiety or increased alertness at night
4) Which of the following is NOT a symptoms suggesting irritation of a spinal nerve
root?
a) Radiating pain
b) Weakness
c) Numbness
d) Fasciculations
5) A patient presents with calf and thigh cramping during exertion. Symptoms improve
when the patient leans forward on a shopping card while walking. Which of the following
would be diagnostic for pseudoclaudication (spinal stenosis)?
a) Abnormal peripheral pulse
b) Normal peripheral pulse
c) Numbness in extremity
d) Tingling in extremity
e) Radiating pain in extremity
6) Indications for spinal radiographs in patients with acute low back pain include fever,
weight loss, severe pain unrelieved in any position, neurologic symptoms, and no
previous episodes if older than age 50. Which of the following would NOT be an
indication regarding a patient’s history?
a) History of back disease
b) History of back surgery
c) History of claudication
d) History of neoplasm
e) History of acute direct trauma to the back
7) What is the initial treatment of acute nonspecific low back pain because it works 90%
of the time to give considerable improvements in six weeks?
a) Reassuring the patient

DO NOT DISTRIBUTE - 93 -
Musculoskeletal 17Mar2009

b) Long term modification of activities of daily living


c) Bed rest for more than three days
d) Long term use of narcotic analgesics or NSAIDS
e) Physical therapy with local heat and ice
8) Which of the following is NOT a major cause of bursitis?
a) Microcrystalline disease
b) Chronic inflammatory arthritis
c) Infection
d) Trauma or overuse
e) Charcot joint
9) Which of the following types of bacteria are commonly found in septic arthritis?
a) Staph & Strep (Gram-positive cocci)
b) E. coli & Salmonella (Gram-negative rods)
c) Neisseria (Gram-negative cocci)
d) Shigella & Yersinia (Gram-negative, H2S negative)
e) Entero species
10) Which of the following is used to treat severe infections of septic bursitis?
a) Aspiration
b) Surgery
c) Antibiotics
d) NSAIDs
e) Analgesics
11) A 60-year-old female presents with new onset aching and morning stiffness in the
proximal musculature. They have problems getting comfortable at night and have mild
constitutional symptoms including sweats, fever, anorexia, and weight loss. Examination
reveals increased ESR (sed rate). Rheumatoid arthritis, chronic infection, and
inflammatory myositis have been ruled out. Which of the following should be used to
treat this patient?
a) Prednisone 40-60 mg daily
b) Prednisone 10-15 mg daily
c) Ibuprofen 100 mg daily
d) Ibuprofen 800 mg daily
e) Naproxen 200 mg daily
12) What disease is present in 15% of patients that are diagnosed with polymyalgia
rheumatic?
a) Wegener granulomatosis
b) Giant cell arteritis
c) Systemic lupus erythmatosus
d) Diabetes
e) Wilson disease

25 – Rheumatology: Vasculitic Syndromes


1) A patient presents with temporal headache, fever, fatigue, diplopia, and jaw
claudication. Ultrasound of the temporal artery reveals a halo sign. If this patient is not
treated within four hours, which of the following can happen?
a) Aneurism

DO NOT DISTRIBUTE - 94 -
Musculoskeletal 17Mar2009

b) Locked jaw
c) Unilateral blindness
d) Unilateral facial drooping
e) Bilateral loss of olfactory sense
2) What test is used to diagnose giant cell (temporal) arteritis?
a) Orthostatic blood pressure
b) Rheumatoid factor and anti-CCP
c) Increased alkaline phosphatase
d) Artery biopsy
e) Increased ESR (sed rate)
3) Which of the following will most likely be markedly increased in giant cell arteritis?
a) ESR
b) RBCs
c) Thombocytes
d) Alkaline phosphatase
e) Creatinine & BUN
4) Which of the following is NOT a criteria for diagnosing temporal arteritis?
a) Tender, swollen temporal artery
b) Jaw claudication
c) Blindness
d) Slow response to corticosteroids
e) Polymyalgia rheumatic symptoms
5) Which of the following would be used to treat a patient when temporal arteritis is
suspected?
a) Prednisone 40-60 mg daily after biopsy confirmation
b) Prednisone 10-15 mg daily after biopsy confirmation
c) Prednisone 40-60 mg daily then schedule a biopsy
d) Prednisone 10-15 mg daily then schedule a biopsy
e) Ibuprofen 800 mg daily without biopsy
6) A 20-year-old Latin American female presents with fever, weight loss, fatigue,
arthralgia, increased ESR, and vascular bruits. The patient is diagnosed with Takayasu
arteritis, which has a 90% survival at 10 years. Which of the following is used to treat
this disorder and is usually adequate alone?
a) Methotrexate
b) Cyclophosmphamide
c) Corticosteroids
d) Surgery
e) Gold
7) Systemic vasculitis usually presents with fever, fatigue, weight loss, occasional
myalgia/arthraligia with multisystem organ involvement.
a) True
b) False, only a single system is involved
8) Which of the following is true of polyarteritis nodosa (PAN) and not microscopic
polyangiitis (MPA)?
a) Positive myeloperoxidase specific p-ANCA or c-ANCA
b) Abnormal renal angiograms

DO NOT DISTRIBUTE - 95 -
Musculoskeletal 17Mar2009

c) Lung involvement
d) Proteinuria without hypertension
e) Can progress to necrotizing glomerulonephritis
9) Which of the following diseases is NOT associated with secondary polyarteritis?
a) Hairy cell leukemia
b) Hepatitis C
c) Rheumatoid arthritis & Sjögren syndrome
d) Mixed cryoglobulinemia
e) HIV & AIDS
10) Which of the following tests, along with angiography, is used to diagnose vasculitis?
a) Orthostatic blood pressure
b) Rheumatoid factor and anti-CCP
c) Increased alkaline phosphatase
d) Tissue biopsy
e) Increased ESR (sed rate)
11) Which of the following diseases will most likely show a positive p-ANCA test?
a) PAN
b) MPA
c) SLE
d) DISH
e) Rheumatoid arthritis
12) Which of the following is the cornerstone of treatment for PAN and MPA?
a) Cyclophosphamide
b) Chlorambucil
c) Corticosteroids
d) Methotrexate
e) Azathioprine
Match the stages of Churg-Strauss vasculitis with the description:
13.1) History of asthma, allergic rhinitis a) First
13.2) Peripheral blood eosinophilia (Lofflers syndrome) b) Second
13.3) Life-threatening vasculitis c) Third
14) A 25-year-old presents with instep claudication and loss of their left fifth finger due
to ischemic injury. The patient has been smoking cigarettes since age 16. Which of the
following is the most likely?
a) Diabetes
b) Buerger disease
c) Wilson disease
d) Rheumatoid arthritis
e) Felty syndrome
15) A 45-year-old patient presents with headache, nausea, and vomitting. Neuological
exam reveals some minor confusion as well as drowsiness. The patient report having had
seizures recently. Lab results for CSF are abnormal. Which of the following is the most
likely?
a) Stroke
b) Temporal arteritis
c) Mixed cryoglobulinemia

DO NOT DISTRIBUTE - 96 -
Musculoskeletal 17Mar2009

d) Schonlein-Henoch vasculitis
e) Isolated (primary) angiitis of the CNS
f) Buerger disease (thromboangiitis obliterans)
16.1) Wegener granulomatosis has a 95% mortality rate within a year if not treated.
Which of the following clinical areas is NOT usually affected?
a) Ears, nose, throat
b) Heart
c) Lungs
d) Kidney
e) Skin
16.2) What test is positive in more than 90% of patients with Wegener granulomatosis?
a) ANA
b) p-ANCA
c) c-ANCA
d) HBsAg
e) PPD
17) Which of the following would NOT likely have a positive p-ANCA test?
a) Churg-Strauss syndrome
b) Wegener granulomatosis
c) Ulcerative colitis
d) Wilson disease
e) Giant-cell arteritis
18) What is the most common cause of small vessel (cutaneous) vasculitis, which often
presents with palpable purpura?
a) SLE
b) Rheuatoid arthritis
c) Infection
d) Liver disease
e) Temporal arteritis
19) A patient presents with abdominal pain, lower extremity purpura, arthritis, and
nephritis (hematuria). The patient is diagnosed with Schonlein-Henoch vasculitis after
normal complement levels are found and deposition of which of the following
immunoglobulins is found in vessel walls?
a) IgA
b) IgD
c) IgE
d) IgG
e) IgM
20) Which of the following is NOT associated with Type I cryoglobulins?
a) Multiple myeloma
b) Lymphoma
c) Waldenstrom macroglobulinemia
d) Wegener granulomatosis
21) Which of the following is NOT a common symptoms associated with Type I
cryoglobulins?
a) Headache

DO NOT DISTRIBUTE - 97 -
Musculoskeletal 17Mar2009

b) Visual disturbances
c) Urethritis
d) Epistaxis (nosebleeds)
22) Which of the following diseases would NOT be associated with Type II (mixed)
cryoglobulinemia, showing anti-immunoglobulin specificity (rheumatoid factor)?
a) Bacterial infection
b) Hepatitis C
c) Autoimmune disorders
d) Lymphoma
23) A patient presents with Hepatitis C and Type II cryoglobulinemia. Which of the
following would NOT likely be seen?
a) Increased ESR
b) Increased immunoglobulin levels
c) Negative rheumatoid factor
d) Low levels of complement

26 – Rheumatology: Antirheumatic Drug Therapies


1) Aspirin, a COX-1 blocker, should be discontinued at least how many days prior to
surgery as that is the half-life of platelets?
a) 1
b) 2
c) 5
d) 10
e) 21
2) Most NSAIDs bind irreversibly to COX-1 and therefore should be discontinued how
many drug half-lives prior to invasive procedures in which bleeding is a concern?
a) 1
b) 2
c) 3
d) 4
e) 5
3) Which of the following groups of patients is at the greatest risk for significant GI
toxicity related to NSAIDs?
a) Alcoholics
b) Tobacco users
c) Corticosteroid users
d) Elderly
e) Infants
4) What is the major toxic effect (organ involved) associated with hydroxychloroquine,
which requires the patient to get exams every 6 months to a year to prevent?
a) Heart
b) Eyes
c) Ears
d) Throat
e) Liver

DO NOT DISTRIBUTE - 98 -
Musculoskeletal 17Mar2009

5) Hydroxychloroquine, which is used for rheumatoid arthritis and SLE, usually doesn’t
show clinical response before how many weeks, with improvement taking up to 6
months?
a) 1
b) 2
c) 4
d) 6
e) 8
6) Which of the following would NOT be treated with sulfasalazine (similar to gold
therapy but with fewer side effects)?
a) Rheumatoid arthritis
b) Inflammatory polyarthritis
c) Seronegative spondyloarthropathies
d) Septic arthritis
e) Psoriatic arthritis
7) Methotrexate is most often used in which of the following cases?
a) Rheumatoid arthritis
b) Inflammatory polyarthritis
c) Seronegative spondyloarthropathies
d) SLE
e) Psoriatic arthritis
8) Which of the following DMARDs has a high efficacy and tolerability, giving anti-
rheumatic effects within 4-6 weeks and 35% still taking the drug after 5 years?
a) Chlorambucil
b) Cyclosporine
c) Methotrexate
d) Gold
e) Azathioprine
9) Which of the following groups of patients would NOT be at risk when taking
methotrexate (MTX)?
a) Patient taking TMP-SMX (Bactrim)
b) Patients with gastric ulcers
c) Patients with significant heart disease
d) Patients with significant liver disease
e) Patients with significant renal disease
10) Which fo the following is NOT a common side effect of methotrexate (MTX)?
a) Esophageal reflux
b) Diarrhea
c) Stomatitis
d) Nausea
e) Vomitting
11) Which of the following groups of patients should NOT be given methotrexate?
a) Newborns
b) Pregnant
c) Elderly
d) Latin Americans with SLE

DO NOT DISTRIBUTE - 99 -
Musculoskeletal 17Mar2009

e) Patients with rheumatoid arthritis


12) Leflunomide, a teratogen used for rheumatoid arthritis, show effects after how long?
a) 1 week
b) 2 weeks
c) 6 weeks
d) 12 weeks
e) 24 weeks
13) Azathiopine, which requires a thiopurine methyltransferase test to predice severe
hematologic reactions, can become cytotoxic (bone marrow toxicity) when combined
with what drug?
a) Colchicine
b) NSAIDs
c) Uricosuric agents
d) Allopurinol
e) Febuxostat
14) Azathioprine, which can cause GI side-effects and cytopenia, is absolutely
contraindicated for patients with:
a) Diabetes
b) Pregnancy
c) Pancreatitis
d) Rheumatoid arthritis
e) Osteoarthritis
15) Patients taking azathioprine who have undergone an organ transplant are at increased
risk for which of the following?
a) Infertility
b) Bacterial infections
c) Neoplasms
d) Cirrhosis
e) Peptic ulcers
16) Cyclophosphamide is used for all of the following, but is the drug of choice for:
a) SLE
b) Rheumatoid arthritis
c) Glomerulonephritis
d) Systemic necrotizing vasculitis
e) Wegener granulomatosis
17) Cyclophosphamide has an increased risk of leukopenia when taken with allopurinol.
In the short term, pulse IV dosing (versus oral) of cyclophosphamide reduces toxic effect
on which of the following?
a) Gall bladder
b) Pancreatic duct
c) Bladder
d) Knee ligaments
e) Esophagus
18) What infection is commonly associated with oral cyclophosphamide use?
a) Staph & strep (Gram-positive)
b) Gram-negative bacteria

DO NOT DISTRIBUTE - 100 -


Musculoskeletal 17Mar2009

c) Epstein-Barr virus
d) Herpes Zoster virus
e) Cytomegalovirus
19) What fertility affects does cyclophosphamide have?
a) Premature ovarian failure
b) Atrophy of seminiferous tubules
c) Teratogen
d) A & C
e) B & C
f) A, B, & C
20) Urinalysis and urine cytology must be done at regular intervals for life in patients
who have had therapy for which of the following?
a) Cyclophosphamide
b) Chlorambucil
c) Glucocorticoids
d) Methotrexate
e) Azathioprine
21) Which of the following can be taken in low doses (< 10mg/day) for day-to-day
management of rheumatoid arthritis but should be taken in high dose for life-threatening
or serious inflammatory disease?
a) Cyclophosphamide
b) Chlorambucil
c) Glucocorticoids
d) Methotrexate
e) Azathioprine
22) Prednisone of more then 30 mg/day is associated with a higher risk of infection, such
as Pneumocystis carinii, especially when given with all of the following EXCEPT:
a) Methotrexate
b) Cyclophosphamide
c) Azathioprine
d) TMP-SMX
23) Which of the following is NOT an unwanted side effect of glucocorticoids?
a) Weight gain
b) Water retention
c) Increased appetite
d) Increased bone mass
e) Hirsutism (hairiness)
24) Which of the following would NOT be used to treat steroid-induced osteoporosis?
a) Calcium
b) Vitamin C
c) Vitamin D
d) Bisphosphonates
25) Due to the primary risk to patients starting anti-cytokine therapy, which of the
following test must be done before commencing therapy?
a) ANA
b) p-ANCA

DO NOT DISTRIBUTE - 101 -


Musculoskeletal 17Mar2009

c) Rheumatoid factor
d) ESR
e) PPD

27 – Rheumatology: Crystalline Arthropathies


1) Which of the following is NOT associated with hyperuricemia?
a) Hypertension
b) Obesity
c) Smoking
d) Renal insufficiency
e) Arteriosclerotic heart disease
2) Which of the following groups of patients get hyperuricemia due to underexcretion of
uric acid?
a) SLE
b) Rheumatoid arthritis
c) Diabetic
d) Obese
e) Gout
3) What percentage of patients with gout have overproduction of uric acid?
a) 10%
b) 25%
c) 50%
d) 75%
e) 90%
4) Lesche-Nyhan syndrome, which is a complete deficiency of hypoxanthine-guanine
phosphoribosyltransferase (HGPT), is characterized by all of the following EXCEPT:
a) Hyperuricemia
b) Self-mutilation
c) Choreoathetosis
d) Glaucoma
e) Xanthinuria
5) What is the inheritance pattern of xanthine oxidase deficiency, which causes
hypouricemia and xanthine stones?
a) X-linked dominant
b) X-linked recessive
c) Autosomal dominant
d) Autosomal recessive
e) Autosomal dominant with decreased penetrance
6) All of the following are associated with hypouricemia seen in Fanconi syndrome
EXCEPT:
a) Wilson disease
b) Malignant neoplasms
c) Use of outdated tetracycline
d) Kawasaki disease
e) High dose aspirin or probenecid
f) High dose radiographic contrast agents

DO NOT DISTRIBUTE - 102 -


Musculoskeletal 17Mar2009

7) Which of the following is most associated with hypouricemia associated with an


increase in excretion of uric acid?
a) Use of allopurinol
b) Use of methotrexate
c) Hodkin lymphoma
d) Fanconi syndrome
e) Gout
8) Which of the following is most associated with hypouricemia associated with a
decrease in uric acid synthesis?
a) Use of allopurinol
b) Use of methotrexate
c) Hodkin lymphoma
d) Fanconi syndrome
e) Gout
9) Which of the following would NOT predispose a patient to an attack of gout or
pseudogout?
a) Trauma or infection
b) Operation (3 days after)
c) Major medical illness
d) Alcohol use
e) Overeating
10) Which of the following types of drugs can induce secondary hyperuricemia?
a) Anesthetics
b) Anti-psychotics
c) DMARDs
d) Diuretics
e) Anti-inflammatory
11) A patient presents with joint pain. Aspiration reveals strongly negatively birefringent,
needle-shaped crystals. Which of the following is the most likely joint that the aspiration
took place?
a) Third distal interphalangeal
b) First metatarsophalangeal
c) Fifth proximal interphalangeal
d) Knee
e) Shoulder
12) Indomethacin and other NSAIDs are used for acute gouty arthritis. Along with
congestive heart failure, who should NOT receive these drugs?
a) Chronic obstructive pulmonary disease
b) Active peptic ulcer disease
c) Hepatic insufficiency
d) Epilepsy
e) Diabetes
13) Which of the following should be used to treat acute gouty arthritis in patients
contraindicated to colchines and NSAIDs?
a) Methotrexate
b) Cyclophosmphamide

DO NOT DISTRIBUTE - 103 -


Musculoskeletal 17Mar2009

c) Corticosteroids
d) Surgery
e) Gold
14) Oral colchicines is associated with which of the following side- effects?
a) Renal insufficiency
b) Bone marrow depression
c) Diarrhea
d) Sepsis
15) Oral colchicines is given prophylactically with probenecid to prevent exacerbations
of acute gout. Which of the following patients should not get probenecid?
a) Patients with diabetes
b) Patients with kidney stones
c) Patients who do not eat meat
d) Anemic patients
e) Patients with asthma
16) Which drug should be used instead of probenecid for lowing uric acid levels in
patients with tophaceous gout or gout complicated by renal insufficiency?
a) Colchicine
b) Dantrolene
c) Uricosuric agents
d) Allopurinol
e) Febuxostat
17) What prophylactic therapy should patients with acute tumor lysis syndrome get prior
to chemotherapy?
a) Adequate hydration and urine alkalinization
b) Adequate hydration and urine acidification
c) Adequate rest (at least 8 hours)
d) Adequate exercise to prevent blood clots
e) Antibiotics to prevent sepsis
18) Which of the following is NOT a disease associated with calcium pyrophosphate
disease (CPPD, pseudogout)?
a) Wilson diesease
b) Hemochromatosis
c) Hyperparathyroidism
d) Gout
e) Osteoporosis
19) A patient presents with joint pain. Aspiration reveals strongly positively birefringent,
rhomboid-shaped crystals. Which of the following is the least likely joint that the
aspiration took place?
a) First metatarsophalangeal
b) Wrist
c) Elbow
d) Ankle
e) Intervertebral disk
20) Which of the following is NOT used for acute pseudogout?
a) NSAIDs

DO NOT DISTRIBUTE - 104 -


Musculoskeletal 17Mar2009

b) Steroid injections
c) Allopurinol
d) IV colchicine
21) A patient presents with sudden onset of pain in the right shoulder with marked loss of
shoulder motion. A paste-like material is aspirated from the shoulder. Alizarin red stain is
positive with crystals being seen under transmission electron microscopy. Which of the
following is the most likely?
a) Gout
b) Pseudogout
c) Calcium oxalate arthropathy
d) Hydroxyapatite deposition disease
e) Cholesterol crystal disease

28 – Rheumatology: Spondyloarthropathies
1) Spondyloarthropathies are characterized by involvement in which joint, that is not
commonly seen in rheumatoid arthritis?
a) Distal interphalangeal
b) Proximal interphalangeal
c) Sternoclavicular
d) Carpometacarpal
e) Sacroiliac
2) Which of the following is most likely to have HLA-B27?
a) Ankylosing spondylitis
b) Reactive arthritis
c) Enteropathic spondylitis
d) Psoriatic spondylitis
3) A patient presents with low back pain and morning stiffness that improves with
exercise. Radiograph shows sacroiliac joint involvement and a “bamboo spine.” Which of
the following is the most likely?
a) Ankylosing spondylitis
b) Reactive arthritis
c) Enteropathic spondylitis
d) Psoriatic spondylitis
e) Rheumatoid arthritis
4) Which of the following is NOT associated with ankylosing spondylitis?
a) Plantar fasciitis
b) Achilles tendonitis
c) Hip & shoulder involvement
d) Cervical subluxation
e) Cord compression
5) Which of the following is NOT a common treatment for akylosing spondylitis?
a) Physical therapy
b) Barbiturates
c) Postural changes
d) Exercise
e) NSAIDs

DO NOT DISTRIBUTE - 105 -


Musculoskeletal 17Mar2009

6) A patient presents with a “sausage” toe, inflammatory eye disease, and keratoderma
blennorrhagicum (palms/soles). Lab results are positive for Salmonella species and
Shigella flexneri. Which of the following is the most likely?
a) Reactive arthritis
b) Myasthenia gravis
c) Systemic lupus erythematosus
d) Ankylosing spondylitis
e) Psoriatic arthritis
7) A patient presents with psoriasis, pitting of the nails, and “sausage” fingers with distal
IP joint erosion (“pencil-in-a-cup” deformity) seen on radiograph. Which of the following
is the most likely?
a) Reactive arthritis
b) Bechet syndrome
c) Systemic lupus erythematosus
d) Ankylosing spondylitis
e) Psoriatic arthritis

29 – Rheumatology: Beçhet Syndrome


1) What disease is associated with patients from Japan or the Middle East, has HLA-B51,
uveitis, and oral/genital ulcerations?
a) Reactive arthritis
b) Beçhet syndrome
c) Systemic lupus erythematosus
d) Ankylosing spondylitis
e) Psoriatic arthritis

30 – Rheumatology: Osteoid Osteoma


1) A 15-year-old patient presents with arm pain at night that is completely relieved with
the use of Aspirin. Radiograph is initially negative but a bone scan helps localize a
benign tumor. What is the definitive treatment for this patient?
a) Radiation therapy
b) NSAIDs
c) Chemotherapy
d) Narcotics
e) Surgical excision

31 – Rheumatology: Systemic Lupus Erythematosus (SLE)


1) Which of the following is NOT one of the criteria for diagnosis SLE?
a) Malar rash
b) Photosensitivity
c) Genital ulcers
d) Oral ulcers
e) Discoid lupus
2) Which of the following people is the most likely to have SLE?
a) 20-year old Caucasian male
b) 20-year-old African American woman

DO NOT DISTRIBUTE - 106 -


Musculoskeletal 17Mar2009

c) 50-year-old Caucasian woman


d) 80-year-old Asian man
e) 5-year-old Native American boy
3) Which of the following human histocompatibility complexes is NOT seen in increased
frequency in patients with SLE?
a) HLA-B8
b) HLA-B27
c) HLA-DR2
d) HLA-DR3
4) Lupus (SLE) symptoms that affect which of the following is the most variable and
unpredictable?
a) Constitution
b) Joints
c) Heart
d) Lungs
e) Brain
5) Which of the following has increased protein (IgG) in the CSF from lumbar puncture
and areas of increased signal in the periventricular white matter found on MRI?
a) Primary Sjögren syndrome
b) SLE
c) Hepatitis C
d) Systemic vasculitis
e) Amyloid disease
6) Which of the following is likely to occur in pregnant women with SLE?
a) Deformed babies
b) Spontaneous abortion
c) Extreme hypotension
d) Extreme hypertension
e) No adverse effects are seen
7) Which of the following lab findings in SLE usually corresponds to disease activity?
a) Idiopathic thrombocytopenic purpura (ITP)
b) Erythrocyte sedimentation rate (ESR)
c) Hypocomplimentemia with increased anti-nDNA antibodies
d) Compliment split products (C3a, C5a)
e) Hemolytic anemia (Coombs positive)
8) Patients with SLE may have false positive results for which of the following?
a) Gonorrhea culture (GC)
b) Chlamydia test
c) Syphilis test (VDRL)
d) HIV test
e) Herpes test (TORCH)
9) Which of the following test is positive in all patients with SLE?
a) ANA
b) p-ANCA
c) Rheumatoid factor
d) ESR

DO NOT DISTRIBUTE - 107 -


Musculoskeletal 17Mar2009

e) PPD
10) Which of the following patients would have the worst prognosis in SLE?
a) African American patient with creatinine value of 1.0 mg/dL
b) Hispanic patient with creatinine value of 4.0 mg/dL
c) Caucasian patient with creatinine value of 4.0 mg/dL
d) Asian patient with creatinine value of 2.0 mg/dL
e) Polish patient with creatinine value of 3.0 mg/dL

32 – Rheumatology: Drug-Induced Lupus


1) What two drugs are strongly implicated in drug-induced lupus?
a) Halothane & succinylcholine
b) Allopurinol & febuxostat
c) Procainamide & hydralazine
d) TMP-SMX & penicillin
e) Colchicine & Azithromycin
2) Which of the following is the most common symptom(s) associated with drug-induced
lupus?
a) Arthralgias & polyarthritis
b) Malaise & fever
c) Pericarditis
d) Pleural-pulmonary symptoms

33 – Rheumatology: Antiphospholipid Syndrome


1) Which of the following is the identifying test for anti-phospholipid syndrome?
a) Normal prothrombin time (PT)
b) Prolonged prothrombin time (PT)
c) Prolonged plasma clot time
d) Prolonged partial thromboplastin time (PTT)
e) Partial thromboplastin time not corrected by adding normal plasma
2) Which of the following is NOT seen in anti-phospholipid syndrome?
a) Stroke
b) MI
c) Transient ischemic attacks
d) Brachial artery thrombosis
e) Pulmonary hypotension
3) What international normalized ratio (INR) value is aimed for when treating anti-
phospholipid syndrome?
a) 1
b) 2
c) 2.5
d) 3
e) 3.5

34 – Rheumatology: Raynaud Phenomenon


1) What is the precipitating agent in Raynaud phenomenon?
a) NSAIDs

DO NOT DISTRIBUTE - 108 -


Musculoskeletal 17Mar2009

b) Cold temperature
c) Increase salt intake
d) Hypertension
e) Water (swimming)
2) Which of the following is used to treat Raynaud phenomenon?
a) Quitting smoking
b) Cold packs on the hands
c) Parasympathetic surgery
d) NSAIDs
e) Narcotics
3) Which of the following is seen in primary Raynaud and not in secondary Raynaud
(associated with connective tissue disease)?
a) Begins by affecting a single digit
b) Has finger edema and digital ulcerations
c) Affects only females at menarche
d) Is infrequent (0-5 per day)
e) Is not precipitated by emotional stress

35 – Rheumatology: Systemic Sclerosis (Scleroderma)


1) Which of the following is NOT a diagnostic criteria for scleroderma?
a) Symmetric indurations of the skin of the fingers
b) Softening of the metacarpophalangeal joints
c) Sclerodactyl
d) Digital pitting scars
e) Bibasilar pulmonary fibrosis
2) Scleroderma primarily affects the skin proximal to the metacarpophalangeal or
metatarsophalangeal joints.
a) True
b) False
3) What is the most frequent GI abnormality seen in patients with scleroderma?
a) Gastric ulcers
b) Duodenal ulcers
c) Small bowel hypomotility
d) Bowel dilation
e) Esophageal dysfunction
4) A patient with scleroderma presents with proteinuria and mild hypertension. Which of
the following drugs can extend life expectancy (help prevent over renal crisis)?
a) Beta-blockers
b) Calcium-channel blockers
c) Antiplatelet drugs
d) ACE-inhibitors
e) Statins
5) Anti-topoisomerase I antibody is found in 25% of patients with:
a) Rheumatic fever
b) Rheumatoid arthritis
c) Scleroderma

DO NOT DISTRIBUTE - 109 -


Musculoskeletal 17Mar2009

d) Osteochondromas
e) Ankylosing spondylitis

36 – Rheumatology: CREST Syndrome


1) Which of the following is NOT seen in CREST syndrome?
a) Calcinosis cutis
b) Rheumatoid arthritis
c) Esophageal dysmotility
d) Sclerodactyl
e) Telangiectasias
2) Patients with CREST syndrome are at risk for developing pulmonary hypertension.
What lab finding is present in 70-90% of patients with CREST syndrome?
a) Anti-RNP
b) Anti-ds-DNA
c) Anti-Sm
d) Anti-centromere
e) Anti-topoisomerase I

37 – Rheumatology: Scleroderma-Like Syndromes


1) A jackhammer operator presents with peau d’orange skin, carpel tunnel syndrome, and
fever after strenuous activity. Which of the following should be given to this patient, who
is diagnosed with eosinophilic fasciitis?
a) Prednisone 40 mg daily
b) Prednisone 10 mg daily
c) Ibuprofen 100 mg daily
d) Ibuprofen 800 mg daily
e) Naproxen 200 mg daily

38 – Rheumatology: Inflammatory Myopathies


1) Which of the following is the most characteristic symptom of polymyositis?
a) Heliotrope rase of eyelids
b) Gottron papules on MTC joints
c) Gottron papules on PIP joints
d) Proximal muscle weakness
e) Distal muscle weakness
2) Which of the following is characteristic of an electromyogram (EMG) seen in
inflammatory myopathies?
a) Increased amplitude and increased spike frequency
b) Increased amplitude and decreased spike frequency
c) Decreased amplitude and increased spike frequency
d) Decreased amplitude and decreased spike frequency
3) Which of the following is NOT characteristic of the muscle biopsy in inflammatory
myopathies?
a) Necrosis
b) Myofibril regeneration
c) Lymphatic infiltrate

DO NOT DISTRIBUTE - 110 -


Musculoskeletal 17Mar2009

d) Monocyte infiltrate
e) Basophil infiltrate
4) What is the treatment for polymyositis?
a) Prednisone 60 mg daily
b) Prednisone 15 mg daily
c) Ibuprofen 50 mg daily
d) Ibuprofen 800 mg daily
e) Surgical excision

39 – Rheumatology: Drug-Induced Myopathies


1) In patients with colchicines polymyositis, which of the following lab values will be
increased?
a) Fibrin degradation products (FDP)
b) Creatinine
c) Aspartate aminotransferase (AST)
d) Lactate dehydrogenase (LDH)
e) Bilirubin
2) Which of the following drugs
a) Colchicine
b) Corticosteroids
c) Lipid-lowering drugs
d) Cocaine
e) Heroine
f) Alcohol

40 – Rheumatology: Infectious Arthritis


1) Which of the following patients is NOT at risk for non-gonococcal bacterial arthritis?
a) Immunosupressed
b) Cancer patients
c) Congestive heart failure patients
d) Diabetics
e) Sickel cell anemia patients
2) Patients with chronic inflammatory and degenerative arthritis are at increased risk for
what medically emergent disease?
a) SLE
b) Rheumatoid arthritis
c) Septic arthritis
d) Vasculitis
e) Tenosynovitis
3) Which of the following is diagnostic for septic arthritis (common bacteria seen)?
a) Gram stain testing
b) Indole testing
c) H2S testing
d) Glucose fermentation testing
e) Catalase testing
Match the pathogen with the common scenario seen for bacterial arthritis:

DO NOT DISTRIBUTE - 111 -


Musculoskeletal 17Mar2009

4.1) Sickle cell anemia a) Anaerobes


4.2) Cat or dog bites b) Pseudomonas
4.3) Human bites c) Salmonella
5) IV drug users tend to get septic arthritis in the sternoclavicular or sacroiliac joint. What
bacteria is commonly involved?
a) Staphylococcus
b) Streptococcus
c) E. coli
d) Pseudomonas
e) Salmonella
6) Which of the following would NOT be a location to get suspected gonococcal arthritis
from for culture?
a) Rectum
b) Blood
c) Joints
d) Genitourinary tract
e) Gastroduodenal junction
7) What is the most common form of septic arthritis in younger, sexually active persons,
presenting with fever, dermatitis, and inflammatory tenosynovitis?
a) Disseminated gonococcal arthritis
b) Non-gonococcal arthritis
c) Rheumatoid arthritis
d) Osteoarthritis
e) Nonarticular rheumatism
8) Which of the following would be used to treat gonococcal arthritis?
a) Penicillin
b) Ceftriaxone
c) Metronidazole
d) Azithromycin
e) Acyclovir
9) Diagnosis of infected joint prostheses is easy as all patients present with fever, lytic
changes around the joint, and increased ESR.
a) True
b) False, detection may be difficult
10) Which of the following is the most likely bacteria to be seen a few weeks after an
implanted prosthesis procedure?
a) Staphylococcus aureus
b) Staphylococcus epidermidis
c) Staphylococcus saprophyticus
d) Viridans Streptococci
e) E. Coli

41 – Rheumatology: Lyme Disease


1) Lyme disease is NOT endemic to which to the following?
a) Minnesota
b) Pennsylvania

DO NOT DISTRIBUTE - 112 -


Musculoskeletal 17Mar2009

c) California
d) Massachusetts
e) Florida
2) Which of the following Lyme disease symptoms is associated with the first stage of
disease?
a) Disseminated infection
b) Cardiac abnormalities (e.g. heart block)
c) Erythema chronicum migrans
d) Chronic arthritis
e) Obliterative endarteritis
3) What test is used to confirm a positive ELISA test for Lyme disease?
a) PCR
b) Northern blot
c) Southern blot
d) Western blot
4) Which of the following is used to treat Lyme disease?
a) Gentamicin
b) Probenecid
c) Chloramphenicol
d) Erythromycin
e) Tetracycline

42 – Rheumatology: Post-Streptococcal Reactive Arthritis


1) Post-streptococcal reactive arthritis (PSRA) occurs in 2/3 of patients with rheumatic
fever. It affects large joints and can lead to “Jaccoud deformity” (ulnar deviation). Which
of the following is used to treat this form of arthritis?
a) Prednisone, high dose
b) Ibuprofen, high dose
c) Aspirin, high dose
d) Oxycontin, high dose
e) Naproxen, high dose

James Lamberg

DO NOT DISTRIBUTE - 113 -


Musculoskeletal 17Mar2009

AnswerKey 11.2) C 4.2) B 13.1) B 1.3) B


MSK #1 11.3) B 4.3) A 13.2) D 2.1) A
1.1) E 11.4) A 4.4) B 14.1) B 2.2) B
1.2) B 11.5) C 4.5) C 14.2) E 3.1) D
2.1) B 11.6) D 4.6) A 3.2) C
2.2) A 11.7) F 4.7) D MSK #5 3.3) D
2.3) C 5) C 1.1) C 3.4) A
3.1) C MSK #2 6.1) C 1.2) D 3.5) C
3.2) A 1.1) E 6.2) B 1.3) D 3.6) B
3.3) B 1.2) B 7.1) B 1.4) A 3.7) E
4.1) D 1.3) C 7.2) C 1.5) D 3.8) E
4.2) E 1.4) A 7.3) C 1.6) A 3.9) A
4.3) C 1.5) C 7.4) E 1.7) E 3.10) C
4.4) C 1.6) B 7.5) D 1.8) B 3.11) E
4.5) D 2.1) C 7.6) A 1.9) D 3.12) D
4.6) F 2.2) B 7.7) C 1.10) B 3.13) E
4.7) D 2.3) B 1.11) E 3.14) E
5.1) D 3.1) C MSK #4 1.12) D 3.15) C
5.2) E 3.2) D 1) D 1.13) C 3.16) B
5.3) A 3.3) C 2) E 1.14) C 4.1) D
6.1) F 3.4) B 3) C 1.15) E 4.2) D
6.2) B 4) E 4) A 1.16) C 4.3) C
6.3) E 5.1) A 5.1) D 1.17) C 4.4) C
6.4) A 5.2) E 5.2) C 1.18) C 4.5) B
6.5) C 6.1) B 6) E 1.19) B 4.6) D
6.6) D 6.2) C 7.1) A 1.20) E 4.7) D
7.1) D 6.3) A 7.2) C 1.21) B 4.8) B
7.2) C 8.1) C 1.22) A 4.9) A
7.3) D MSK #3 8.2) D 1.23) D 4.10) E
7.4) C 1.1) D 8.3) D 1.24) A 4.11) C
7.5) E 1.2) A 9.1) A 1.25) B 5.1) B
7.6) A 1.3) E 9.2) C 1.26) D 5.2) A
7.7) F 1.4) C 9.3) A 1.27) D 5.3) C
7.8) B 1.5) B 9.4) E 1.28) E 5.4) E
8) E 1.6) E 10.1) A 1.29) B 5.5) D
9.1) C 1.7) F 10.2) D 2.1) A 5.6) D
9.2) A 1.8) A 10.3) F 2.2) D 5.7) B
10.1) C 1.9) B 10.4) C 2.3) E 6) D
10.2) A 1.10) E 10.5) B 3.1) D 7.1) C
10.3) A 2.1) E 10.6) E 3.2) B 7.2) A
10.4) D 2.2) D 11.1) A 4.1) D 8) A
10.5) E 3.1) A 11.2) D 4.2) D 9) D
10.6) B 3.2) C 11.3) C 10) D
10.7) E 3.3) E 12.1) C MSK #6 11.1) B
10.8) E 3.4) A 12.2) B 1.1) A 11.2) D
11.1) A 4.1) E 12.3) D 1.2) D 11.3) A

DO NOT DISTRIBUTE - 114 -


Musculoskeletal 17Mar2009

11.4) C 1.4) E 1.6) B 5.2) C 2) A


11.5) A 1.5) D 1.7) D 5.3) A 3.1) C
11.6) B 1.6) B 1.8) D 5.4) B 3.2) E
12) D 1.7) A 1.9) A 5.5) D 3.3) A
13.1) B 2) C 1.10) B 5.6) E 3.4) B
13.2) A 3) C 1.11) C 5.7) C 3.5) D
13.3) C 4.1) F 1.12) D 5.8) E 4.1) C
13.4) C 4.2) A 1.13) B 5.9) C 4.2) B
13.5) C 4.3) G 1.14) B 5.10) D 5.1) A
13.6) E 4.4) B 1.15) B 6.1) C 5.2) D
14) E 4.5) E 1.16) B 6.2) A 5.3) C
15.1) A 4.6) C 2.1) B 5.4) A
15.2) B 4.7) D 2.2) D MSK #14 5.5) D
15.3) D 5.1) A 2.3) C 1) B 5.6) C
15.4) D 5.2) A 2.4) E 2) D
5.3) B 2.5) C 3) B MSK #16
MSK #7 5.4) C 4.1) J 1.1) A
1.1) D 5.5) E MSK #11 4.2) I 1.2) B
1.2) A 5.6) D 1.1) E 4.3) H 2.1) C
2) E 5.7) a 1.2) D 4.4) G 2.2) E
3.1) B 5.8) B 4.5) F 3) D
3.2) C 5.9) D MSK #12 4.6) E 4.1) A
4.1) D 5.10) A 1.1) C 4.7) D 4.2) E
4.2) A 5.11) A 1.2) A 4.8) C 5) D
5.1) E 5.12) E 1.3) B 4.9) B 6) E
5.2) E 5.13) C 1.4) D 4.10) A 7.1) A
5.3) A 1.5) B 5.1) E 7.2) C
5.4) B MSK #9 1.6) E 5.2) C 7.3) E
5.5) C 1.1) D 1.7) A 5.3) A 7.4) B
5.6) C 1.2) F 2) C 5.4) D 8) C
6.1) D 1.3) E 3.1) B 5.5) B 9.1) D
6.2) E 1.4) D 3.2) D 5.6) B 9.2) A
6.3) C 1.5) C 4) A 5.7) D 9.3) A
6.4) D 1.6) B 5) C 5.8) A 10) C
7.1) B 1.7) A 6) E 5.9) D 11) B
7.2) D 2.1) C 7) A 5.10) A 12.1) A
7.3) A 2.2) C 5.11) E 12.2) D
7.4) A 2.3) D MSK #13 5.12) C 12.3) C
7.5) A 3) A 1.1) B 6) B 12.4) A
7.6) E 1.2) C 7) C 12.5) E
7.7) B MSK #10 2) E 8.1) D 13) C
1.1) C 3.1) A 8.2) E 14.1) E
MSK #8 1.2) E 3.2) B 9) C 14.2) A
1.1) D 1.3) C 4.1) E
1.2) E 1.4) A 4.2) B MSK #15 MSK #17
1.3) B 1.5) E 5.1) D 1) B 1.1) D

DO NOT DISTRIBUTE - 115 -


Musculoskeletal 17Mar2009

1.2) C 25.1) D 10.1) B 25) B 19) B


1.3) E 25.2) D 10.2) D 26) A 20) B
2.1) D 26) E 10.3) A 27) D 21) D
2.2) B 27.1) A 11) C 28) C 22) E
2.3) D 27.2) D 12) B 29) E 23) A
2.4) A 28) C 13) C 30) E 24) C
2.5) C 29) A 14) C 31) E 25) C
3) E 30.1) E 15) E 32) D 26) E
4.1) C 30.2) C 16.1) C 33) A 27) B
4.2) D 31) E 16.2) C 34) B 28) A
5.1) B 32) A 17) C 35) C 29) D
5.2) E 33) B 18.1) D 36) E 30) B
6) E 34) C 18.2) B 37) E 31) D
7.1) C 35) D 19.1) C 38) A 32) D
7.2) D 36) C 19.2) A 39) D 33) B
8.1) C 37) B 20.1) C 40) C 34) A
8.2) F 38) B 20.2) A 41) D 35) E
9.1) B 39) D 20.3) B 42) E 36) D
9.2) A 40.1) A 21) D 43) A 37) E
10.1) D 40.2) B 22) E 44) C 38) B
10.2) D 40.3) C 45) E
11.1) C 41) B MSK #19 46) C MSK #21
11.2) B 42) C 1) D 47) D 1) E
11.3) C 43.1) A 2) C 48) B 2) D
12) E 43.2) A 3) B 49) D 3.1) B
13.1) A 44) D 4) C 50) A 3.2) B
13.2) C 45) C 5) A 3.3) C
14.1) B 46) A 6) B MSK #20 4) D
14.2) A 7) E 1) D 5) A
14.3) E MSK #18 8) D 2) E 6) F
15) D 1.1) C 9) D 3) B 7.1) B
16) E 1.2) A 10) D 4) C 7.2) B
17.1) A 1.3) C 11) B 5) E 8) E
17.2) C 2) B 12) A 6) C
17.3) D 3) C 13) E 7) A MSK #22
17.4) A 4.1) B 14) B 8) B 1) C
17.5) B 4.2) E 15) E 9) D 2) A
18) D 5) A 16) C 10) E 3.1) B
19.1) A 6) C 17) D 11) D 3.2) A
19.2) D 7.1) A 18) E 12) E 4) E
20) B 7.2) A 19) A 13) A 5) B
21) C 8.1) D 20) E 14) C 6) B
22) E 8.2) B 21) A 15) D 7) D
23.1) C 9.1) B 22) B 16) B 8) D
23.2) E 9.2) A 23) D 17) D 9) A
24) C 9.3) C 24) E 18) C 10) D

DO NOT DISTRIBUTE - 116 -


Musculoskeletal 17Mar2009

11) C 9) E MSK #27 8) D 4.1) C


12) A 10) D 1) C 9) A 4.2) B
13) D 11) B 2) E 10) B 4.3) A
14) B 12) C 3) A 5) D
15) C 13.1) A 4) D MSK #32 6) E
16) B 13.2) B 5) D 1) C 7) A
17) B 13.3) C 6) D 2) A 8) B
18) E 14) B 7) D 9) B
19) C 15) E 8) A MSK #33 10) B
20) D 16.1) B 9) E 1) E
21) C 16.2) C 10) D 2) E MSK #41
22.1) B 17) D 11) B 3) D 1) E
22.2) A 18) C 12) B 2) C
22.3) C 19) A 13) C MSK #34 3) D
23) D 20) D 14) C 1) B 4) E
24) A 21) C 15) B 2) A
25) D 22) A 16) D 3) C MSK #42
26) C 23) C 17) A 1) C
18) D MSK #35
MSK #23 MSK #26 19) A 1) B
1) E 1) D 20) C 2) A
2) D 21) D 3) E
MSK #24 3) D 4) D
1.1) C 4) B MSK #28 5) C
1.2) E 5) E 1) E
2) B 6) D 2) A MSK #36
3) A 7) A 3) A 1) B
4) D 8) C 4) D 2) D
5) B 9) C 5) B
6) C 10) A 6) A MSK #37
7) A 11) B 7) E 1) A
8) E 12) D
9) A 13) D MSK #29 MSK #38
10) C 14) C 1) B 1) D
11) B 15) C 2) C
12) B 16) E MSK #30 3) E
17) C 1) E 4) A
MSK #25 18) D
1) C 19) F MSK #31 MSK #39
2) D 20) A 1) C 1) B
3) A 21) C 2) B 2) F
4) D 22) D 3) B
5) C 23) D 4) E MSK #40
6) C 24) B 5) B 1) C
7) A 25) E 6) B 2) C
8) B 7) B 3) A

DO NOT DISTRIBUTE - 117 -

You might also like